Themis MPRE

Ace your homework & exams now with Quizwiz!

Which of the following statements regarding the regulation of lawyers after admission to the state bar is FALSE? A) Before a lawyer may be disciplined for fraud under the rules of professional conduct, a victim must come forward to establish the elements of reliance and damages. B) Procedural fraud includes lying to a tribunal. C) A large number of speeding tickets probably does not reflect poorly on a lawyer's fitness to practice law. D) Fraud or deceit that is noncriminal may lead to discipline under the rules of professional conduct.

A) Before a lawyer may be disciplined for fraud under the rules of professional conduct, a victim must come forward to establish the elements of reliance and damages.

Which of the following statements regarding the attorney-client privilege is FALSE? A) It applies to physical evidence of a crime given to the lawyer by the client. B) It covers testimonial communications made by the client to the lawyer. C) It requires that communications made by the client to the lawyer be made in confidence. D) It does not apply to disputes between the attorney and the client, such as a dispute over fees.

A) It applies to physical evidence of a crime given to the lawyer by the client.

Which of the following statements regarding the lawyer-client relationship is TRUE? A) A lawyer is generally not under a duty to accept representation of a client. B) A lawyer does not have a duty to reject a client. C) Court appointments may never be declined. D) Court appointments may only be declined if representation would violate a rule of professional responsibility.

A) A lawyer is generally not under a duty to accept representation of a client.

Which of the following statements regarding the unauthorized practice of law is TRUE? A) A lawyer may be disciplined in a state for conduct that occurred outside of that state. B) A lawyer may not practice law, even on a temporary basis, in a state in which the lawyer is not licensed. C) A lawyer can practice law in a state in which the lawyer is not licensed if the lawyer associates with local counsel as a silent partner. D) A lawyer cannot engage in ghostwriting for pro se litigants.

A) A lawyer may be disciplined in a state for conduct that occurred outside of that state.

Which of the following statements regarding witnesses is TRUE? A) A lawyer may not tamper with a witness. B) A lawyer may pay any witness a fee for testifying. C) An expert witness may collect a contingency fee. D) A lawyer may not pay for the witness's costs.

A) A lawyer may not tamper with a witness.

A conflict of interest may exist in which of the following circumstances? A) A lawyer represents a client in a case in which the opposing party is represented by the lawyer's spouse. B) A lawyer has negotiated to acquire the client's "story rights" after the conclusion of the representation. C) A lawyer has a pre-existing sexual relationship with a client. D) A lawyer drafts a will for her father in which the lawyer is the primary beneficiary.

A) A lawyer represents a client in a case in which the opposing party is represented by the lawyer's spouse.

FILL IN THE BLANK. Under the ABA Model Rules of Professional Conduct and the ABA Model Code of Judicial Conduct, _______________ must avoid the appearance of impropriety. A) A judge B) A lawyer C) Both a lawyer and a judge D) Neither a lawyer nor a judge

A) A judge

During the trial of an age discrimination lawsuit, both the judge and the attorney referenced the plaintiff's age and political affiliation multiple times in closing argument and jury instructions. The judge stated that "while members of an older generation may be more touchy about being passed over for promotions, since they may feel they are owed more respect for their seniority, the law does not allow for actual discrimination." He also stated that individuals of the plaintiff's political party might face additional challenges in the plaintiff's particular occupational field, which tends to be composed mostly of members of an opposing political party, and that this fact may have made the plaintiff feel alienated by his peers. Is it proper for the judge to reference the plaintiff's age and political affiliation in the above-referenced manner? A) No, as to both age and political affiliation B) Yes, as to both age and political affiliation C) Yes, as to age only D) Yes, as to political affiliation only

A) No, as to both age and political affiliation Answer choice A is correct. In performing judicial duties, a judge must not by words or conduct manifest bias or prejudice. Examples of manifestations of bias or prejudice include, but are not limited to: negative stereotyping; attempted humor based upon stereotypes; suggestions of connections between race, ethnicity, or nationality and crime; and irrelevant references to personal characteristics. The requirements of the Code of Judicial Conduct do not preclude judges or attorneys from making legitimate reference to race, sex, gender, religion, national origin, ethnicity, disability, age, sexual orientation, marital status, socioeconomic status, or political affiliation, or similar factors, when they are relevant to an issue in a proceeding. Here, the judge's reference to age reflected negative stereotyping, and was not permitted. Political affiliation was not relevant at all to an age discrimination issue, so the reference was not a legitimate exercise of judicial authority. Answer choice B is incorrect because the judge's comments for both age and political affiliation were improper. Answer choice C is incorrect because the judge's comments about age reflected inappropriate stereotyping. Answer choice D is incorrect because the judge should not have referenced the plaintiff's political affiliation, which was not a relevant issue.

A for-profit research company uses publicly available information to create client leads for law firms. Law firms pay the company a fixed amount per lead, and none of the company's employees are attorneys. The company does not contact potential clients directly, but instead markets its services solely to law firms, so potential clients are not aware of the company's activities. Is the managing partner of a law firm that pays the company for its leads subject to discipline? A) No, because a law firm is permitted to pay the company to generate client leads. B) No, because the research company does not provide legal services. C) Yes, because a lawyer is not permitted to pay the company for recommending a client to the lawyer. D) Yes, because a managing lawyer is responsible for the conduct of non-lawyers hired by the law firm.

A) No, because a law firm is permitted to pay the company to generate client leads. Answer choice A is correct. A lawyer may pay others for generating client leads, such as Internet-based client leads, if the lead generator does not recommend the lawyer or make false statements about the lawyer's services, and any payment to the lead generator is consistent with the Model Rules. Here, the research company has no contact with the client, and payment by the law firm for the leads does not constitute a sharing of fees earned from any client. In addition, the payment does not compromise the independence of the law firm. Answer choice B is incorrect. A lawyer who has managerial authority over the professional work of a law firm, such as a partner of a partnership, must make reasonable efforts to ensure the firm has measures in place that give reasonable assurance that the conduct of non-lawyers employed by, retained by, or associated with the firm, conforms to the Model Rules, even when the non-lawyers are not engaged in the provision of legal services. Answer choice C is incorrect. There is a general prohibition on a lawyer giving anything of value to a person for recommending the lawyer's services to a potential client, but there is no corresponding prohibition on paying a person for recommending a potential client to the lawyer. In other words, lawyers should not pay to have themselves recommended to the client, but can pay to receive leads about potential clients. Answer choice D is incorrect. A lawyer who has managerial authority over the professional work of a law firm, such as a partner of a partnership, must make reasonable efforts to ensure the firm has measures in place that give reasonable assurance that the conduct of all lawyers in the firm, as well as that of non-lawyers employed by, retained by, or associated with the firm, conforms to the Model Rules. However, the research company, which generates its leads through access to publicly available information, has not engaged in an activity that violates the Model Rules. Answer choice B is incorrect. A lawyer who has managerial authority over the professional work of a law firm, such as a partner of a partnership, must make reasonable efforts to ensure the firm has measures in place that give reasonable assurance that the conduct of non-lawyers employed by, retained by, or associated with the firm, conforms to the Model Rules, even when the non-lawyers are not engaged in the provision of legal services. Answer choice C is incorrect because, although there is a general prohibition on a lawyer giving anything of value to a person for recommending the lawyer's services to a potential client, there is not a corresponding prohibition on a payment made by a lawyer to a person for recommending a potential client to the lawyer (i.e., for providing the lawyer with a lead to a potential client). Answer choice D is incorrect. A lawyer who has managerial authority over the professional work of a law firm, such as a partner of a partnership, must make reasonable efforts to ensure the firm has measures in place that give reasonable assurance that the conduct of all lawyers in the firm, as well as that of non-lawyers employed by, retained by, or associated with the firm, conforms to the Model Rules. However, the research company, which generates its leads through access to publicly available information, has not engaged in an activity that violates the Model Rules.

A plaintiff, who was an attorney, brought an action against a defendant, who was also an attorney, for alleged defamation. The defendant had issued a complaint to the bar association after the defendant personally observed a man he reasonably believed to be the plaintiff visiting a hospital to pass out business cards to known victims of a massive train accident. After an investigation, the bar association determined that the defendant's allegations were without merit. The plaintiff then brought a defamation action against the defendant on the basis of the complaint. Is the plaintiff likely to succeed? A) No, because a privilege against defamation applies to the required reporting of another lawyer's misconduct. B) No, because litigation privilege is an absolute defense to all defamation actions brought against lawyers. C) Yes, because litigation privilege does not generally extend to communications made without the intent of future litigation. D) Yes, because the bar association determined that the defendant's allegations were without merit.

A) No, because a privilege against defamation applies to the required reporting of another lawyer's misconduct. Answer choice A is correct. To protect the reporting lawyer from a retaliatory defamation action by the lawyer engaged in misconduct, a privilege against defamation applies to the required reporting of another lawyer's misconduct. Therefore, the plaintiff's claim is likely to fail. Answer choice B is incorrect because litigation privilege only applies to a narrow subset of communications by lawyers. Litigation privilege is an absolute defense to a defamation action for the publication of a matter relating to a non-client if the publication occurs in communications initiating legal proceedings as long as the lawyer participates as counsel in the proceedings, the matter is published to someone who may be involved in the proceedings, and the publication has some relation to the proceedings. For this reason, answer choice B is incorrect. Answer choice C is incorrect. Although it is true that the litigation privilege does not generally extend to communications made without the intent of future litigation, here we are dealing with required reporting of misconduct, which is always privileged against defamation claims. Answer choice D is incorrect because the privilege against defamation always applies to the required reporting of another lawyer's misconduct, regardless of whether the report results in discipline.

An attorney's childhood friend purchased an antique car many years ago with the intent of slowly restoring it with the help of the attorney. Over the years, the attorney and her friend worked on the car together, took the car to many antique car shows, and went on many road trips together in the car. In addition, the attorney drove the car by herself on a frequent basis. One day, the friend asked the attorney to draft a will for him. Specifically, he asked her to bequeath his car to her because she loved it so much and had taken such good care of it over the years. Per his instructions, the attorney drafted a will that gave his car to her. Did the attorney act properly? A) No, because an attorney cannot draft a will that gives a substantial gift to herself. B) No, because the attorney should have obtained the friend's written consent to having the attorney draft the will and receive the car. C) Yes, because the attorney did not solicit the gift of the car from her friend. D) Yes, because the gift of the car met general standards of fairness.

A) No, because an attorney cannot draft a will that gives a substantial gift to herself. Answer choice A is correct. A lawyer may not solicit a substantial gift or prepare an instrument (such as a will, trust agreement, or deed) that gives a substantial gift to the lawyer or a person related to the lawyer, unless the client is related to the donee. Here, the attorney is not related to her friend, and the car qualifies as a substantial gift. Therefore, the attorney did not act properly by giving herself the car in the will. Answer choice B is incorrect because there is no requirement that an attorney should obtain a client's written consent prior to drafting a will or giving herself a substantial gift in it. Answer choices C and D are incorrect because they state the rule for unsolicited gifts. A lawyer may accept an unsolicited gift from a client, if the transaction meets general standards of fairness. Here, we are dealing with an attorney who is preparing a will that gives a substantial gift to herself, so this rule does not apply to her.

A father and daughter practiced law together in a small law firm for many years. The law firm's name featured both of their names. After 15 years of practicing law together, the father died. The daughter maintained the practice under its current name and rented out her father's office space to a sole practitioner on a yearly basis. The daughter and the sole practitioner kept separate practices, and did not share office expenses, revenues, or clients, but they did enter a referral agreement for the duration of the sole practitioner's lease under which they agreed to refer clients to each other when a client sought representation for a case in the other attorney's area of expertise. The referral agreement was non-exclusive, and each attorney disclosed the agreement to each referred client. The daughter and sole practitioner did not share legal fees and took reasonably adequate measures to protect their confidential client information. The daughter also added the sole practitioner's name to the law firm's name on the sign in front of the law office. Is the daughter's conduct proper under the Model rules? A) No, because attorneys may not hold themselves out as practicing law together in one firm when they are not a firm. B) No, because the father's name should have been removed from the firm's name after he passed away. C) No, because the sole practitioner and the daughter entered an improper referral agreement. D) Yes, the daughter's conduct was proper under the Model rules.

A) No, because attorneys may not hold themselves out as practicing law together in one firm when they are not a firm. Answer choice A is correct. Lawyers may not imply or hold themselves out as practicing together in one firm when they are not a firm. Here, the daughter and the sole practitioner kept separate practices, and did not share office expenses, revenues, or clients. The mere sharing of office space by lawyers is ordinarily insufficient to result in the lawyers being treated as a firm. Therefore, they are not a firm, and should not have held themselves out as practicing together by putting their names together on the sign in front of the law office. For this reason, answer choice D is incorrect. Answer choice B is incorrect. A firm may be designated by the names of deceased members where there has been a succession in the firm's identity. Here, there was a succession in the firm's identity, so the father's name could remain on the sign. Answer choice C is incorrect. Under certain circumstances, a lawyer may agree to refer clients to another lawyer or a non-lawyer professional, in return for the undertaking of that person to refer clients or customers to the lawyer. The reciprocal referral agreement must be neither exclusive nor of indefinite duration, and the client must be informed of the existence and nature of the agreement. Since this agreement met these requirements, the reciprocal referral agreement was proper under the Model Rules.

An attorney worked in a small law office alongside a paralegal and a legal secretary. The attorney specialized in personal injury litigation, and due to her reputation in the community, she had a hefty caseload. As a result, she relied heavily on her legal staff to assist her on her cases. The attorney supervised all work delegated to her non-legal staff, and made reasonable efforts to ensure that the conduct of all of her staff conformed to the Model Rules of Professional Conduct. She also rewarded their hard work by sharing 5% of the law firm's profits with them via a compensation plan in addition to their yearly salary. Is the attorney subject to discipline under the Model Rules? A) No, because sharing profits with legal personnel in this manner is permissible under the Model Rules. B) No, because the paralegal and the legal secretary only received 5% of the law firm's profits via the compensation plan. C) Yes, because the attorney relied heavily on her legal staff to assist her on her cases. D) Yes, because the Model Rules prohibit fee sharing by lawyers with non-lawyers.

A) No, because sharing profits with legal personnel in this manner is permissible under the Model Rules. Answer choice A is correct. The Model Rules generally prohibit fee sharing by lawyers with non-lawyers. However, fee-sharing with non-lawyers is permitted when fees are shared with law firm personnel via a profit-sharing compensation or retirement plan. Here, the attorney shared fees with law firm personnel via a profit-sharing compensation plan, which was permissible under the Model Rules. Therefore, answer choice D is incorrect. Answer choice B is incorrect because there is no maximum amount set forth by the Model Rules as to how much profit-sharing is permitted between a lawyer and a non-lawyer. Answer choice C is incorrect because the Model Rules permit a lawyer to employ the services of paraprofessionals and delegate functions to them, so long as the lawyer supervises the delegated work and makes reasonable efforts to ensure that the conduct of all of her staff conformed to the Model Rules of Professional Conduct.

In an attempt to grow her business, a newly licensed attorney accepted nearly all cases brought to her. After a few months, her workload became so heavy that she did not have sufficient time to devote to the cases, and she subsequently hired a paralegal and a secretary to alleviate some of the work. She also telephoned each of her clients and informed them that her secretary and paralegal would complete much of the legal legwork because she simply did not have time to do so herself. The attorney subsequently missed a major deadline, and a client filed a malpractice suit against her. During discovery, the attorney stated that her paralegal was responsible for tracking deadlines and that her excessive workload prevented her from being able to work full-time on each case herself. Were the attorney's actions with regard to managing her workload proper? A) No, because she had a duty to manage her workload more effectively. B) No, because the attorney did not inform her clients in writing that her staff would take on some of the work. C) Yes, because it was not possible for the attorney to work on all details of each case in light of her workload. D) Yes, because the attorney adequately supervised her staff.

A) No, because she had a duty to manage her workload more effectively. Answer choice A is correct. An attorney cannot justify a lack of diligence based on illness, subordinates, personal animosity, or the inability to balance other work. An attorney must control her workload to ensure that she can handle all matters competently. Here, the attorney took on more cases than she could handle, and she was unable to competently handle her cases, despite the fact that she hired additional staff to compensate for having an unmanageable caseload. Answer choice B is incorrect because an attorney is permitted to employ paraprofessionals, such as secretaries and paralegals, to help the attorney with certain duties, so long as the attorney properly supervises the paraprofessionals. There is no requirement that the attorney inform her clients in writing that she delegated duties to such paraprofessionals. Answer choice C is incorrect because an excessive workload is not a valid excuse to justify a lack of diligence. Answer choice D is incorrect because, although the attorney supervised her staff, she could not allow them to take sole responsibility for all deviations from reasonable diligence.

A plaintiff filed a civil action against a defendant for breach of contract in the amount of $8,000. The defendant hired an attorney to file an answer to the complaint, and the attorney had thirty days within which to file a timely answer. After thirty days passed, the attorney failed to file an answer on behalf of the defendant, and a default judgment in the amount of $8,000 was issued against the defendant. The attorney apologized for his mistake and offered to pay the defendant $11,000 to cover the default judgment in exchange for a release from the defendant of any potential malpractice claim. The attorney advised the defendant in writing to seek independent legal counsel before accepting her settlement offer. The defendant said he would consider her offer. Two months later, the defendant agreed to settle the claim without meeting with independent counsel. The attorney paid the defendant $11,000, and the defendant signed a release form. Subsequently, the defendant paid the default judgment of $8,000. Fearing that her insurance rates would increase, the attorney did not report her mistake and the settlement to her malpractice insurance carrier. Is the attorney subject to discipline? A) No, because the attorney advised the defendant in writing to seek independent legal advice and gave the defendant a reasonable opportunity to seek such advice. B) No, because the settlement offer was higher than the default judgment. C) Yes, because the attorney did not require the client to obtain independent legal counsel before signing the release. D) Yes, because the attorney did not report her mistake and the settlement to her malpractice insurance carrier.

A) No, because the attorney advised the defendant in writing to seek independent legal advice and gave the defendant a reasonable opportunity to seek such advice. Answer choice A is correct. Model Rule 1.8(h)(2) requires a lawyer seeking to settle a claim or potential claim for malpractice liability with an unrepresented client or former client to advise that person in writing of the desirability of seeking the advice of independent legal counsel and ensure that the person is given a reasonable opportunity to do so. Here, this duty was satisfied by the attorney's written advice to the defendant and the passage of a reasonable period of time to seek the advice of independent legal counsel. Answer choice B is incorrect. Once a lawyer has given an unrepresented client or former client the advice required by Model Rule 1.8(h)(2), and a reasonable time has passed before the settlement is concluded, the amount of any settlement is a matter of private agreement and not a matter of professional discipline. Therefore, the fact that the settlement was higher than the default judgment is not determinative. Answer choice C is incorrect. Independent representation is not necessary if the appropriate advice is given and a reasonable time passes before a settlement is concluded. Answer choice D is incorrect. The Model Rules do not require lawyers to carry malpractice insurance, so a lawyer is not subject to discipline for failing to report potential claims or settlements to an insurer.

An attorney who worked at a large law firm decided to volunteer at a legal aid clinic after a devastating hurricane passed through his city, causing many people to lose their homes and livelihoods. At the clinic, attorneys provide short-term limited legal services to people, but there is no expectation that the attorney's representation of a person will go beyond a limited consultation with the attorney. Prior to providing any advice, attorneys secure a person's informed consent regarding the limited scope of representation. While at the clinic, the attorney met with a woman who had been served with an eviction notice by her landlord when she fell behind on her rent after the hurricane closed her business. After securing the woman's informed consent regarding the limited scope of his representation, the attorney offered the woman advice on how to proceed in the face of the eviction notice. Unbeknownst to the attorney, the landlord was a client of the law firm at which he worked, and a partner at the law firm had handled the eviction filings against the woman on behalf of the landlord. Is the attorney subject to discipline? A) No, because the attorney did not know that the representation involved a conflict of interest. B) No, because the attorney secured the woman's informed consent to the limited scope of the representation prior to accepting her as a client. C) Yes, because attorneys should always screen for conflicts of interest before undertaking the representation of a client. D) Yes, because there was an actual conflict of interest between the landlord and the woman.

A) No, because the attorney did not know that the representation involved a conflict of interest. Answer choice A is correct. Under the Model Rules, a lawyer who, under the auspices of a program sponsored by a non-profit organization or court, provides short-term limited legal services to a client without expectation by either the lawyer or the client that the lawyer will provide continuing representation in the matter, is only subject to the Model Rules regarding conflicts of interest if the lawyer knows that the representation of the client involves a conflict of interest for the lawyer or for another lawyer associated with the lawyer in a law firm. Here, the attorney provided short-term legal services through the legal aid clinic to the woman, and there was no expectation that the attorney would provide continuing representation in the eviction matter. As such, the attorney would not be subject to the Model Rules regarding conflicts of interest since the attorney did not know that his representation of the woman involved a conflict of interest with a partner in the law firm. Therefore, the attorney is not subject to discipline for failing to screen for conflicts of interest prior to taking on the representation of the woman. For this reason, answer choice D is incorrect. Answer choice B is incorrect. The fact that the attorney secured the woman's informed consent to the limited scope of the representation prior to accepting her as a client would not protect the attorney from discipline if he knew that the representation of the woman involved a conflict of interest. Answer choice C is incorrect because it is too broad. As stated above, lawyers who provide short-terms legal services through non-profit or court sponsored programs are only subject to the Model rules regarding conflicts of interest if the lawyer knows that the representation of the client involves a conflict of interest.

An attorney who owned her own law practice decided to sell it, move to a neighboring state where she was licensed to practice law, and open up a new law office once she was settled there. A number of lawyers who were interested in purchasing the attorney's law practice reached out to her with offers. After they signed confidentiality agreements, the attorney allowed these lawyers to review her books and client files so that the lawyers could determine the value of her law practice. After the lawyers submitted their final bids, the attorney decided to sell her law practice to the highest bidder. The attorney then told her clients about the proposed sale and their right to retain counsel or take possession of their files. She also told them that she would assume that they consented to the transfer of their files if they did not take any action or object within 120 days of receipt of the notice. Was the attorney's conduct proper? A) No, because the attorney did not obtain the consent of each client before allowing the prospective buyers to review the clients' files. B) No, because the attorney planned to continue practicing law in the neighboring state after selling her law practice. C) Yes, because the attorney gave proper notice to her clients regarding the sale of her law practice. D) Yes, because the attorney required each prospective buyer to sign a confidentiality agreement before sharing client files with them.

A) No, because the attorney did not obtain the consent of each client before allowing the prospective buyers to review the clients' files. Answer choice A is correct. The facts reveal that the attorney breached the attorney's duty to preserve client confidences under Model Rule 1.6(a). Subject to exceptions, Rule 1.6(a) prohibits a lawyer from revealing information relating to the representation of a client without client consent. No exception applies that would allow the attorney to reveal client files to prospective purchasers of her law practice. In the context of sale of a law practice, client consent is required when providing a prospective purchaser access to detailed information relating to the representation, such as the client's file. Answer choice B is incorrect. Model Rule 1.17 does not prohibit a lawyer who sells a law practice from practicing law in another state. However, the fact that the attorney intended to practice in a neighboring state did not justify the attorney's disclosure of detailed client information, including client files. Answer choice C is incorrect because giving clients notice of an impending sale does not satisfy the client consent requirement. Answer choice D is incorrect because requiring each prospective buyer to sign a confidentiality agreement before sharing client files with them does not satisfy the client consent requirement.

An attorney agreed to defend a nonprofit business in a negligence lawsuit at half her usual hourly rate due to the nonprofit status of the business. The fee agreement was properly communicated to the business and agreed to in a signed writing before representation began. The attorney obtained a favorable settlement on behalf of the business and submitted a bill for her fees for 60 hours. However, even at the reduced rates, the business was unable to pay the attorney's fees. Because the business was suffering from financial problems, it was unlikely that it would ever be able to pay the attorney's fees. After considering the futility of bringing a claim against the business to enforce the fee agreement, the attorney decided to release the business from its contractual obligation to pay the bill. The applicable jurisdiction has adopted a rule that requires attorneys to report the extent to which they have performed at least 50 hours of pro bono legal services each year according to the ABA Model Rule on voluntary pro bono service. Would it be proper for the attorney to report the 60 hours as pro bono hours? A) No, because the attorney did not perform these legal services without the expectation of a fee. B) No, because pro bono services must be rendered on behalf of individual persons of limited means. C) Yes, because the attorney did not receive any payment for the work she performed for the business. D) Yes, because the legal services were performed on behalf of a nonprofit business.

A) No, because the attorney did not perform these legal services without the expectation of a fee. Answer choice A is correct. The Model Rules set forth a voluntary professional responsibility for a lawyer to accept representation of clients who are unable to pay, suggesting that lawyers should offer at least 50 hours of pro bono legal services per year. Model Rule 6.1(a) states that the substantial majority of the 50 hours of pro bono legal services must consist of legal services without fee or expectation of fee. In this case, the representation started out with the expectation of a fee. Because the attorney in this situation expected to be paid when she performed these hours of legal work, the attorney cannot report the 60 hours of work for which the business could not pay as pro bono hours. Answer choice B is incorrect because Model Rule 6.1 does consider the provision of legal services to some organizations to be pro bono work. Specifically, pro bono work can include work performed for charitable, religious, civic, community, governmental, and educational organizations in matters that are designed primarily to address the needs of persons of limited means. Answer choice C is incorrect because Model Rule 6.1(a) does not consider whether the attorney actually received payment, but whether the representation was undertaken without an expectation of payment. It would be improper for the attorney to claim that hours of legal work performed with the expectation of a fee constitute pro bono work simply because she has decided not to pursue a legal remedy against a client who cannot pay due to financial trouble. Answer choice D is incorrect. Model Rule 6.1(a)(2) allows a lawyer to provide legal services to organizations in matters that are designed to address the needs of persons of limited means. However, this rule requires that the services be performed with no expectation of a fee, which did not occur here.

An attorney represented a worker at an administrative hearing regarding the worker's right to worker's compensation. At the hearing, the attorney presented testimony by a former co-worker in support of the worker's contentions. The administrative law judge who presided over the hearing rendered a decision in favor of the worker, based to a substantial degree on the former co-worker's testimony. The employer appealed this decision to the state worker's compensation commission. While the appeal was pending, the attorney learned that the former co-worker's testimony was false. The attorney, after unsuccessfully seeking the client's permission to notify the commission of the false testimony, withdrew from representing the client, even though doing so had an adverse effect on the worker. Was the attorney's conduct proper? A) No, because the attorney failed to inform the commission of the co-worker's false testimony B) No, because the attorney could not withdraw from representation of the client due to its adverse effect on the worker. C) Yes, because the proceeding was before an administrative agency rather than a court. D) Yes, because the attorney did not learn of the co-worker's false testimony until after the administrative law judge rendered a decision.

A) No, because the attorney failed to inform the commission of the co-worker's false testimony Answer choice A is correct. Not only is a lawyer prohibited from knowingly offering false evidence, but if a witness called by the lawyer has offered material evidence and the lawyer comes to know of its falsity, the lawyer must also take reasonable remedial measures. Reasonable remedial measures can include confidentially advising the client of the lawyer's duty of candor and urging the client's cooperation with respect to the withdrawal or correction of the false statements or evidence. If that fails, the lawyer must take further remedial action. If withdrawal from the representation is not permitted or will not undo the effect of the false evidence, the lawyer must make such disclosure to the tribunal as is reasonably necessary to remedy the situation, even if doing so requires the lawyer to reveal information that would otherwise be protected as confidential under the Model Rules. Here, the attorney failed to reveal to the commission that the co-worker's testimony was false, which was a substantial factor in the administrative law judge's decision. Answer choice B is incorrect. A lawyer is permitted to withdraw even when there is a material adverse effect on the client's interests if the client persists in a course of action involving the lawyer's services that the lawyer reasonably believes is criminal or fraudulent. Here, the client's refusal to inform the commission of the co-worker's false testimony would constitute sufficient grounds for the attorney's withdrawal. Answer choice C is incorrect because the prohibition on knowingly offering false evidence as well as the duty to take reasonable remedial measures upon learning of the falsity of evidence after it has been admitted apply to tribunal proceedings, which include proceedings before an administrative agency acting in an adjudicative capacity. Here, the administrative law judge was acting in an adjudicative capacity when determining the worker's right to compensation. Answer choice D is incorrect because the duty to take reasonable remedial measures upon learning of the falsity of evidence after it has been admitted applies through the conclusion of the proceeding, which is defined as when a final judgment in the proceeding has been affirmed on appeal or the time for review has passed. Since the worker's compensation commission had not rendered a final judgment on the employer's appeal, the attorney had the duty to inform the commission of the false testimony.

A well-known defense attorney met with a criminal defendant regarding representation of the defendant in a highly publicized case. During the meeting, the attorney told the defendant, who was indigent, that she would represent him if he agreed to grant her movie rights regarding the representation. The client agreed to these terms, and the attorney provided him with a written consent form setting forth the terms of the representation and advising him to seek independent counsel. The attorney met with the defendant several days later, at which time he returned a signed copy of the written consent form. He told her that he had not consulted with another attorney. The attorney succeeded in obtaining an acquittal at trial and began shopping a movie based on the case to television studios shortly thereafter. Were the attorney's actions in securing the movie rights based on the case proper? A) No, because the attorney negotiated for movie rights prior to the conclusion of the representation. B) No, because the defendant did not obtain independent legal counsel before signing the consent form. C) Yes, because the defendant signed a written consent form after being advised of his right to seek independent counsel. D) Yes, because the attorney negotiated for movie rights as a replacement for fees.

A) No, because the attorney negotiated for movie rights prior to the conclusion of the representation. -Read the call of the question first. This is an issue of CONFLICT, specifically, STORY RIGHTS. The rule is: YOU CAN NOT NEGOTIATE STORY RIGHTS UNTIL REPRESENTATION IS OVER. -A) is correct because the negotiation happened BEFORE representation was over. -B) is wrong because story rights is a bright-line rule and this does not apply. -C) is wrong even though conflicts can be waivable, but NOT for story rights. -D) is wrong even though true how the story rights were negotiated, BUT AGAIN, you can't do it until REPRESENTATION IS OVER.

An attorney was hired by the state department of housing to counsel it on contracts regarding low-income housing projects. After the department gave a construction company a contract to build a low-income apartment building, the attorney received an anonymous e-mail from someone claiming to be a department employee. The e-mail stated that the secretary of housing, who oversaw low-income housing contracts, had accepted a bribe when deciding who would receive the contract. The attorney, who was friends with the secretary and had worked with her for many years, investigated the allegations in the e-mail and reasonably concluded that the secretary had not accepted a bribe. For this reason, she did not report the allegations to the state office that investigates suspected wrongdoing by state government employees. Is the attorney subject to discipline? A) No, because the attorney reasonably concluded that the secretary had not accepted a bribe. B) No, because the attorney was not permitted to disclose the allegations which were protected by the professional rule of confidentiality. C) Yes, because the attorney did not report the allegations to the state office that investigates suspected wrongdoing by government employees. D) Yes, because the attorney was friends with the secretary and had worked with her for many years.

A) No, because the attorney reasonably concluded that the secretary had not accepted a bribe. Answer choice A is correct. Model Rule 1.13(b) requires a lawyer for an organizational client to take action if the lawyer knows that a person associated with the organization is engaged in action that is a violation of a legal obligation to the organization, or is a violation of law that could be imputed to the organization, and that is likely to result in substantial injury to the organization. Specifically, the lawyer must refer the matter to higher authority in the organization. Although the taking of bribes by the secretary of the department would qualify as illegal action warranting referral to a higher authority, in this case the attorney did not have knowledge of misconduct. The only suggestion of misconduct was an anonymous tip, which the attorney investigated and determined to be unsupported. Answer choice B is incorrect. Model Rule 1.13(c) permits a lawyer to reveal information relating to the representation of an organizational client to the extent necessary to prevent substantial injury to that client. As a result, a lawyer could be subject to discipline for failing to disclose confidential information if disclosure were required to prevent wrongdoing that would result in a substantial injury to the organization. The rule applies, however, only when a lawyer has made the efforts required by Model Rule 1.13(b) to have the highest authority that can act on behalf of the organization take action to rectify a known wrongdoing. Because the attorney did not have knowledge of wrongdoing, the attorney was not required to notify a higher authority and was not permitted to reveal confidential client information. Answer choice C is incorrect. Model Rule 1.13(c) permits a lawyer who represents an organizational client to reveal information related to the representation, such as to a state office that investigates wrongdoing by state employees, when the lawyer reasonably believes that disclosure is necessary to prevent substantial injury to the organization. However, the lawyer may do so only if two requirements are met. First, the lawyer must know that a constituent of the organization has engaged in action that is a violation of a legal obligation to the organization, or is a violation of law that could be imputed to the organization, and that is likely to result in substantial injury to the organization. Second, the lawyer must inform higher authorities in the organization about the matter and the higher authorities must fail to act on the lawyer's information. Neither requirement is met here. Answer choice D is incorrect because it implies that a lawyer may be subject to discipline for being too close to a constituent of an organizational client. However, in this case, the attorney investigated the allegations in the e-mail and reasonably concluded that the secretary had not taken a bribe.

An attorney represented a client in the purchase of a car wash. The attorney suspected that the client was laundering money through another car wash that she already owned by creating fake customers and tacking on extra services to legitimate transactions. The attorney asked her client whether she was conducting a legitimate business, but the client's answers were vague, and she refused to tell the attorney any information that would alleviate the attorney's concerns. Although he could not confirm that his client was laundering money, the attorney reasonably believed that the client wanted to acquire another car wash for the purpose of expanding a money laundering operation. For this reason, the attorney refused to assist the client any further in acquiring the car wash and withdrew from the representation. As a result, another buyer purchased the car wash. Is the attorney subject to discipline? A) No, because the attorney was allowed to withdraw from representation to avoid possibly assisting in the commission of a crime. B) No, because the attorney was required to withdraw from representation to avoid possibly assisting in the commission of a crime. C) Yes, because the attorney did not know that the purchase of the car wash would involve the commission of a crime. D) Yes, because the attorney materially prejudiced his client when he abruptly stopped working on the purchase of the car wash.

A) No, because the attorney was allowed to withdraw from representation to avoid possibly assisting in the commission of a crime. Answer choice A is correct. Model Rule 1.16(b)(2) permits a lawyer to withdraw from a representation if a client persists in a course of action involving the lawyer's services that the lawyer reasonably believes is criminal or fraudulent. Here, the attorney reasonably believed that the client's purchase of the car wash would involve a crime, so the attorney was permitted to withdraw. Answer choice B is incorrect. Model Rule 1.16(b)(2) permits, but does not require, withdrawal under these circumstances. Withdrawal is required only if the lawyer knows that continued representation will assist a crime or fraud. Answer choice C is incorrect because actual knowledge of a crime or fraud is not required to permit withdrawal from representation. Here, the attorney's reasonable belief that the client's purchase of the car wash would involve a crime was sufficient to permit the attorney to withdraw. Answer choice D is incorrect. Under Model Rule 1.16(b)(1), a lawyer may withdraw, without cause, from a representation only if the withdrawal can be accomplished without material adverse effect on the interests of the client. Here, the attorney had cause to withdraw: the reasonable belief that the client intended to use the attorney's services to violate money-laundering laws. As a result, withdrawal was permitted without regard to any potential adverse effect on the client's interests.

An attorney was retained to represent a client charged with assault. The attorney interviewed several of the client's friends and colleagues in a search of character witnesses. In one such interview, the client's secretary revealed that the client, who was a banker, often skirted legal and ethical lines. After the client was acquitted, the attorney's friend called to congratulate him. The attorney mentioned that it might not be long before the client found himself in legal trouble again, and he told his friend about the conversation with the client's secretary. Was the attorney's action in revealing what he learned from the client's secretary proper? A) No, because the attorney was bound by the duty of confidentiality. B) No, because the conversation with the secretary was protected by attorney-client privilege. C) Yes, because the attorney's representation of the client had concluded. D) Yes, because the conversation was not protected by the attorney-client privilege.

A) No, because the attorney was bound by the duty of confidentiality. -Read the call of the question first. This is a issue of CONFIDENTIALITY. -B) is wrong because privilege is about SHIELDING INFORMATION from the court. -C) is wrong because confidentiality it LASTS FOREVER (even after if the attorney or client is dead) UNLESS it gets WAIVED -D) is wrong because this not an issue about privilege, this is a ISSUE OF CONFIDENTIALITY.

An attorney represented a factory worker who was administratively challenging his employer's denial of his worker's compensation claim. A factory co-worker who had witnessed the incident that led to the worker's injury, but otherwise was not involved, contacted the attorney during his investigation of the incident. The co-worker had no supervisory powers in the employer's business and no authority to act as an agent for the business. The attorney ensured that the co-worker understood who the attorney represented, but did not notify the attorney for his client's employer about the contact. The co-worker provided information that supported the worker's compensation claim. Assuming that the attorney violated no duties to his client, is the attorney subject to discipline for communicating with the co-worker regarding the case? A) No, because the attorney was permitted to communicate freely with this co-worker about the incident. B) No, because the co-worker initiated the contact with the attorney. C) Yes, because the attorney failed to notify the employer's attorney before interviewing the co-worker. D) Yes, because the co-worker provided information that supported the worker's compensation claim.

A) No, because the attorney was permitted to communicate freely with this co-worker about the incident. Answer choice A is correct. In representing a client, a lawyer is not permitted to communicate about the subject of the representation with a person the lawyer knows to be represented by another lawyer in the matter, unless the lawyer has the consent of the other lawyer or is authorized to do so by law or a court order. In the case of a represented organization, a lawyer is prohibited from communicating with a constituent (e.g., employee) of the organization who supervises, directs, or regularly consults with the organization's lawyer concerning the matter or has authority to obligate the organization with respect to the matter, or whose act or omission in connection with the matter may be imputed to the organization for purposes of civil or criminal liability. Here, since the co-worker does not fall within one of the prohibited classes, the attorney may communicate with the co-worker without first notifying the employer's attorney or securing that attorney's consent. Answer choice B is incorrect because the restriction on communicating with a constituent of an organization applies even when the constituent initiates the communication. Answer choice C is incorrect because, as noted regarding answer choice A, the restriction on communicating with a constituent of an organization does not apply unless the constituent supervises, directs, or regularly consults with the organization's lawyer concerning the matter or has authority to obligate the organization with respect to the matter, or whose act or omission in connection with the matter may be imputed to the organization for purposes of civil or criminal liability. Here, since the co-worker does not fall within one of the prohibited classes, the attorney may communicate with the co-worker without first notifying the employer's attorney or securing that attorney's consent. Answer choice D is incorrect because the restriction on communicating with a constituent of an organization is not keyed to whether the constituent provides information that is harmful to the organization.

A plaintiff was suing a defendant golf course in a personal injury action to recover for injuries she suffered after she slipped and fell on some loose steps on the golf course. Before the case went to trial, the two parties entered into settlement negotiations. During the negotiations, the plaintiff's mother unexpectedly passed away, so the plaintiff became busy with funeral arrangements. As a result, she was unavailable to actively participate in the settlement negotiations. However, the plaintiff authorized her attorney to accept a settlement offer of $1 million or more from the defendants, and to reject any settlement offers that were below that amount. The plaintiff did not put this authorization in writing. One day later, the defendant made a settlement offer of $950,000 to the attorney. The attorney did not communicate the offer to the plaintiff. Instead, the attorney rejected the settlement offer. Is the attorney subject to discipline? A) No, because the client authorized the attorney to reject settlement amounts below $1 million. B) No, because the ultimate decision as to whether to accept a settlement offer rests with the attorney. C) Yes, because the attorney failed to communicate the settlement offer to the client. D) Yes, because the plaintiff did not put the authorization to accept the settlement offer in writing.

A) No, because the client authorized the attorney to reject settlement amounts below $1 million. Answer choice A is correct. Generally, a lawyer must communicate all bona fide offers of settlement to the client, and the ultimate decision as to whether to accept rests with the client. An exception exists when the client has authorized the lawyer to accept or to reject the offer. Here, the plaintiff authorized her attorney to accept a settlement offer of $1 million or more from the defendants, and to reject any settlement offers that were below that amount. Per those instructions, the attorney rejected the offer of $950,000. Therefore, the attorney is not subject to discipline. Answer choice B is incorrect. The ultimate decision as to whether to accept a settlement offer rests with the client, not the attorney. Answer choice C is incorrect. Although a lawyer generally must communicate all bona fide offers of settlement to the client, an exception exists when the client has authorized the lawyer to accept or to reject the offer. Answer choice D is incorrect. There is no requirement that the client's advance authorization be in writing.

An individual is suspected of the murder of his spouse. The prosecutor has convened a grand jury to investigate the murder. The prosecutor has called the spouse's attorney to testify before the grand jury about a conversation between the attorney and the spouse shortly before the spouse was killed. The prosecutor has reason to believe that the spouse was seeking the attorney's advice about obtaining a divorce. The prosecutor is aware that the attorney-client privilege applies to the conversation, but reasonably believes that the conversation is evidence that is essential to the successful completion of an ongoing investigation and that there is no other feasible alternative to obtain the information. Is the prosecutor entitled to receive this information? A) No, because the conversation is protected by the attorney-client privilege. B) No, because the attorney's duty of confidentiality applies to the conversation. C) Yes, because the prosecutor reasonably believes that the conversation is evidence that is essential to the successful completion of an ongoing investigation. D) Yes, because the prosecutor reasonably believes that there is no other feasible alternative to obtain the information.

A) No, because the conversation is protected by the attorney-client privilege. Answer choice A is correct. Under Rule 3.8(e), a prosecutor is not permitted to subpoena a lawyer in a grand jury or other criminal proceeding to present evidence about a past or present client unless the prosecutor reasonably believes that: (i) the information sought is not protected from disclosure by any applicable privilege; (ii) the evidence sought is essential to the successful completion of an ongoing investigation or prosecution; AND (iii) there is no other feasible alternative to obtain the information. Since the prosecutor knows that the conversation between the attorney and the spouse was protected by the attorney-client privilege, the prosecutor may not call the attorney to testify. Answer choice B is incorrect because, although the attorney's duty of confidentiality does apply to attorney's conversation with the spouse, this duty is not directly relevant to whether the prosecutor can subpoena the attorney to testify about the conversation. If the attorney-client privilege did not apply, the prosecutor could seek a court order to compel the attorney to testify, even as to confidential information. Answer choice C is incorrect because, even though the prosecutor reasonably believes that the evidence sought is essential to the successful completion of the ongoing investigation or prosecution, the prosecutor cannot reasonably believe that the attorney-client privilege does not protect the conversation between the attorney and the spouse from disclosure. Answer choice D is incorrect because, even though the prosecutor reasonably believes that there is no other feasible alternative to obtain the information, the prosecutor cannot reasonably believe that the attorney-client privilege does not protect the conversation between the attorney and the spouse from disclosure.

An international corporation that produced and distributed coffee beans globally was headquartered in the United States. The corporation employed an attorney as the chief legal officer of the corporation's legal department. The attorney was licensed in the state where the corporation was headquartered. In her role as chief legal officer, the attorney was authorized to hire and supervise lawyers for the legal department. Because the corporation intended to develop its business in a foreign country, the attorney hired a foreign lawyer who was licensed only in that country. The foreign lawyer is a member in good standing of the legal profession in that country and is subject to regulation by that country's disciplinary authority. After a few years, the foreign lawyer moved to the United States and established a continuous and systematic presence in the state where the corporation is headquartered. The state allows foreign lawyers who have a master's degree to take the bar exam. However, the foreign lawyer has not applied for a license to become a lawyer in the state. The foreign lawyer's responsibilities are limited to reviewing the company's employment and export contracts with the foreign country where he is licensed to practice for compliance with the laws of the foreign country. The foreign lawyer does not work on any legal matters involving United States law. Is the attorney subject to discipline for employing the foreign lawyer? A) No, because the foreign lawyer is licensed in the foreign country and provides legal counsel to the corporation only on matters related to the foreign country's laws. B) No, because the state disciplinary authority has no jurisdiction over a foreign lawyer practicing only foreign law within its territory. C) Yes, because the foreign lawyer has established a systematic and continuous presence in the state. D) Yes, because the foreign lawyer has not sought to become licensed in the state where the corporation is headquartered.

A) No, because the foreign lawyer is licensed in the foreign country and provides legal counsel to the corporation only on matters related to the foreign country's laws. Answer choice A is correct. Model Rule 5.5 states that a foreign lawyer may provide legal advice to the lawyer's employer or its organizational affiliates, and the legal services provided by the lawyer are not services for which the jurisdiction requires pro hac vice admission. Note that a foreign lawyer must be a member in good standing of a recognized legal profession in a foreign jurisdiction, the members of which are subject to effective regulation and discipline by a duly constituted professional body or public authority. Here, the foreign lawyer is a member in good standing of a recognized legal profession in the foreign jurisdiction. Therefore, the attorney is not assisting in the unauthorized practice of law by hiring the foreign lawyer to provide legal counsel to the corporation on matters related to the foreign country's laws. Answer choice B is incorrect. A lawyer is prohibited from engaging in the unauthorized practice of law or assisting another person engaged in the unauthorized practice of law. Therefore, it is irrelevant whether the state disciplinary body has jurisdiction over the foreign lawyer. The relevant question is whether the attorney is subject to discipline for hiring the foreign lawyer. Answer choice C is incorrect. As long as the lawyer meets the conditions of Model Rule 5.5, she may provide legal services through an office or other systematic and continuous presence in the jurisdiction without being subject to discipline. Therefore, the attorney is not subject to discipline for hiring the foreign lawyer, even though the foreign attorney has established a systematic and continuous presence in the state. Answer choice D is incorrect. Because the conditions under Model Rule 5.5 are met, the foreign lawyer need not be licensed in the state where the services are performed or where the corporation is located. Further, the propriety of the attorney's hiring the foreign lawyer does not turn on whether the foreign lawyer is licensed in the state where the corporation is headquartered.

Three lawyers, Alpha, Beta, and Delta, formed a partnership to practice law with offices in both State First and State Second. Alpha is admitted to practice only in State First. Beta is admitted to practice only in State Second, and Delta is admitted to practice in both State First and State Second. The following letterhead is on stationery used by their offices in both states: Alpha, Beta, and Delta Attorneys at Law 100 State Street City, State First (200) 555-5555 200 Bank Building City, State Second (202) 555-5555 Attorney Alpha Admitted to practice only in State First Attorney Beta Admitted to practice only in State Second Attorney Delta Admitted to practice in State First and State Second Are the members of the partnership subject to discipline? A) No, because the letterhead states the jurisdictions in which each partner is admitted. B) Yes, because there is no jurisdiction in which both Alpha and Beta are admitted to practice. C) Yes, because the firm name used by each office contains the name of a lawyer not admitted to practice in that jurisdiction. D) Yes, unless Delta actively practices law in both State First and State Second.

A) No, because the letterhead states the jurisdictions in which each partner is admitted. Answer choice A is correct. Under MRPC 7.5(b), firms with offices in more than one jurisdiction may use the same name in each of the jurisdictions in which they operate. Such firms must indicate which partners are not licensed to practice in the particular jurisdiction. This letterhead complies with the rule, so the partners are not subject to discipline. Answer choice B is not correct, as it does not matter that there is no jurisdiction in which both Alpha and Beta are admitted to practice. MRPC 7.5(b) simply requires the partners to indicate on the letterhead where they are not admitted to practice. Answer choice C is not correct, as MRPC 7.5(b) allows firms with offices in more than one state to use the same name in each state, so long as the letterhead indicates which partners are not licensed in each particular jurisdiction. Answer choice D is not correct, as Delta need not actively practice in both states. MRPC 7.5(b) controls.

A former employee brought a discrimination lawsuit against her former employer contending that she was dismissed because of her gender. The former employer's in-house counsel attended the meeting at which the former employee was fired. During the meeting, the counsel created a written, verbatim transcript of the meeting. This transcript was created the ordinary course of business and not in anticipation of any litigation regarding events at the meeting. The former employee has made a discovery request for this transcript, but the request did not establish a substantial need for the transcript. The in-house counsel has resisted on the grounds of the work product doctrine. Is it proper for the in-house counsel to refuse to turn over the transcript? A) No, because the transcript was prepared in the ordinary course of business. B) No, because the transcript does not contain the attorney's thoughts or mental impressions. C) Yes, because the discovery request failed to establish a substantial need for the transcript. D) Yes, because the in-house counsel prepared the transcript in his role as the former employer's attorney.

A) No, because the transcript was prepared in the ordinary course of business. Answer choice A is correct. Documents prepared by a lawyer for the lawyer's own use in anticipation of litigation, including summaries of or notes regarding a witness's statement, are protected under the "work product" doctrine. However, a document prepared in the ordinary course of business rather than in anticipation of litigation it is not subject to protection under the "work product" doctrine. Since the transcript in question was prepared by the in-house counsel in the ordinary course of business, it was not protected by the "work product" doctrine. Therefore, it is improper for the in-house counsel to refuse to turn it over on that basis. Answer choice B is incorrect because, while the mental impressions, conclusions, and trial tactics of a lawyer are protected from discovery, regardless of another party's need for the information or inability to otherwise obtain it, a document that is prepared in anticipation of litigation is nevertheless protected by the "work product" doctrine unless the party seeking disclosure (i) demonstrates a substantial need for the information, and (ii) cannot obtain the information by any other means without undue hardship. Therefore, even if the transcript had not been prepared in the ordinary course of business, the fact that this document did not contain the attorney's thoughts or impressions would not be determinative to whether it was proper for the in-house counsel to refuse to turn it over to the former employee. Answer choice C is incorrect. When a document is prepared in the ordinary course of business rather than in anticipation of litigation, it is not subject to protection under the "work product" doctrine. Consequently, the former employee was not required to make a showing of substantial need and undue hardship in order to be entitled to the transcript. Answer choice D is incorrect because not all documents prepared by a lawyer for a client are protected by the "work product" doctrine. Since the transcript in question was not prepared in anticipation of the litigation, it is not protected by the "work product" doctrine.

An attorney who was representing a wife in a divorce e-mailed the husband's attorney with a property settlement offer. A few hours later, the wife's attorney inadvertently received an e-mail directly from the husband. The e-mail laid out the husband's reaction to the wife's settlement offer, what aspects of the offer he was willing to accept, and what elements of the offer he planned on challenging. After reading the e-mail, the wife's attorney promptly notified the husband's attorney that the husband had mistakenly sent the e-mail to him. The wife's attorney also disclosed the contents of the husband's e-mail to the wife. The husband and wife subsequently agreed to a settlement of how to divide their assets, and the wife's attorney's use of the information from the husband's e-mail did not ultimately harm the husband during the negotiation process. Is the wife's attorney subject to discipline? A) No, because the wife's attorney promptly notified the husband's attorney about the inadvertently sent e-mail. B) No, because the wife's attorney's use of the information in the inadvertently sent e-mail did not ultimately harm the husband. C) Yes, because the wife's attorney read the inadvertently sent e-mail. D) Yes, because the wife's attorney disclosed the contents of the husband's e-mail to the wife.

A) No, because the wife's attorney promptly notified the husband's attorney about the inadvertently sent e-mail. Answer choice A is correct. The attorney acted in accordance with the requirements under Model Rule 4.4(b). That rule simply requires a lawyer to promptly notify a sender when the lawyer receives a document or electronically stored information relating to the representation of the lawyer's client and the lawyer knows or reasonably should know that the document or electronically stored information was inadvertently sent. Because the wife's attorney knew that the husband was represented by a lawyer, the attorney was required to notify the husband's lawyer, which he did here. Answer choice B is incorrect. The propriety of the wife's attorney's conduct does not turn on the fact that the information did not hurt the person who inadvertently sent the document. Answer choice C is incorrect. Model Rule 4.4(b) does not prohibit lawyers from reviewing the contents of inadvertently sent documents. Some lawyers may choose to return a document or delete electronically stored information unread, but a lawyer's decision to do so is a matter of professional judgment ordinarily reserved to the lawyer. Answer choice D is incorrect. Model Rule 4.4(b) does not prohibit a lawyer from sharing information contained in inadvertently sent documents or electronically stored information with the lawyer's client.

An attorney represented an incorporated delivery business in a negligence lawsuit stemming from an accident involving a driver employed by the business. The attorney was selected and directed by the president and sole shareholder of the business. After interviewing another employee who witnessed the accident, the attorney engaged in sexual relations with this witness-employee. The attorney did not discuss his professional responsibility obligations regarding sexual relations with the witness-employee, and the witness-employee had no further involvement with the negligence lawsuit. Is the attorney subject to discipline for engaging in sexual relations with the witness-employee? A) No, because the witness-employee did not supervise, direct, or regularly consult with the attorney concerning the delivery business's legal matters. B) No, because the conflict of interest arising from sexual relations with a client does not apply when the client is an entity rather than an individual. C) Yes, because the attorney engaged in a sexual relationship with an employee of the attorney's client. D) Yes, because the attorney did not seek the informed consent of the witness-employee before engaging the sexual relations.

A) No, because the witness-employee did not supervise, direct, or regularly consult with the attorney concerning the delivery business's legal matters. Answer choice A is correct. An attorney is generally prohibited from engaging in sexual relations with a client. When the client is an organization, this prohibition applies to an employee of the client who supervises, directs, or regularly consults with the attorney concerning the organization's legal matters. Since the witness-employee was not such an employee, the prohibition against sexual relations does not apply. Answer choice B is incorrect because the prohibition of engaging in sexual relations with a client may apply to employees of a client that is an organization. Answer choice C is incorrect because, although an attorney may be subject to discipline for engaging in a sexual relationship with an employee of a client, a conflict of interest does not arise with respect to all employees, but only those employees who supervise, direct, or regularly consult with the attorney. Answer choice D is incorrect because a client cannot waive this prohibition through informed consent.

A partner in a law firm was a witness to an assault that occurred in his neighborhood. The partner will likely have to testify in the civil action brought by the victim of the assault for damages. An associate at the partner's firm has agreed to represent the victim in the assault action. Is the associate subject to disqualification? A) No, because there is no conflict of interest. B) No, because the action is not a criminal case. C) Yes, because the partner is likely to be called as a witness. D) Yes, because the possibility of the partner being called as a witness creates a conflict that is imputed to the associate.

A) No, because there is no conflict of interest. Answer choice A is correct. An attorney is permitted to act as an advocate in a trial in which another attorney in his firm is likely to be called as a witness, unless the conflict-of-interest rules otherwise preclude the attorney from doing so. Here, the facts do not indicate that there is any conflict, and the fact that the partner may be called as a witness does not prohibit a member of his firm from representing that client. Answer choice B is incorrect because the advocate-witness rule applies to civil as well as criminal actions. Answer choice C is incorrect because, unless some other conflict would prevent the associate from representing the client, the likelihood of the partner being called as a witness would not prohibit the associate from serving as the client's advocate. Answer choice D is incorrect because absent a conflict of interest regarding another matter, the partner's disqualification from serving as an attorney for the victim is not imputed to an attorney who is a member of the partner's law firm.

A testator hired a new attorney to prepare a codicil to a previously drafted will. The testator has been referred to the attorney by the testator's niece, whom the attorney currently represents in negotiations to purchase land as an investment. At their first meeting, the testator told the attorney that her niece was named as the residuary beneficiary in the previously drafted will, but the testator now wants to leave all of her property to a charity. If the testator had no will, the niece would be entitled to her aunt's property under the laws of intestate succession. The attorney suggests that the testator draft a new will instead of a codicil, but otherwise follows her directions in drafting the will. The attorney does not give and the testator does not ask for any additional advice regarding the testator's wish to disinherit her niece. The attorney's fee is paid in full by the testator. The attorney does not reveal the substance of the will to the niece or otherwise discuss his representation of the aunt in this matter. Is the attorney subject to discipline for drafting the aunt's will? A) No, because there is not a conflict of interest between the attorney's two clients. B) No, because the aunt left her estate to charity. C) Yes, because the niece recommended the attorney to her aunt. D) Yes, because the aunt disinherited her niece, who is a current client of the attorney.

A) No, because there is not a conflict of interest between the attorney's two clients. Answer choice A is correct. A lawyer must not represent a client when doing so is directly adverse to the interests of another client or there is a significant risk that the representation of the client will be materially limited by the lawyer's responsibilities to the current client, without each client's informed consent. MRPC 1.7. For direct adversity to exist between clients, there must be a conflict as to the legal rights and duties of the clients. Here, the niece did not have a legal claim to inherit her aunt's estate; she had a mere expectancy. Therefore, because the attorney has not acted adversely to the niece's legal rights by drafting a will that left no property to the niece, there is no conflict of interest between these two clients. Answer choice B is incorrect because the charitable nature of the beneficiary of the will is irrelevant as to whether the attorney breached his ethical duties to his clients. Answer choice C is incorrect. A lawyer must not allow a person who recommends his legal services to another to direct or regulate the lawyer's professional judgment in rendering those services (MRPC 5.4(c)). However, the will drafted here by the attorney did not favor the niece. Instead, it resulted in her being disinherited. Therefore, there is no evidence that the niece's wishes controlled the attorney's professional judgment in this case. Answer choice D is incorrect because a lawyer who drafts a will does not, by doing so, owe any duties to a potential beneficiary other than to effect the testator's intent. Moreover, as noted with respect to answer choice A, any duties owed by the attorney to the niece with regard to a transaction to purchase land did not pose a significant risk that the attorney would be unable to perform his duties for the aunt.

Which of the following statements regarding the ABA Model Rules of Professional Conduct is FALSE? A) The Rules within the Model Rules are merely suggestions. B) The Comments to the Model Rules are merely suggestions. C) Violation of a mandatory rule can lead to discipline. D) Aspirational rules tell lawyers what they should do.

A) The Rules within the Model Rules are merely suggestions.

A lawyer may generally share fees with which of the following? A) The estate of a deceased lawyer as a death benefit B) A non-lawyer who brings cases to the lawyer C) A private detective hired for a divorce case D) A paralegal who works in the lawyer's law firm

A) The estate of a deceased lawyer as a death benefit

Which of the following statements regarding recusal of a judge is FALSE? A) The rule of necessity requires the recusal of a judge in circumstances when it otherwise would not be required. B) Recusal may be waived by the parties, even in cases of mandatory recusal. C) A judge must recuse herself from a case over which she has previously presided in another court. D) A judge must recuse himself when his spouse has an economic interest in the subject matter of the controversy.

A) The rule of necessity requires the recusal of a judge in circumstances when it otherwise would not be required.

A client has retained an attorney to represent him in a contract suit. The attorney's retainer agreement provided that the attorney's fees would be based on a fixed hourly rate, payable at the end of each calendar month. Two months before trial, the client fell behind in the payment of the attorney's monthly billing for fees. The attorney included the following statement on his last billing to the client: "Your account is more than 30 days past due. If amounts due are not paid promptly in accordance with our agreement, I will terminate the representation. If you cannot pay the amount due, I will accept an assignment of your cause of action as security for your fee to me." Two weeks after the last billing, the attorney telephoned the client and told him that he would withdraw from representation if the bill was not paid within 48 hours or adequate security given for its payment. If the bill remains unpaid or unsecured after forty-eight hours, it would be proper for the attorney to: A) Upon notice to the client, move the court for permission to withdraw. B) Turn the client's file over to another experienced lawyer in town and notify the client that he no longer represents him. C) Accept an assignment of the client's cause of action as security for his fee. D) All of the above are proper.

A) Upon notice to the client, move the court for permission to withdraw. Answer choice A is correct. A lawyer is permitted to move for the court's permission to withdraw from representing a client in a case when there has been non-payment of fees. Answer choice B is not proper, as MRPC 1.6 provides that a lawyer is prohibited from disclosing information relating to the representation of a client unless the client gives informed consent. Here, turning over the client's file would violate that rule. Answer choice C is not proper, as a lawyer must not obtain a proprietary interest in the cause of action or subject matter of litigation in which a client is represented, except when: (i) the lawyer acquires a lien granted by law to secure payment of a fee, or (ii) the lawyer contracts for a reasonable contingent fee, provided the case is not a criminal or matrimonial/domestic matter. MRPC 1.8(i). Answer choice D is incorrect because it includes two improper courses of action.

An attorney was subpoenaed by a grand jury and asked questions regarding the whereabouts of her client. The attorney considered whether the lawyer-client privilege applied to the revelation of this information. Determining that it was more likely than not that the privilege did not apply, the attorney answered the grand jury questions. Is the attorney subject to discipline for disclosing her client's whereabouts? A) Yes, because the attorney failed to assert a claim that the lawyer-client privilege applied. B) Yes, because an attorney may not reveal the whereabouts of her client. C) No, because the attorney determined that it was more likely than not that the lawyer-client privilege did not apply. D) No, because the duty of confidentiality does not apply when an attorney is called as a witness.

A) Yes, because the attorney failed to assert a claim that the lawyer-client privilege applied. Answer choice A is correct. While there is an exception to the duty of confidentiality if the lawyer reasonably believes it necessary to comply with a court order or other law, this exception states that the lawyer should assert all non-frivolous claims that the information sought is protected against disclosure by the lawyer-client privilege or other applicable law. Even though the attorney did not think that the attorney-client privilege applied, asserting the claim on the client's behalf would have been non-frivolous. Answer choice B is incorrect because, while the client's whereabouts is typically information protected by the duty of confidentiality, a lawyer is permitted to reveal such information to the extent that the lawyer reasonably believes it necessary to comply with the law or a court order. Answer choice C is incorrect because the "compliance with other law or court order" exception to the duty of confidentiality does not apply unless the lawyer has first asserted all non-frivolous claims that the information sought is protected against disclosure by the lawyer-client privilege or other applicable law. Answer choice D is incorrect because there is no blanket exception to the duty of confidentiality when a lawyer is called as a witness.

An attorney agrees to represent a defendant in a criminal prosecution in federal district court. The prosecutor seeks to disqualify the attorney based on the attorney's participation in the indictment of the defendant. While the federal district court has adopted most of the ethical rules of the state in which the court is located, the court has modified the relevant rule. Under the state ethical rule, the attorney would be subject to disqualification, but under the version of this rule adopted by the federal court, the attorney would not be disqualified. Can the federal court rely on its version of the relevant ethical rule in determining whether to disqualify the attorney? A) Yes, because each federal court has the inherent authority to regulate who may appear before it. B) Yes, because the federal rule would permit the criminal defendant to be represented by the lawyer of his choice. C) No, because only the U.S. Supreme Court can adopt ethical rules governing the practice of law in federal courts. D) No, because a federal court must apply the legal ethical rules of the state in which it sits.

A) Yes, because each federal court has the inherent authority to regulate who may appear before it. Answer choice A is correct. Each federal court has the inherent authority to regulate lawyers who appear before it. Answer choice B is incorrect because the Sixth Amendment Right to Counsel does not negate the authority of trial courts to establish criteria for admitting lawyers to argue before them. Answer choice C is incorrect because neither a federal statute nor a U.S. Supreme Court order mandates that the ethical rules for lawyers appearing before a federal court be promulgated by the U.S. Supreme Court. Answer choice D is incorrect because, although many federal courts do, a federal court is generally not required to apply the ethical rules of the state in which the court sits.

A certified public accountant has proposed to an attorney, a recognized specialist in the field of tax law, that she and the attorney form a partnership for the purpose of providing clients with tax-related legal and accounting services. Both the accountant and the attorney have deserved reputations of being competent, honest, and trustworthy. The accountant further proposes that the announcement of the proposed partnership, the firm stationery, and all public directory listings clearly state that the accountant is a certified public accountant and that the attorney is a lawyer. Is the attorney subject to discipline if he enters into the proposed partnership with the accountant? A) Yes, because one of the activities of the partnership would be providing legal services to clients. B) Yes, because the attorney would be receiving fees paid for other than legal services. C) No, because the partnership will assure to the public high-quality services in the fields of tax law and accounting. D) No, if the attorney is the only person in the partnership who gives advice on legal matters.

A) Yes, because one of the activities of the partnership would be providing legal services to clients. Answer choice A is correct. Under MRPC 5.4(b), a lawyer is prohibited from entering into a partnership with a non-lawyer if any of the partnership's business will include the practice of law. Answer choice B is not correct, as the Model Rules do not prohibit a lawyer from receiving fees for services other than legal services. They do, however, prohibit a lawyer from entering into a partnership with a non-lawyer who will share in the fees from the practice of law. Answer choice C is not correct, as it is irrelevant whether the partnership will assure to the public high-quality services in both tax law and accounting. The Model Rules prohibit a partnership between a lawyer and a non-lawyer when any of the partnership's business will include the practice of law. Answer choice D is not correct, as the Model Rules prohibit a partnership if any of the business of the partnership will include the practice of law, even if the lawyer is the only person providing legal advice.

An attorney entered into a written retainer agreement with a defendant in a criminal case. The defendant agreed in writing to transfer title to her automobile to the attorney if the attorney successfully prevented her from going to prison. Later, the charges against the defendant were dismissed. Is the attorney subject to discipline for entering into this retainer agreement? A) Yes, because the attorney agreed to a fee contingent on the outcome of a criminal case. B) Yes, because a lawyer may not acquire a proprietary interest in a client's property. C) No, because the charges against the defendant were dismissed. D) No, because the retainer agreement was in writing.

A) Yes, because the attorney agreed to a fee contingent on the outcome of a criminal case. Answer choice A is correct. MRPC 1.5(d)(2) provides that a lawyer may not represent a defendant in a criminal case on a contingent fee basis. Here, the client's payment of a fee (title to an automobile) was contingent on a successful outcome in the criminal case. Answer choice B is incorrect. MRPC 1.8(i) provides that a lawyer may not obtain a proprietary interest in the cause of action or the subject of the litigation the lawyer is conducting for a client. But the attorney has not done so in this case because the automobile is not involved in the litigation. MRPC 1.8(i) does not prohibit a lawyer in all cases from obtaining a proprietary interest in a client's property. The fee agreement, however, was impermissible under MRPC 1.5(d)(2) because the lawyer's fee was contingent on the successful outcome in the criminal case. Answer choice C is incorrect because, even though the lawyer achieved success, he is subject to discipline for entering into an impermissible fee agreement. Answer choice D is incorrect because, although it is true that a contingent fee agreement must be in writing, this agreement was improper for a different reason.

An attorney represents a corporation which is a defendant in a product liability case. An engineer of the corporation nearing retirement was likely to be a key witness in the case, as she had been in charge of all of the corporation's product safety testing during the relevant period. The engineer had been very critical of the corporation's safety testing procedures during that period and had repeatedly complained that the product at issue had not been adequately tested. The engineer's views were reduced to writing and were well known to many employees of the corporation. Because of the early stage of the case, however, plaintiff's counsel was not yet aware of the engineer's existence or her views. Aware of the engineer's views, the corporation's attorney approached the corporation's officials and recommended that the corporation offer the engineer a special package of severance benefits if she would retire immediately and move to the Bahamas. The attorney believed that if the engineer accepted this offer, she would be beyond the subpoena power of the court in which the suit against the corporation was pending. The corporation adopted the attorney's recommendation and made the offer. The engineer accepted it. The attorney did not disclose the engineer's identity to plaintiff's counsel. Is the attorney subject to discipline? A) Yes, because the attorney caused the engineer to leave the jurisdiction of the court for the purpose of making her unavailable as a witness. B) Yes, because opposing counsel had not yet had a reasonable opportunity to learn of the engineer's views. C) No, because the engineer's views were reduced to writing and are well known to many other employees of the corporation. D) No, unless there was a pending request for the engineer's testimony at the time the retirement offer was made to the engineer.

A) Yes, because the attorney caused the engineer to leave the jurisdiction of the court for the purpose of making her unavailable as a witness. Answer choice A is correct. Under MRPC 3.4(a), a lawyer must not unlawfully obstruct another party's access to evidence or counsel or assist another person to do any such act. Here, by causing the engineer to leave the court's jurisdiction for the purpose of making her unavailable as a witness, the attorney was obstructing the plaintiff's access to evidence in the case and would be subject to discipline. Answer choice B is not correct, because whether or not opposing counsel had a reasonable opportunity to learn of the engineer's views is not controlling with regard to whether the attorney should be subject to discipline. What matters is that the attorney caused the engineer to leave the court's jurisdiction for the purpose of making her unavailable as a witness. Answer choice C is not correct, because although the engineer's views may have been available to the opposing party, the opposing party would not have an opportunity to fully examine the engineer on the issues. By causing the engineer to leave the court's jurisdiction for the purpose of making her unavailable as a witness, the attorney is subject to discipline. Answer choice D is not correct, as the fact that there may not have been a pending request for the engineer's testimony at the time the offer was made is not controlling. The attorney was aware that the engineer's testimony could be significant evidence in the case.

An attorney regularly appears before a trial court judge who is running for reelection in six months. Over the past year, the attorney has noticed that the judge has become increasingly ill-tempered on the bench. Not only is the judge abrupt and critical of lawyers appearing before him, he is also rude and abusive to litigants. On more than one occasion, the judge has thrown his gavel across the courtroom in a fit of temper. The judge's conduct on the bench is often the subject of discussion whenever a group of lawyers meets. Some lawyers are automatically filing requests for judicial substitution whenever a case in which they are to appear is assigned to the judge. The attorney discussed the matter with her law partners, who rarely make court appearances. The attorney's law partners suggested that she, too, file a request for judicial substitution whenever one of her cases is assigned to the judge. In addition, the attorney and her law partners discussed the possibility of reporting the judge to the appropriate disciplinary authority but are concerned that this would alienate the other judges to whom their cases are assigned. The attorney has reluctantly started filing for substitution of the judge in every one of her cases to which the judge is assigned, but she has taken no further action. Is the attorney subject to discipline? A) Yes, because the attorney failed to inform the appropriate authorities about the judge's conduct. B) Yes, because, by filing automatic requests for substitution of the judge, the attorney undermined public confidence in the administration of justice. C) No, because the attorney has a duty to represent her clients zealously. D) No, because the judge is running for re-election and may not be re-elected.

A) Yes, because the attorney failed to inform the appropriate authorities about the judge's conduct. Answer choice A is correct. Under MRPC 8.3, a lawyer who has actual knowledge that a judge has committed a violation of the rules of judicial conduct that raises a substantial question as to that person's fitness for judicial office must inform the appropriate authority. In this fact pattern, the judge has, on more than one occasion, thrown a gavel across the courtroom and has been critical of lawyers and rude to litigants. Such behavior crosses the line into conduct that raises a substantial question as to his ability to judge. Answer choice B is incorrect; under MRPC 8.3, a lawyer who has actual knowledge that a judge has committed a violation of the rules of judicial conduct that raises a substantial question as to that person's fitness for judicial office must inform the appropriate authority. Filing requests for substitution of the judge does not excuse the duty to report, but it really has nothing to do with public confidence in the judiciary. Failing to report the judge's behavior does affect public confidence. Answer choice C is incorrect because zealous representation of the attorney's clients does not relieve her of the duty to report the judge. Answer choice D is incorrect because whether or not the judge will be reelected does not affect the attorney's duty to report this matter to the appropriate authority.

An attorney was engaged to represent an individual in litigation over injuries suffered by the individual when attacked by a homeowner's dog while visiting the home. While this matter was pending, the attorney agreed to represent a criminal defendant accused of robbing a convenience store. The attorney learned that the individual from the dog attack case was the key eyewitness for the prosecution and that there were aspects of the individual's identification of the defendant that could be called into question on cross-examination. The attorney reasonably believed that there was not a significant risk that the representation of the defendant would be materially limited by the attorney's responsibilities to the individual since the two matters were unrelated and the attorney had not acquired information in representing the individual that was relevant to the individual's identification of the defendant. To avoid surprising the individual when the individual testified at the defendant's trial, the attorney informed the individual of his representation of the defendant, but did not seek nor obtain the individual's consent to the attorney's representation of the defendant. The attorney also informed the criminal defendant that he represented the individual in another matter and secured the defendant's informed consent in writing to the attorney's representation of the individual. Is the attorney subject to discipline? A) Yes, because the attorney failed to secure the individual's informed consent to the attorney's representation of the defendant. B) Yes, because the individual's testimony at the defendant's trial would create a conflict of interest that cannot be waived. C) No, because the criminal prosecution of the defendant was unrelated to the individual's litigation to recover damages for injuries suffered from a dog attack. D) No, because the individual would not be called by the attorney as a witness for the defendant.

A) Yes, because the attorney failed to secure the individual's informed consent to the attorney's representation of the defendant. Answer choice A is correct. A lawyer must not represent a client if doing so would be directly adverse to the interests of another current client unless, among other things, each affected client gives informed consent, confirmed in writing. Even though the matter regarding which the attorney represents each client is unrelated, a directly adverse conflict can arise when a lawyer is required to cross-examine a client who appears as a witness in an unrelated lawsuit involving another client, such as when the testimony will be damaging to the client who is represented in the lawsuit. Here, the individual's testimony for the prosecution will be damaging to the defendant, and the cross-examination of the individual to call into question the individual's credibility will be directly adverse to the individual. Consequently, the attorney should have sought the individual's informed consent, confirmed in writing. Answer choice B is incorrect because, although the individual's testimony for the prosecution would create a conflict of interest, this conflict may be waived if both clients give informed consent in writing. Note that a conflict of interest might not arise if the individual was being called by the attorney to testify on behalf of the defendant. However in this case, there is a conflict because the individual is being called by the prosecution to give evidence against the defendant and the attorney will be required to cross-examine the individual. Answer choice C is incorrect because, even though the matter regarding which the attorney represents each client is unrelated, a directly adverse conflict can arise when a lawyer is required to cross-examine a client who appears as a witness in an unrelated lawsuit involving another client, as when the testimony will be damaging to the client who is represented in the lawsuit. Here, the individual's testimony in the prosecution of the defendant will be damaging to the defendant and the cross-examination of the individual to call into question the individual's credibility will be directly adverse to the individual. Answer choice D is incorrect. If the individual were called by the attorney as a witness for the defendant, it is unlikely that a conflict would exist since the attorney's purpose would be to offer the individual's testimony into evidence, not to challenge its validity.

An attorney represented a client as a plaintiff in a personal injury matter under a standard contingent fee contract. The client agreed to settle the case for $1,000,000, from which funds the attorney would receive $250,000. The client informed the attorney that she planned to take $25,000 of the settlement funds and spend the money purchasing lottery tickets. The attorney told the client that he disagreed with this plan and encouraged the client to take some classes on investing money. The client agreed to take the classes, but still insisted on playing the lottery. The attorney received the check for $1,000,000 three days before the client was to attend the investing classes. The attorney held the check for one week, giving the client at least a few days of classes. The attorney then informed the client of the receipt of the funds, disbursed the funds according to the agreement, and also furnished the client with an accounting. The attorney told the client that he had delayed notice to allow time for the client to come to her senses. The client laughed and said, "I guess your plan worked, because these classes have convinced me to invest my money in the stock market instead of playing the lottery." Is the attorney subject to discipline? A) Yes, because the attorney had a duty to promptly notify the client of the receipt of the $1,000,000. B) Yes, because the attorney gave unsolicited advice about non-legal matters. C) No, because the client did not object to the withholding of the notice and funds. D) No, because the attorney acted in the client's best interest.

A) Yes, because the attorney had a duty to promptly notify the client of the receipt of the $1,000,000. Answer choice A is correct. MRPC 1.15(d) requires the lawyer to "promptly notify the client" upon receiving funds in which the client has an interest. Here, the attorney deliberately delayed notifying the client for one week, until the client had been to classes in investing. Answer choice B is incorrect because the lawyer was permitted to give this unsolicited advice. MRPC 2.1 permits a lawyer who is advising a client to "refer not only to law but to other considerations such as moral, economic, social, and political factors that may be relevant to the client's situation." Answer choice C is incorrect because the lawyer's failure to promptly notify the client of the lawyer's receipt of client funds and to promptly deliver the funds to the client, as required by MRPC 1.15(d), is not excused by the client's later failure to object. Answer choice D is incorrect because MRPC 1.15(d) provides that the lawyer must promptly notify the client when the lawyer receives funds belonging to the client. There is no exception when the lawyer thinks that it would be best for the client to delay notification.

A seller was engaged in negotiations to sell his interest in a large tract of land to a buyer who was unrepresented in the transaction. Before the seller went out of town for a few days, he told the buyer to call his attorney if the buyer had any questions about the property. The buyer called the seller's attorney, asked certain questions about the size of the tract, and expressed hesitations concerning the high asking price for the tract. The attorney responded that, based on his experience handling real estate transactions in the neighborhood, the buyer would be getting a lot of property for the price. At the time the attorney spoke to the buyer, the attorney knew that there was a defect in the title and that the buyer's attempt to purchase the seller's interest in the tract would not result in the buyer's acquisition of any interest in the property. Relying on the attorney's assurance, the buyer agreed to make the purchase. Shortly after the sale closed, the buyer discovered that his acquisition was worthless. Is the attorney subject to civil liability to the buyer? A) Yes, because the attorney knowingly made false representations of fact to the buyer. B) Yes, because the attorney implied that his opinion regarding the value of the property was a disinterested opinion. C) No, because the attorney's statement that the buyer would be getting a lot of property for the money was a statement of opinion regarding the value of the property. D) No, because the buyer was not a client of the attorney.

A) Yes, because the attorney knowingly made false representations of fact to the buyer. Answer choice A is correct. Normally, lawyers do not owe a duty to the opposing side of a transaction. But a lawyer may not make intentional or negligent misrepresentations to a third person. In this case, the attorney made a misrepresentation when he falsely told the buyer that he would be getting a lot of property for the price, because he knew that the buyer's attempt to purchase the seller's interest would not result in the buyer's acquisition of any interest in the property. Answer choice B is incorrect because whether the attorney was or was not disinterested does not change the fact that the attorney made a misrepresentation to the buyer. Answer choice C is incorrect; the attorney's statement could not be construed as an "opinion." The attorney told the buyer that he would be getting a lot of property for the price, knowing that the buyer's investment would be compromised because of the title issue. Answer choice D is incorrect. Normally, attorneys do not owe a duty to the opposing side of a transaction. In this case, however, the attorney made a statement to the buyer that was patently false, exposing himself to civil liability.

An attorney represented a plaintiff in an action against several defendants. The retainer agreement provided that the plaintiff would pay all costs and expenses of litigation and would, on demand, reimburse the attorney for any costs or expenses advanced by the attorney. After serving process on two defendants, the attorney had difficulty locating and serving the remaining defendants. The plaintiff approved the hiring of an investigator to locate and serve the defendants, and the attorney advanced the costs for the investigator. When the attorney asked the plaintiff for reimbursement, the plaintiff refused to pay. The attorney then told the plaintiff that she would do no more work on the case until she was reimbursed for the amount advanced. Thereafter, one of the defendants filed a counterclaim that required a responsive pleading within thirty days. Because the attorney had not been paid, the attorney permitted the time to respond to the counterclaim to expire without filing a responsive pleading, and a default was entered on the counterclaim. Later, the plaintiff reimbursed the attorney for the costs the attorney had advanced, and the attorney was successful in having the default on the counterclaim set aside. The case was tried, and the plaintiff prevailed on his complaint, and the counterclaimant recovered nothing. Is the attorney subject to discipline for not initially filing a responsive pleading to the counterclaim? A) Yes, because the attorney neglected the plaintiff's case. B) Yes, unless the attorney had asked leave of court to withdraw. C) No, because the plaintiff breached the agreement to reimburse the attorney. D) No, because the plaintiff did not sustain any prejudice as a result of the attorney's action.

A) Yes, because the attorney neglected the plaintiff's case. Answer choice A is correct. A lawyer may properly withdraw if the client refuses to abide by the terms of an agreement relating to the representation, such as an agreement concerning fees or costs; however, permission of the court must be granted. MRPC 1.16(c). Here, the attorney did not ask the court for leave to withdraw. Moreover, even if withdrawal was permitted, MRPC 1.16(d) requires a withdrawing lawyer to take all reasonable steps to mitigate the consequences to the client. Here, the attorney allowed the filing period to lapse and entry of default to occur and did not take any steps to warn the plaintiff. Answer choice B is not correct, as even if the attorney had asked leave of court to withdraw, the attorney still needed the court's permission to actually stop working on the case and needed to take reasonable steps to mitigate the consequences of the withdrawal to the client. Answer choice C is not correct, as even though the plaintiff breached the agreement, MRPC 1.16 requires court permission to withdraw and the attorney to have taken steps to mitigate the consequences of the withdrawal to the client. Answer choice D is not correct, as default was entered on the counterclaim, which does constitute a negative consequence, even though it was subsequently set aside.

As required by a purchase agreement, a consumer and a retailer submitted a dispute to an arbitration panel. The consumer and retailer each named an attorney to be a partisan member of the panel and together those two panel members chose the third member of the panel. After listening to both sides, the panel ruled 2-1 in favor of the retailer, with the panel member named by the consumer voting in favor of the consumer. Without seeking the consent of the retailer, the consumer employed the attorney that the consumer had appointed to the panel to challenge the arbitration of the dispute. Is it proper for the attorney to accept this employment? A) Yes, because the attorney served as a partisan member of the three-member arbitration panel. B) Yes, because the attorney voted against the decision reached by the panel. C) No, because the retailer did not consent to the attorney's subsequent representation of the consumer with regard to the arbitrated matter. D) No, because the attorney personally and substantially participated as a member of the arbitration panel.

A) Yes, because the attorney served as a partisan member of the three-member arbitration panel. Answer choice A is correct. Generally, an attorney who personally and substantially participates as an arbitrator may not represent anyone in connection with the matter arbitrated, unless all parties to the proceeding give informed, written consent. However, this restriction does not apply to an arbitrator selected as a partisan of a party in a multi-member arbitration panel. Answer choice B is incorrect because there is no requirement that an attorney who serves as an arbitrator vote against the panel's decision in order to be able to represent a party in connection with the matter arbitrated. Answer choice C is incorrect. As noted with respect to answer choice A, while generally each party to an arbitration must give informed, written consent for an attorney who personally and substantially participated as an arbitrator to represent anyone in connection with the matter arbitrated, this restriction does not apply to an arbitrator selected as a partisan of a party in a multi-member arbitration panel. Answer choice D is incorrect because, although the attorney personally and substantially participated as a member of the arbitration panel by listening to both parties to the arbitration present their cases and voting on the matter being arbitrated, the attorney could nevertheless represent the consumer in an action challenging the arbitration of the dispute because the attorney was selected as a partisan of the party in the multi-member arbitration panel.

An attorney represented the plaintiff in a personal injury matter. The attorney had heard that the defendant in the matter was anxious to settle the case and reasonably believed that the defendant's lawyer had not informed the defendant about the attorney's recent offer of settlement. The attorney instructed her non-lawyer investigator to tell the defendant about the settlement offer so that the attorney could be sure that the defendant's lawyer did not force the case to trial merely to increase the defendant's lawyer's fee. Is the attorney subject to discipline? A) Yes, because the defendant was represented by counsel. B) Yes, because the attorney was assisting the investigator in the unauthorized practice of law. C) No, because the investigator is not a lawyer. D) No, because the attorney reasonably believed that the defendant's lawyer was not keeping the defendant informed.

A) Yes, because the defendant was represented by counsel. Answer choice A is correct. MRPC 4.2 prohibits a lawyer from communicating about a matter with a person known to be represented in the matter unless the person's lawyer consents to the communication or the communication is authorized by law. Under MRPC 8.4(a), a lawyer may not violate the rules directly or through the acts of another. Here, the attorney used a non-lawyer agent to violate the no-contact rule. Answer choice B is incorrect because the investigator merely relayed a communication from the attorney to the defendant and did not thereby engage in the unauthorized practice of law. Answer choice C is incorrect because of MRPC 4.2, as discussed above. Such communication is impermissible regardless of whether it is done by the attorney or by an agent of the attorney. Answer choice D is incorrect also because of MRPC 4.2; the defendant's lawyer did not consent to the communication, and the fact that the defendant's lawyer was not relaying settlement offers did not otherwise justify the communication.

The owner of undeveloped real property entered into a contract to sell the property to an investor. The investor's obligation to purchase the property was conditioned on the title to the property being marketable. A lawyer employed by the investor investigated the validity of the seller's title to the property and reported to the investor that there was a cloud on the seller's title, which made the property unmarketable. As a consequence, on the date set for closing, the investor did not tender the purchase price. On that date, due to independent economic events, the fair market value of the property was less than the purchase price. Subsequently, it was uncovered that the lawyer misread a key document and the seller had clear title to the property. Is the lawyer subject to discipline? A) Yes, because the lawyer failed to competently investigate the validity of the seller's title to the property. B) Yes, because the seller suffered an economic loss when the buyer refused to complete the sale. C) No, because due to the drop in the fair market value of the property, the investor did not suffer an economic loss in foregoing the purchase of the property. D) No, because the lawyer's error was not intentional.

A) Yes, because the lawyer failed to competently investigate the validity of the seller's title to the property. Answer choice A is correct. A lawyer is required to provide competent representation to a client. Here, the lawyer failed to provide such representation by negligently conducting the title search. Answer choice B is incorrect because, although the seller presumably suffered an economic loss when the seller eventually sold the property since, at the time of closing, the property's fair market value had dropped below the contract price, it is not necessary that a third party suffer harm for a lawyer to be subject to discipline for violation of a rule. Answer choice C is incorrect because, although the investor did not suffer an economic loss by not completing the sale since the fair market value of the property had dropped below its contract price, it is not necessary that a client suffer harm for a lawyer to be subject to discipline for a violation of a rule. Answer choice D is incorrect because, while the punishment imposed for violation of a rule of professional conduct may be less when the lawyer's violation was not willful, a lawyer is nevertheless subject to discipline for violation of a rule even though the violation was inadvertent rather than intentional.

A lawyer has just joined a professional corporation engaged solely in the practice of law. The lawyer is a salaried associate and is not a member or shareholder of the professional corporation. A partner's spouse, who is not a lawyer, is vice president of the corporation and office manager. All of the other officers are lawyers in the firm. All of the corporate shares are held by lawyers in the corporation, except for ten shares held by the executor under the will of a lawyer-member who died one month previously and whose will is now being probated. The lawyer knows that the partner's spouse is an officer and not a lawyer. Is the lawyer subject to discipline? A) Yes, because the partner's spouse is an officer of the corporation. B) Yes, if a non-lawyer holds the stock as the executor of the will of the deceased member. C) No, because the lawyer is a salaried employee and not a member or shareholder of the corporation. D) No, if the partner's spouse does not participate in any decision regarding a client or a client's case.

A) Yes, because the partner's spouse is an officer of the corporation. Answer choice A is correct. Under MRPC 5.4(d), a lawyer is not permitted to practice with or in the form of a professional corporation or association authorized to practice law for a profit if a non-lawyer is an officer of the corporation. Because the lawyer knows that the partner's spouse is a non-lawyer, the lawyer would be subject to discipline. Answer choice B is not correct. A non-lawyer who is an executor may hold the stock of a lawyer in a professional corporation for a reasonable time during the administration of the lawyer's estate. Answer choice C is not correct, as the fact that the lawyer is only a salaried employee does not absolve the lawyer of responsibility under MRPC 5.4(d), which applies to all lawyers associated with the corporation, regardless of whether or not they are members or shareholders. Answer choice D is not correct, as the fact that the partner's spouse does not participate in decisions regarding clients or their cases is irrelevant. MRPC 5.4(d) controls.

A corporation hired a law firm to handle all of its corporate work. The firm had not previously represented a corporation on an ongoing basis, but decided that it wanted to attract additional corporate clients. Accordingly, the partners handling the corporation's work began a practice of giving to all lawyers in the firm, on a monthly basis, detailed descriptions of the work they were doing for the corporation for the purpose of illustrating what the firm could do for corporate clients. One of the partners mentioned this practice to the corporation's management, and the corporation complained that its confidences had been violated. Was it proper for the partners to give detailed descriptions of the work being done for the corporation to other lawyers in the firm? A) Yes, because, absent client instructions to the contrary, lawyers may discuss client information with other lawyers in the firm. B) Yes, because lawyers may discuss client information with other lawyers in the firm, regardless of client instructions to the contrary, so long as the disclosure does not disadvantage the client. C) No, because sharing the information increased the risk that it might be improperly disclosed to third parties outside the firm. D) No, because lawyers may not disclose client information, even to other firm lawyers, unless the disclosure is in furtherance of the representation.

A) Yes, because, absent client instructions to the contrary, lawyers may discuss client information with other lawyers in the firm. Answer choice A is correct. Although the nature of the work done for clients of the firm is confidential under MRPC 1.6 and generally may not be disclosed to third parties without client consent, "[l]awyers in a firm may, in the course of the firm's practice, disclose to each other information relating to a client of the firm, unless the client has instructed that particular information be confined to specified lawyers." MRPC 1.6, cmt. [5]. Answer choice B is incorrect; if the client instructs that particular confidential information be disclosed only to particular lawyers in the firm, the firm must abide by that instruction. Answer choice C is incorrect. To facilitate a law firm's practice, its lawyers may share client confidential information with each other, even though doing so may increase the risk that the information will be disclosed to third parties outside the firm, because clients generally understand that this is done and implicitly consent to it. Law firms take other measures to minimize the risk that the information will be disseminated to third parties. Therefore, except when the client instructs otherwise or there are special reasons to limit disclosures within the firm, client information may be shared, as in this case, among the firm's lawyers for legitimate business purposes. Answer choice D is incorrect. Disclosures of client information may be made within a law firm to carry out the representation, but that is not the only circumstance in which disclosures may be made within the firm. In general, they may also be made for the purpose of facilitating the law firm's practice, such as for billing purposes or for the education of young lawyers or, as in this case, to familiarize the firm's lawyers with the nature of each other's practice.

An attorney represented a buyer in a real estate transaction. Due to the attorney's negligence in drafting the purchase agreement, the buyer was required to pay for a survey that should have been paid for by the seller, the other party to the transaction. The attorney fully disclosed this negligence to the buyer, who suggested that he would be satisfied if the attorney simply reimbursed the buyer for the entire cost of the survey. Although the buyer might have recovered additional damages if a malpractice action were filed, the attorney reasonably believed that the proposed settlement was fair to the buyer. Accordingly, in order to forestall a malpractice action, the attorney readily agreed to make the reimbursement. The attorney drafted a settlement agreement, and it was executed by both the attorney and the buyer. Was the attorney's conduct proper? A) Yes, if the attorney advised the buyer in writing that the buyer should seek independent representation before deciding to enter into the settlement agreement. B) Yes, because the attorney reasonably believed that the proposed settlement was fair to the buyer. C) No, because the attorney settled a case involving liability for malpractice while the matter was still ongoing. D) No, unless the buyer was separately represented in negotiating and finalizing the settlement agreement.

A) Yes, if the attorney advised the buyer in writing that the buyer should seek independent representation before deciding to enter into the settlement agreement. Answer choice A is correct. Under MRPC 1.8(h)(2), a lawyer may not settle a claim or potential claim for malpractice liability with an unrepresented client or former client unless that person is advised in writing of the desirability of seeking independent legal counsel with regard to the settlement and is given a reasonable opportunity to seek such advice. Because the answer choice indicates that the attorney properly advised the buyer of the need for independent representation, the attorney's conduct was proper. Answer choice B is not correct, as fairness alone does not control the attorney's responsibility to the buyer. The Model Rules require advising the client in writing of the desirability of seeking independent legal counsel. Answer choice C is not correct. The facts indicate that the transaction has already occurred because the client had to pay for a survey that should have been paid for by the seller. Additionally, regardless of whether there any performace remains to be satisfied under the transaction, a cause of action for malpractice generally accrues when the client suffers actual damages. Here, the client suffered damages when he had to pay for this survey. Therefore, a cause of action for malpractice has accrued, and the attorney may settle this claim without the client being independently represented by counsel, so long as the lawyer advises the client, in writing, of the desirability of seeking independent legal counsel. Answer choice D is not correct, because once an underlying matter is concluded a lawyer may settle a client's claim for malpractice without the client being independently represented by counsel, so long as the lawyer advises the client, in writing, of the desirability of seeking independent legal counsel. If the lawyer is attempting to prospectively limit malpractice liability (i.e., regarding claims that have not yet accrued), then independent counsel is required.

An attorney, who had represented a testator for many years, prepared the testator's will and acted as one of the two subscribing witnesses to its execution. The testator's sister and brother were his sole heirs. The will left the testator's entire estate to his sister and nothing to his brother. Upon the testator's death two years later, the executor named in the will asked the attorney to act as his lawyer in the probate of the will and the administration of the estate. At that time, the executor informed the attorney that the testator's brother would concede that the will was properly executed but intended to contest the will on the ground that he had been excluded because of fraud previously practiced on the testator by the testator's sister. The other subscribing witness to the will predeceased the testator, and the attorney will be called as a witness solely for the purpose of establishing the due execution of the will. Is it proper for the attorney to accept the representation? A) Yes, if there is no contested issue of fact with respect to the formal execution of the will. B) Yes, because the executor has no beneficial interest under the will. C) No, unless the attorney's services are necessary to avoid substantial hardship to the executor. D) No, because the attorney will be called as a witness in the case.

A) Yes, if there is no contested issue of fact with respect to the formal execution of the will. Answer choice A is correct. Under MRPC 3.7(a), a lawyer is permitted to act as an advocate at a trial in which the lawyer is likely to be a necessary witness if the testimony relates to an uncontested issue. If there is no contested issue of fact with respect to the formal execution of the will, then it is proper for the attorney to accept the representation. Answer choice B is not correct, as it is irrelevant that the executor has no beneficial interest under the will. A lawyer is only permitted to act as an advocate at a trial in which the lawyer is likely to be a necessary witness if (i) the testimony relates to an uncontested issue, (ii) the testimony relates to the nature and value of legal services rendered in the case, or (iii) disqualification of the lawyer would work substantial hardship on the client. MRPC 3.7(a). Answer choice C is not correct. Even though avoiding substantial hardship to the client is a ground for the attorney to properly represent the executor under MRPC 3.7(a), it is not the only ground, which the phrasing of this answer choice suggests. Answer choice D is not correct, as MRPC 3.7(a) does allow a lawyer to be a witness and act as an advocate when the testimony relates to an uncontested issue.

An attorney has recently started her own law firm with four other lawyers as associates. The law firm has moved into offices in a new building that is owned by a bank. The attorney has borrowed heavily from the bank to finance her new law firm. In addition, the bank provides the law firm with accounting services through its computer. At the bank's suggestion, an employee of the bank, who is not a lawyer, serves as a part-time office manager for the law firm without compensation from the firm. The duties of the office manager are to advise the firm generally on fees and time charges and on program matters for the computer services, and to consult with the attorney on accounting and billing practices to ensure solvency. Is the arrangement with the bank proper? A) Yes, unless secrets or confidences of clients may be disclosed to the bank. B) Yes, because the office manager is paid by the bank. C) No, because a non-lawyer will be advising the law firm on fees and time charges. D) No, because the bank will be involved in the practice of law.

A) Yes, unless secrets or confidences of clients may be disclosed to the bank. Answer choice A is correct. The bank employee is serving as an office manager. There is no violation of the Model Rules when a lawyer has offices in a creditor's building, obtains accounting services from a creditor, or hires a creditor's employee, paid by the creditor. If the bank employee discloses secrets or confidences of the attorney's clients, however, that would violate the attorney's ethical duty of confidentiality under MRPC 1.6 and the attorney's responsibility under MRPC 5.1 to ensure that subordinates, including non-lawyers, follow the requirements of the Model Rules. Answer choice B is incorrect, because the fact that the office manager is paid by the bank is not relevant, as she is not practicing law at the firm, but performing clerical and office tasks. Answer choice C is incorrect, as advising the firm on fees and time charges is not improper under the Model Rules. It does not constitute the unauthorized practice of law by the office manager. Answer choice D is incorrect, as the office manager is not practicing law and therefore her activities, if attributed to the bank, her employer, would not be improper.

Attorney advertises on the local television station. In the advertisements, a professional actor says: "Do you need a lawyer? Call Attorney - her address is 512 Main Street and her telephone number is area code 555-555-5555. Her fees might be lower than you think." Is the advertisement proper? A) Yes. B) No, unless Attorney's fees are lower than those generally charged in the area where she practices. C) No, because she used a professional actor for the television advertisement. D) No, if she makes a charge for the initial consultation.

A) Yes. Answer choice A is correct. Under the Model Rules, a lawyer is generally permitted to advertise her services so long as the communications regarding such services are not false or misleading in violation of MRPC 7.1, or in violation of the rules against solicitation of clients under MRPC 7.3. MRPC 7.2(a). This rule permits public dissemination of information concerning a lawyer's name or firm name, address, and telephone number; the kinds of services the lawyer will undertake; the basis on which the lawyer's fees are determined, including prices for specific services and payment and credit arrangements; a lawyer's foreign language ability; names of references; and other information that might invite the attention of those seeking legal assistance. MRPC 7.2, cmt 2. Under MRPC 7.2(c), any advertisement must include the name and office address of at least one lawyer or law firm responsible for its content. MRPC 7.2(c). All of these requirements are met in Attorney's advertisement. Answer choice B is not correct, as it does not matter if Attorney's fees are lower than those generally charged in the area. Attorney made no such claim. Answer choice C is not correct, as there is no prohibition against using professional actors in an advertisement. Answer choice D is not correct; it is irrelevant whether or not Attorney charges for an initial consultation, as it was not raised in the advertisement.

Which of the following is a permissible advertisement? A) "I am the best attorney ever." B) "I am a criminal defense attorney." C) "I am an estate planning certified specialist," without a certification. D) "I have never lost a case."

B) "I am a criminal defense attorney."

Which of the following statements regarding candor to the tribunal is FALSE? A) A lawyer may not offer evidence that he knows is false. B) A lawyer need not disclose unfavorable controlling authority that the other side should have presented. C) A lawyer generally may refuse to offer evidence that he reasonably believes is false. D) Candor to the tribunal includes courts as well as other tribunals such as administrative agencies.

B) A lawyer need not disclose unfavorable controlling authority that the other side should have presented.

Which of the following statements regarding perjury by a criminal defendant is FALSE? A) The defendant's lawyer has a duty to attempt to dissuade the defendant from offering perjured testimony. B) A lawyer's duty to correct errors in the litigation continues after the litigation ends. C) When the defendant's lawyer cannot withdraw from the case, she may allow the defendant to testify by narrative. D) The defendant's lawyer has a duty to correct errors during the litigation.

B) A lawyer's duty to correct errors in the litigation continues after the litigation ends.

Which of the following is least likely a permissible real-time direct contact to solicit business? A) Approaching a family member about representation B) Approaching a new client for the purpose of pecuniary gain C) Approaching a pre-existing client for the purpose of pecuniary gain D) Volunteering for charitable work

B) Approaching a new client for the purpose of pecuniary gain

An attorney, who represented a plaintiff, received a check from the defendant payable to the attorney's order in the sum of $10,000 in settlement of the plaintiff's claim against the defendant. The plaintiff had previously paid the attorney a fee, so no part of the $10,000 was owed to the attorney. Which of the following would not be proper? A) Endorse the check and send it to the plaintiff. B) Deposit the check in the attorney's personal bank account and send the attorney's personal check for $10,000 to the plaintiff. C) Deposit the check in a Clients' Trust Account, advise the plaintiff, and forward a check drawn on that account to the plaintiff. D) All of the above would be proper.

B) Deposit the check in the attorney's personal bank account and send the attorney's personal check for $10,000 to the plaintiff. Answer choice B is correct. A lawyer who receives funds or other property in which a client has an interest must promptly notify the client and is required to pay over the funds, making answer choice A incorrect, because it is a proper course of action. MRPC 1.15 requires that a lawyer hold funds that are in a lawyer's possession in connection with a representation separate from the lawyer's own funds. Therefore, depositing the check in the attorney's personal bank account would be improper, making answer choice B correct. Answer choice C is incorrect because it, too, is a proper course of action, as it avoids commingling the client's funds with the lawyer's funds and meets the requirements of MRPC 1.15.

Which of the following is a sufficient character and fitness ground for denying an applicant a license to practice law? A) Conviction of a minor crime, such as disorderly conduct B) Failure to disclose a minor crime C) Carrying substantial debt D) Not being a resident of the state in which admission is sought

B) Failure to disclose a minor crime

Which of the following statements regarding fee sharing agreements is FALSE? A) The overall fee must be fair to the client. B) Fees can be shared with both lawyers and non-lawyers. C) The referring lawyer must provide services or joint representation in connection with the case. D) The referral fee must be disclosed to the client.

B) Fees can be shared with both lawyers and non-lawyers.

An attorney represented a client who filed a civil suit against a defendant for battery after the defendant attacked her during a failed robbery attempt. During their interview prior to the trial, the client told the attorney that she only suffered injuries to her arm during the attack. However, when the client took the stand during the trial, she stated that she had also suffered extreme head injuries from the attack which resulted in constant headaches, vertigo, and permanent vision blurring. After her testimony, the attorney took the client aside, advised her of the attorney's duty of candor to the tribunal, and confidentially advised her to correct her false testimony. The client refused to retract her testimony. The attorney believed that withdrawal from representation of the client would not undo the effect of her false testimony. Because of this, the attorney disclosed to the judge what he believed was reasonably necessary about the client's false testimony to remedy the situation. Is the attorney subject to discipline? A) No, because an attorney may reveal confidential information the extent the lawyer reasonably believes is necessary to prevent the client from committing a fraud. B) No, because an attorney must take reasonable remedial measures when his client has offered material evidence and the attorney comes to know of its falsity. C) Yes, because the attorney was required to withdraw from representation of the client when the client refused to retract her testimony. D) Yes, because the client did not give her informed consent for the attorney to disclose confidential information to the judge.

B) No, because an attorney must take reasonable remedial measures when his client has offered material evidence and the attorney comes to know of its falsity. Answer choice B is correct. If the lawyer's client has offered material evidence and the lawyer comes to know of its falsity, the lawyer must take reasonable remedial measures. Reasonable remedial measures can include confidentially advising the client of the lawyer's duty of candor and urging the client's cooperation with respect to the withdrawal or correction of the false statements or evidence. If that fails, the lawyer is required to take further remedial action. If withdrawal from the representation is not permitted or will not undo the effect of the false evidence, the lawyer must make such disclosure to the tribunal as is reasonably necessary to remedy the situation, even if doing so requires the lawyer to reveal information that otherwise would be protected as confidential by the Model Rules. Here, the client offered material evidence that was false regarding her head injuries, so the attorney was required to take reasonable remedial measures. The attorney advised the client of the attorney's duty of candor and advised her to correct her false testimony, but the client refused to retract her testimony. The attorney believed that withdrawal from representation of the client would not undo the effect of her false testimony, so he disclosed to the judge what he believed was reasonably necessary about the client's false testimony to remedy the situation. Therefore, the attorney is not subject to discipline because he took reasonable remedial measures under the circumstances. Answer choice A is incorrect. A lawyer is permitted, but not required, to reveal confidential information concerning the representation of a client to the extent the lawyer reasonably believes it necessary to prevent the client from committing a crime or fraud that is reasonably certain to result in substantial injury to the financial interests or property of another and in furtherance of which the client has used or is using the lawyer's services. However, when an attorney learns that a client has offered material false testimony, the lawyer is required, not merely permitted, to take remedial measures, up to and including disclosure to the tribunal. Answer choice C is incorrect. An attorney is not required to withdraw from representation of a client when the client has refused to retract her false testimony. Answer choice D is incorrect. As discussed above, the lawyer must make such disclosure to the tribunal as is reasonably necessary to remedy the situation when the client has offered false evidence, even if doing so requires the lawyer to reveal information that otherwise would be protected as confidential by the Model Rules. In other words, the lawyer can reveal confidential information to the judge without the client's consent without being subject to discipline.

An attorney regularly represented oil companies that supported drilling in the state in which he lived and worked. The attorney diligently represented his clients and did not let his personal beliefs interfere with his ability to represent them, even though he was morally opposed to drilling. He also belonged to a nonprofit legal services agency organized to represent the residents of cities and towns affected by hazardous environmental agents in any legal action based on the effects of those agents. The group also raised community awareness on the dangers of drilling on various segments of the community, including children. The attorney attended all of the group's meetings and events but abstained from voting on all matters related to the organization's mission and objectives. Is the attorney subject to discipline for his involvement with the agency? A) No, because he did not serve as a director or an officer of the agency. B) No, because he did not knowingly participate in decisions incompatible with his obligation to his clients or the agency's clients. C) Yes, because the agency's efforts might have an adverse effect on the attorney's clients. D) Yes, because the attorney knowingly participated in an agency whose efforts might have an adverse effect on the attorney's clients.

B) No, because he did not knowingly participate in decisions incompatible with his obligation to his clients or the agency's clients. Answer choice B is correct. An attorney may serve as a director, an officer, or a member of a legal services organization, apart from the law firm in which the attorney practices, even if the organization serves persons having interests that are adverse to a client of the attorney. The attorney, however, is not permitted to knowingly participate in a decision or action of the organization if such participation would be incompatible with the attorney's obligations to a client under the Model Rules of Professional Conduct concerning conflicts of interest, or when the decision or action could have a material adverse effect on the representation of a client of the organization whose interests are adverse to a client of the attorney. Answer choice A is incorrect because the attorney may serve as an officer or director of such an agency, provided he does not knowingly participate in decisions incompatible with his obligations to a client and there is no other conflict of interest. Answer choice C is incorrect because even if the organization and the client have adverse interests, the attorney may still serve as a director, an officer, or a member if he does not participate in decision-making that has such an effect. Answer choice D is incorrect because the attorney may participate in such an organization even if he knows of the adverse interests.

An attorney worked at a law firm that encouraged its attorneys to provide at least 100 hours of pro bono service per year. The attorney typically surpassed this goal. Last year, however, the attorney was involved in a long, complex trial, and he did not provide any pro bono service. The attorney did not make any contributions to nonprofit legal organizations during the year. Would the attorney's failure to provide pro bono services or to contribute to nonprofit legal organizations subject him to discipline? A) No, because the Model Rules allow pro bono hours exceeding the 50-hour minimum to be carried forward to subsequent years. B) No, because the Model Rules indicate only that an attorney should aspire to perform 50 hours of pro bono services annually. C) Yes, because the Model Rules require an attorney to provide a minimum of 50 hours of pro bono legal services each year. D) Yes, because the Model Rules require an attorney to either render a minimum of 50 hours of pro bono legal services each year or make a monetary contribution to nonprofit legal organizations in lieu of such service.

B) No, because the Model Rules indicate only that an attorney should aspire to perform 50 hours of pro bono services annually. Answer choice B is correct. The Model Rules of Professional Conduct set forth an aspirational goal for an attorney to accept representation of clients who are unable to pay, suggesting that attorneys offer at least 50 hours of pro bono legal services per year. This goal is not enforced through the disciplinary process. Accordingly, the attorney would not be subject to discipline. Answer choice A is incorrect because an attorney may not be disciplined for failure to perform pro bono service, and thus it is irrelevant whether hours can be carried forward. Answer choice C is incorrect because the guidelines regarding pro bono service in the Model Rules of Professional Conduct are voluntary, and thus the attorney would not be subject to discipline for failure to meet the suggested minimum. Answer choice D is incorrect because the Model Rules of Professional Conduct do not subject an attorney to discipline for the failure to either provide pro bono service or to make a monetary contribution to a nonprofit legal organization.

An attorney conducted a significant amount of legal research to support a senior partner's defense of a business in an insurance fraud lawsuit brought by the state attorney general. Because the attorney only conducted research related to jurisdictional issues, he did not learn any material confidential information about the substance of the insurance fraud lawsuit, including information about the business, legal strategies, or any facts that were in dispute. Two years later, the attorney obtained a new job with the office of the state attorney general, where he was assigned to work on the ongoing insurance fraud lawsuit. The attorney informed his former senior partner of his new assignment, and after consulting with the business, the senior partner informed the attorney that the business had given its written informed consent for the attorney to work with the state attorney general on the lawsuit. The attorney subsequently began working on the insurance fraud lawsuit, but he did so without first obtaining informed consent, confirmed in writing, from the office of the attorney general. Was it proper for the attorney to work on the insurance fraud lawsuit at the office of the attorney general? A) No, because an attorney may not switch sides during a lawsuit even with the opposing party's written informed consent. B) No, because the attorney did not obtain informed consent, confirmed in writing, from the office of the attorney general before beginning work on the lawsuit. C) Yes, because the attorney did not learn any material confidential information about the lawsuit while working at his former law firm. D) Yes, because the business gave its written informed consent for the attorney to work on the lawsuit.

B) No, because the attorney did not obtain informed consent, confirmed in writing, from the office of the attorney general before beginning work on the lawsuit. Answer choice B is correct. A lawyer currently serving as a government employee is generally subject to the general ethics rules relating to current and former clients. In addition, the government lawyer is not permitted to participate in a matter in which the lawyer participated personally and substantially while in private practice or non-governmental employment, unless the appropriate government agency gives its informed, written consent. Here, although the attorney did not learn material confidential information about the previous matter, he was still required to obtain informed consent, confirmed in writing, from the attorney general's office. Answer choice A is incorrect because a lawyer is permitted to switch sides during a lawsuit provided that the appropriate governmental agency gives its informed consent, confirmed in writing. Answer choice C is incorrect because the requirement to obtain the informed, written consent of the appropriate governmental agency does not turn on whether a lawyer obtained material confidential information about a matter, but instead on whether the lawyer participated personally and substantially in the matter while in private practice or non-governmental employment. Because the attorney conducted significant legal research to support the senior partner's defense of the business in this action, he is required to obtain unless the appropriate government agency's informed, written consent in order to work on the insurance fraud lawsuit at the office of the attorney general. Answer choice D is incorrect because the consent of the business was inadequate; the attorney was also required to obtain the informed consent, confirmed in writing, of the attorney general's office.

An attorney and a client entered into a standard engagement agreement that contained a clause requiring the arbitration of any potential malpractice claims. Arbitration clauses were enforceable under the laws of the attorney's home state if they were clearly disclosed. The arbitration clause was printed in a bold and prominent font, and clearly explained that arbitration would involve resolving the dispute outside of the court system without the opportunity for a jury trial. In addition, the attorney explained to the client the scope and effect of the arbitration clause. However, the attorney did not require her client to be represented by independent counsel in agreeing to arbitration. Is the attorney subject to discipline? A) No, because agreements between lawyers and clients are treated the same as any other agreements made at arm's length. B) No, because the attorney fully explained and clearly disclosed the arbitration clause to her client. C) Yes, because the attorney did not require her client to be represented by independent legal counsel in agreeing to arbitration. D) Yes, because the client's agreement to arbitrate a malpractice claim is tantamount to a prospective waiver of the attorney's malpractice liability.

B) No, because the attorney fully explained and clearly disclosed the arbitration clause to her client. Answer choice B is correct. A lawyer is prohibited from making an agreement prospectively limiting malpractice liability to a client, unless the client is represented by an independent lawyer in making the agreement. However, an agreement to submit a future malpractice dispute to arbitration is permitted under the Model Rules when the client is informed as to the scope and the effect of the arbitration clause. Here, the client was full informed of the scope and effect of the agreement by the attorney. Thus, the attorney is not subject to discipline. For this reason, answer choice D is incorrect. Answer choice A is incorrect because it misstates the law. In fact, the model rules restrict a lawyer's ability to enter into a business transaction with a client or knowingly acquire any interest adverse to a client in several ways. They also limit their ability to contract with clients to acquire media rights, solicit gifts, or limit malpractice recovery. Answer choice C is incorrect. Because an arbitration clause is not equivalent to an agreement prospectively limiting the lawyer's liability to a client for malpractice, there is no requirement that the client be represented by independent counsel.

An attorney was engaged by the board of directors of a homeowner's association for a residential community of over 100 homes to represent the association in a dispute with the developer of the community. The dispute concerned a contract with the developer to provide maintenance services to the community. The contract had been executed when the developer controlled the association. The attorney kept the board members, who were all homeowners in the community, reasonably informed as to the negotiations with the developer. However, he did not have any contact with the other homeowners who were members of the association, even though not all the homeowners had the same interests in the matter. When the negotiations were completed, the attorney, at the request of the board, presented the developer's settlement offer to a meeting of the homeowner's association members for their approval as mandated by the association's articles of incorporation. Is the attorney subject to discipline? A) No, because the attorney had no a duty to inform the association's board or its members about the negotiations until a settlement offer was reached. B) No, because the attorney kept the members of the association's board of directors informed about the negotiations. C) Yes, because the attorney should have kept all of the association's members informed about the negotiations. D) Yes, because the attorney breached his duty of confidentiality to the board of directors by presenting the developer's settlement offer to all of the association's members.

B) No, because the attorney kept the members of the association's board of directors informed about the negotiations. Answer choice B is correct. An organizational lawyer owes the duties of loyalty and confidentiality to the organization. Among a lawyer's duties in representing a client is to keep the client reasonably informed about the status of the matter for which the lawyer is providing representation. A lawyer employed or retained to represent an organization represents the organization acting through its duly authorized constituents. Here, the attorney kept the association's governing body, its board of directors, informed as to the status of the negotiations with the developer. Consequently, the lawyer is not subject to discipline. Answer choice A is incorrect because, as noted with regard to answer choice B, the attorney did have a duty to keep his client reasonably informed about the negotiations with the developer, which the attorney satisfied in his dealings with the association's board of directors. Answer choice C is incorrect because, as noted with regard to answer choice B, the attorney's client was the association itself, and not the individual members of the association. A lawyer, in dealing with an organization's constituents, must explain the identity of the client when the lawyer knows or reasonably should know that the organization's interests are adverse to those of the constituents with whom the lawyer is dealing. However, since the attorney did not have contact with the individual homeowners other than the members of the board in their capacity as board members, the attorney did not have a duty to dispel any confusion as to his role that could have arisen from such contact. Answer choice D is incorrect. The association and not its individual members was the attorney's client. Consequently, though the association acted through the board of directors, the attorney owed a duty of confidentiality to the association itself. ?

An attorney defended his client in a criminal trial over a valuable ruby necklace that the client allegedly stole from a jewelry store where she worked. The client maintained her innocence during the trial, claiming that she had been framed by her boss. The jury acquitted the defendant on all charges. The day after the deadline for post-trial motions had passed, the client admitted to the attorney that she had lied on the stand when she testified that she had not stolen the necklace from the store. In fact, she had stolen the necklace, and she was now feeling remorseful about lying on the stand. The attorney listened to the client's confession, and then did nothing more about the matter. Is the attorney subject to discipline? A) No, because special protections are provided to criminal defendants when the lawyer knows that the defendant has offered false testimony. B) No, because the attorney only learned of the client's perjury after the conclusion of the proceedings. C) Yes, because the attorney should have attempted to persuade the client to correct the false testimony. D) Yes, because the attorney should have disclosed to the tribunal what was necessary to remedy the client's fraud.

B) No, because the attorney only learned of the client's perjury after the conclusion of the proceedings. Answer choice B is correct. The prohibition on knowingly offering false evidence and the duty to take reasonable remedial measure on learning of the falsity of evidence after it has been admitted applies through the conclusion of the proceeding. A lawyer is under no duty to reveal false evidence or perjury that is discovered after the conclusion of the proceedings. In a criminal case, a verdict of acquittal concludes the proceedings because the Double Jeopardy Clause of the Constitution prevents the state from retrying the defendant. Here, the attorney learned that his client had offered false testimony after the defendant had been acquitted and the deadline for post-trial motions had passed. Therefore, he was under no duty to reveal his client's perjury. Answer choice A is incorrect. The Model Rules do not permit a lawyer to refuse to offer the testimony of a criminal defendant client when the lawyer reasonably believes but does not know that the testimony will be false. Unless the lawyer knows the testimony will be false, the lawyer must honor the client's decision to testify. However, the Model Rules to not grant criminal defendants any special protections after a lawyer learns that the client has offered false testimony. Answer choices C and D are incorrect because the prohibition on knowingly offering false evidence and the duty to take reasonable remedial measure on learning of the falsity of evidence after it has been admitted end at the conclusion of the proceeding. Therefore, the attorney is neither required to persuade the client to correct the false testimony nor to disclose to the tribunal what is necessary to remedy the situation.

After surviving a deadly car accident, an attorney who was a sole practitioner decided to set up a contingency plan to protect her clients in the event of her untimely death. As part of her plan, she appointed another lawyer to review her client files upon proper notice of her death, to notify her clients of her death, and to determine if there was a need for immediate action in any of her cases in order to avoid neglect. The attorney started including this information in her standard engagement agreement with new clients. However, existing clients were not informed about the plan, nor did the attorney obtain their consent to the plan. Is the attorney subject to discipline? A) No, because state disciplinary authorities have no jurisdiction over matters of law practice management. B) No, because the attorney prepared an appropriate plan to protect her clients in the event of her untimely death. C) Yes, because the attorney did not inform existing clients of the plan. D) Yes, because the attorney did not obtain the consent of existing clients to the plan.

B) No, because the attorney prepared an appropriate plan to protect her clients in the event of her untimely death. Answer choice B is correct. In order to prevent neglect of client matters in the event of death or disability of a sole practitioner, the practitioner should prepare a plan, in conformity with applicable rules, that designates another competent lawyer to review client files, notify each client of the lawyer's death or disability, and determine whether there is a need for immediate protective action. The sole practitioner's current clients are not required to receive notice of such a plan, and their consent is not required to satisfy the Model Rules. Answer choice A is incorrect because a lawyer's practice management is subject to various rules of professional conduct. Answer choice C is incorrect. A sole practitioner is not required to notify the practitioner's clients of any contingency plans the practitioner may have established to ensure protective action is taken in the event of the practitioner's death. Answer choice D is incorrect because consent of the sole practitioner's current clients is not required.

An attorney recently obtained his license to practice law after several unsuccessful attempts to pass the bar. He set up a solo practice. Through a mutual friend, a prospective client contacted the attorney about representing her in a disability claim before the Social Security Administration. Although the attorney had no previous personal experience with any type of Social Security claim and had received no training during law school with regard to such claims, he reasonably believed that he could achieve the necessary competence without unreasonable delay or association with another attorney. He erroneously assumed that the prospective client was aware of his lack of experience through their mutual friend. Therefore, the attorney did not mention his lack of experience to the prospective client, and the prospective client did not ask about it. The attorney proposed to represent the prospective client under a contingency fee arrangement, but the prospective client declined to engage the attorney, even though the fee arrangement was reasonable. Is the attorney subject to discipline? A) No, because no attorney-client relationship formed between the attorney and the prospective client. B) No, because the attorney reasonably believed that he could achieve the necessary competence without unreasonable delay. C) Yes, because the attorney failed to warn the prospective client about his lack of experience. D) Yes, because the attorney lacked experience of any sort with regard to Social Security claims.

B) No, because the attorney reasonably believed that he could achieve the necessary competence without unreasonable delay. Answer choice B is correct. While a lawyer is obligated to provide competent representation to a client, a lawyer who reasonably believes that he can become competent through preparation may offer to represent a prospective client. Answer choice A is incorrect because, while a lawyer is obligated to provide competent representation to a client whom the lawyer has agreed to represent, a lawyer must also decline to represent a client when he cannot become competent on his own or through association with competent counsel without unreasonable delay. Therefore, the fact that the prospective client ultimately declined to hire the attorney would not exempt the attorney from liability if he had offered to represent the client in violation of this rule. Answer choice C is incorrect because an attorney who reasonably believes that he can become competent through preparation may offer to represent a prospective client without disclosing to the client his lack of experience. Answer choice D is incorrect because mere lack of experience with regard to a legal matter is not by itself sufficient grounds on which to base an ethical violation when the lawyer reasonably believes he can achieve the requisite level of competence by reasonable preparation.

An attorney represented a wife in a wrongful death action of her husband, whose death was caused by a drunk driving accident while the wife was in the car with him. During the representation, the woman was oftentimes overcome with grief over the death of her husband, and this grief impeded her ability to focus and organize her thoughts when the attorney tried to prepare her for the hearing. Due to her diminished capacity, the attorney reasonably believed that the wife could not make adequately considered decisions and that she was at risk of substantial harm with respect to her objective of obtaining a favorable outcome in the wrongful death action. The attorney counseled the wife to get help from a psychologist, as the attorney believed that the wife's testimony about seeing the defendant run a red light was crucial to obtaining a favorable outcome. The wife refused to seek help. Left with no other choice and because he desperately needed to prepare for trial, the attorney reached out to the wife's sister for assistance in convincing the wife to seek counseling. However, the attorney took this action without the wife's consent. Is the attorney subject to discipline? A) No, because attorneys have an implied authorization to disclose confidential information to third persons in the course of the representation. B) No, because the attorney reasonably believed that the wife's diminished capacity put her at risk of an unfavorable outcome in the wrongful death action. C) Yes, because the attorney contacted the wife's sister without the wife's consent. D) Yes, because the attorney tried to influence the wife's testimony in the trial.

B) No, because the attorney reasonably believed that the wife's diminished capacity put her at risk of an unfavorable outcome in the wrongful death action. Answer choice B is correct. Model Rule 1.14(b) permits a lawyer to take reasonably necessary protective action on behalf of a client when the lawyer reasonably believes that the client has diminished capacity, is at risk of substantial physical, financial, or other harm unless action is taken, and cannot adequately act in the client's own interest. Under this rule, protective action includes consulting individuals or entities that have the ability to take action to protect the client, such as family members. Here, the attorney reasonably believed that contacting the wife's sister was necessary to mitigate the substantial risk that the wife would not get a favorable outcome in the wrongful death action. Answer choice A is incorrect. Generally, Model Rule 1.6(a) prohibits a lawyer from disclosing information relating to a representation without the client's informed consent. However, a lawyer may, under certain circumstances, have implied authority to disclose client information to third parties in order to carry out a representation, such as to admit a fact that cannot properly be disputed. That exception does not apply here, however, where the attorney's disclosure to protect the wife from substantial physical, financial, or other harm is expressly permitted by Model Rule 1.14(b). Answer choice C is incorrect. Under normal circumstances, Model Rule 1.6(a) prohibits a lawyer from disclosing information relating to a representation without the client's informed consent. However, when a lawyer is permitted to take protective action on behalf of a client with diminished capacity, the lawyer is impliedly authorized to reveal information about the client to the extent reasonably necessary to protect the client's interests. Answer choice D is incorrect. The attorney did not attempt to influence the wife's testimony at the trial. Rather, the attorney made the disclosure to help convince the wife to seek treatment for her grief prior to the trial. In any event, a lawyer is permitted to assist clients and other witnesses in preparing to testify truthfully.

A defendant charged with arson told her attorney that she had burnt down her ex-husband's home. During the trial, the prosecution offered the ex-husband's new girlfriend as their principal witness. The attorney forcefully cross-examined the girlfriend, and even though the attorney knew that she had testified truthfully, he asked her questions to make it look like she was not a credible witness. The girlfriend was overwhelmed by the attorney's cross-examination, and as a result, she did not appear to be a convincing witness. The defendant was subsequently acquitted. Is the attorney subject to discipline? A) No, because the attorney was allowed to offer evidence that the attorney knew to be false. B) No, because the attorney was allowed to test the credibility of the prosecution's witness. C) Yes, because the attorney's cross-examination implied that the girlfriend was not a credible witness even though he knew she had testified truthfully. D) Yes, because the attorney zealously defended the defendant even though the attorney knew that she had committed arson.

B) No, because the attorney was allowed to test the credibility of the prosecution's witness. Answer choice B is correct. Although Model Rule 3.1 prohibits lawyers from bringing, defending, and asserting frivolous claims and assertions, the rule specifically states that a lawyer for the defendant in a criminal proceeding may nevertheless defend the proceeding as to require that every element of the case be established. In addition, a lawyer may cross-examine a witness with respect to testimony that the lawyer knows to be truthful, including harsh implied criticism of the witness's testimony, character or capacity for truth-telling. Here, the attorney did not violate this rule by challenging the credibility of the girlfriend, even though he knew that she had testified truthfully. Therefore, the attorney is not subject to discipline. For this reason, answer choice C is incorrect. Answer choice A is incorrect because Model Rule 3.3(a)(3) expressly prohibits a lawyer from knowingly offering evidence that the lawyer knows is false. Answer choice D is incorrect because Model Rule 3.1 permits a lawyer representing a criminal defendant to defend the proceeding as to require the state to prove every element of the state's case.

An attorney filed a motion for summary judgment on behalf of his client despite knowledge of directly adverse case law from the highest court in the state. Since the issuance of that decision, however, courts in other states had reached the opposite conclusion. In the motion, the attorney cited the decisions from other states but did not mention the decision from the state's highest court. The opposing attorney filed a brief in opposition to the motion for summary judgment that did not cite the decision from the state's highest court. The court did not hold oral argument but granted summary judgment based on the briefs. Were the attorney's actions in not disclosing the decision of the state's highest court proper? A) No, because the attorney was required to disclose the decision in his initial brief supporting the motion for summary judgment. B) No, because the attorney was required to disclose the decision after the opposing counsel failed to disclose it. C) Yes, because an attorney does not have a duty to disclose cases that are adverse to his client's position. D) Yes, because disclosure of the case would violate the attorney's duty to zealously advocate his client's position.

B) No, because the attorney was required to disclose the decision after the opposing counsel failed to disclose it. -Read the call of the question first. This is an issue of EVIDENCE ON THE OTHER SIDE/CANDOR TO THE COURT. The RULE is: you have to DISCLOSE EVIDENCE (even if it doesn't work in your favor) if the other party DOESN'T DISCLOSE IT because of CANDOR TO THE COURT.. It is not a violation when the attorney initially does not disclose it. It's bad practice but not a violation. -A) is wrong because it does not align with the rule. -B) is correct because this aligns with the rule. -C) is wrong because does not align with the rule. -D) is wrong because you still have an obligation to disclose it.

The seller of a gas filling station withheld from his attorney the fact that the gasoline storage tanks buried under the station were leaking. The attorney prepared the necessary documents for the sale, including a statement by the seller that, to his knowledge, the tanks were sound and free from leaks. Shortly before the sale, the attorney learned about the leaks from a station employee, but she was unsuccessful in her attempts to convince the seller to inform the buyer. The attorney subsequently withdrew from representing the seller, gave the buyer notice of her withdrawal, and disaffirmed the documents relating to the sale that she had prepared. The attorney gave similar notice to the institutional lender who was providing financing to the buyer. Is the attorney subject to discipline for notifying the institutional lender? A) No, because the attorney was required to reveal her client's crime or fraud to those affected by it. B) No, because the client was planning to commit a crime or fraud that was reasonably certain to result in substantial financial injury. C) Yes, because the institutional lender was not the buyer trying to purchase the filling station. D) Yes, because the leaks were not reasonably certain to result in death or substantial bodily harm to the lender.

B) No, because the client was planning to commit a crime or fraud that was reasonably certain to result in substantial financial injury. Answer choice B is correct. A lawyer may reveal confidential information concerning the representation of a client to the extent the lawyer reasonably believes it necessary to prevent the client from committing a crime or fraud that is reasonably certain to result in substantial injury to the financial interests or property of another and in furtherance of which the client has used or is using the lawyer's services. Here, the attorney's services had been used to prepare a fraudulent statement contained in the sale documents that was reasonably certain to result in substantial injury to the financial interests of the lender who was financing the sale of the gas filling station. Consequently, the attorney was permitted to inform the lender of the attorney's withdrawal from representation of the seller and her disaffirmance of the sale documents she had prepared. Answer choice A is incorrect because, although the attorney was permitted to notify the lender, the attorney was not required to do so under the Model Rules. Answer choice C is incorrect because a lawyer is permitted to reveal confidential information when the lawyer's services are being used to commit fraud that is reasonably certain to result in substantial injury to the financial interests or property of another. Here, although the lender was not the purchaser of the filling station, the lender was reasonably certain to suffer substantial financial injury as a result of the borrower's purchase of the filling station with leaking gasoline storage tanks. Answer choice D is incorrect. While a lawyer may reveal confidential information concerning the representation of a client to the extent the lawyer reasonably believes disclosure is necessary to prevent reasonably certain death or substantial bodily harm, that is not the only exception to the lawyer's duty of confidentiality. As noted with regard to answer choice B, there is another exception when the lawyer's services are or have been used by a client to commit a crime or fraud.

An attorney and a prospective client met to discuss whether the attorney would represent the client in a contractual dispute. During the conversation, the potential plaintiff spoke to the attorney about her litigation objectives and how much she would be able to pay the attorney. As they were wrapping up the meeting, the client noticed a picture of the attorney's teenaged son on the wall. The client confided in the attorney that she had a son the same age, but she had given him up for adoption because she was an unwed teenager when he was born. She told the attorney that no one except her family knew about the adoption, and she asked the attorney to keep it confidential. Is the information about the client's pregnancy protected by the attorney-client privilege? A) No, because the woman had not retained the attorney when the conversation took place. B) No, because the communication was not relayed for legal advice. C) Yes, because the woman had a reasonable expectation that she had established an attorney-client relationship with the attorney. D) Yes, because the woman had a reasonable expectation that she had established an attorney-client relationship with the attorney, and she stated that the communication was confidential.

B) No, because the communication was not relayed for legal advice. Answer choice B is correct. A confidential communication between a client and her attorney is privileged. The evidentiary attorney-client privilege covers the client's communication to an attorney whom the client reasonably believes represents the client, and the circumstances indicate a desire by the client for confidentiality. Although the attorney does not need to give advice or agree to the representation for the privilege to exist, the communication must be for the purpose of seeking legal advice or representation. Although the statement about the client's child was given in confidence and would be protected by the attorney's ethical duty of confidentiality, it is not subject to the attorney-client privilege because it was not relayed for the purpose of obtaining legal advice. Answer choice A is incorrect because, although the woman was a potential client with regard to the contract matter, the privilege would not apply to the statement regarding the adoption because the statement was not made for the purpose of obtaining legal advice. Answer choice C is incorrect because the communication was not made for the purpose of obtaining legal advice, so it is not privileged even if the woman had a reasonable expectation of representation. Answer choice D is incorrect because, although the communication would have been privileged if it had been relayed for the purpose of obtaining legal advice, it was not relayed for that purpose.

A newly installed judge was given an expensive watch upon leaving the law firm at which the judge had practiced law for a number of years as a recognition of his years with the firm. The firm typically makes a similar gift when an attorney with the same number of years with the firm retires from the practice of law. The state has adopted an ethics rule that automatically disqualifies the judge from hearing any case in which the judge was a member of a law firm representing a party within the preceding two years. Must the judge publicly report the gift of the watch? A) No, because the firm is not treating the judge differently from a lawyer who retires from the firm. B) No, because the judge is subject to the two-year automatic disqualification rule. C) Yes, because the watch is of material value. D) Yes, because the gift was not made by a close family member.

B) No, because the judge is subject to the two-year automatic disqualification rule. Answer choice B is correct. While a judge may not accept a gift that would appear to a reasonable person to undermine the judge's independence, integrity, or impartiality, a judge may accept a gift without publicly reporting it from friends, relatives, or other persons, including lawyers, whose appearance or interest in a proceeding pending or impending before the judge would in any event require disqualification of the judge. CJC 3.13. Since there is an ethics rule that requires the judge to disqualify himself from hearing any case in which the judge was a member of a law firm representing a party within the preceding two years, the judge may accept the gift of the watch without publicly reporting it. Answer choice A is incorrect because, even if the gift of the watch may be a gift that would not appear to a reasonable person to undermine the judge's independence, integrity, or impartiality because a similar gift is given to retiring lawyer, and thus may be accepted by the judge, the gift would have to be publicly reported since the source was a party or other person, including a lawyer, who is likely to come before the judge, were it not for the two-year disqualification rule. Answer choice C is incorrect because, while the value of the watch alone could arguably make the gift seem to a reasonable person to undermine the judge's independence, integrity, or impartiality, here the ethics rule makes its unreasonable that the gift would call into question the judge's independence, integrity, or impartiality. Answer choice D is incorrect because, although a gift from a close family member also may be accepted and not publicly reported because the judge would be required to disqualify himself because of his familial relationship with the family member, this exception also applies to gifts from the judge's former law firm due to the ethics rule.

At the first status conference in a civil case, the trial judge disclosed on the record that the defendant's lawyer was the personal attorney for the judge's brother, but said that he believed he could be fair and impartial in the matter and was willing to serve on the case. The judge asked the parties and their lawyers whether they wanted to waive his disqualification or have the case assigned to another judge. The parties conferred with their lawyers at counsel table. The judge remained in the courtroom and overheard the discussion, but did not participate in the discussion. Although the plaintiff expressed reservations about the judge's continued service on the case, the plaintiff's lawyer advised her that, because it was a jury trial where the jury would decide the merits, and because the judge was an experienced trial court judge, the plaintiff should waive the disqualification. Both parties waived the judge's disqualification on the record. Did the judge handle the matter properly? A) No, because the judge's disqualification was required since the defendant's lawyer was the personal attorney of the judge's brother. B) No, because the judge remained in the courtroom while the parties conferred with their lawyers at counsel table about the disqualification. C) Yes, because the judge believed that he could be fair and impartial in the matter. D) Yes, because both parties waived the judge's disqualification on the record.

B) No, because the judge remained in the courtroom while the parties conferred with their lawyers at counsel table about the disqualification. Answer choice B is correct. Unless a judge is subject to disqualification for a personal bias or prejudice or personal knowledge of facts that are in dispute in the proceeding, the parties may waive the judge's disqualification. However, in addition to disclosing on the record the basis of the judge's disqualification, the judge must ask the parties and their lawyers to consider, outside the presence of the judge and court personnel, whether to waive the judge's disqualification. Here, the judge improperly remained in the courtroom and overheard the discussion. Answer choice A is incorrect. The fact that the defendant's lawyer was the personal attorney of the judge's brother does not specifically violate the relationship ground for disqualification set out in the CJC, which requires a judge to disqualify herself when the judge knows that she, her spouse or domestic partner, a person within the third degree of relationship to either of them, or the spouse or domestic partner of such a person is a party, lawyer, material witness, or person who has more than a de minimis interest that could be affected by the proceedings. However, while the judge presumably nevertheless concluded that the relationship between the defendant's lawyer and the judge's brother was such that the judge's impartiality could reasonably be questioned and therefore merited disqualification, since this disqualification did not arise from a personal bias or prejudice of the judge or a personal knowledge of the disputed facts, the judge could properly request the parties to consider waiving the disqualification. Unfortunately, the judge failed to follow the proper procedures for obtaining this waiver. Answer choice C is incorrect because, while the judge's subjective belief that he could be fair and impartial is a requirement for the judge to be able to request that the parties consider waiving a disqualification, the judge did not employ the proper process for obtaining the waiver. Answer choice D is incorrect because, although both parties did waiver the judge's disqualification on the record, the judge failed to follow proper procedure by remaining in the courtroom while the parties considered the waiver request.

The following advertisement appeared in a daily newspaper in a state in which both parties are members of the bar: A. ALPHA, M.D., J.D. and B. BETA, J.D. Attorneys at Law 1000 "A" Street, City, State, 00000 Telephone (555) 555-5555. Are Alpha and Beta subject to discipline? A) No, because both law and medicine are licensed professions. B) No, if they possess the degree(s) stated. C) Yes, because the reference to the M.D. degree is self-laudatory. D) Yes, unless they limit their practice to areas in which a medical degree is relevant.

B) No, if they possess the degree(s) stated. Answer choice B is correct. Lawyers are permitted to advertise through public media under the Model Rules, so long as their advertisements are not false, misleading, or in violation of the rules against client solicitation. MRPC 7.2. This rule permits public dissemination of information concerning a lawyer's name or firm name, address, and telephone number; the kinds of services the lawyer will undertake; the basis on which the lawyer's fees are determined, including prices for specific services and payment and credit arrangements; a lawyer's foreign language ability; names of references; and other information that might invite the attention of those seeking legal assistance. MRPC 7.2, cmt 2. Any advertisement must also include the name and office address of at least one lawyer or law firm responsible for its content. MRPC 7.2(c). Here, all of these rules are satisfied. So long as Alpha and Beta have the degrees they list in the advertisement, there would be nothing false or misleading, and they would not be subject to discipline under the Model Rules. Answer choice A is not correct, as it does not matter that both law and medicine are licensed professions. The fact that Alpha and Beta both possess the degrees that they advertised, so that the advertisement is not false or misleading, is what controls. Answer choice C is not correct, as it is not improper under the Model Rules for a lawyer to truthfully list his educational degrees. Answer choice D is not correct, as there is no requirement under the Model Rules that they limit their practice to areas in which a medical degree would be relevant.

A member of a law firm that operates as a limited liability company was elected mayor of a city. The law firm, which included the last name of the member-mayor in the name of the firm, has sent out a notice to its clients and taken out an ad in the local newspaper. The notice announces that, as a consequence of the member's election, she is taking a leave of absence from the firm during her four-year term, but that with her consent, her last name will continue to appear in the firm's name. By law, the mayor of the city cannot serve two successive terms. Is the firm's use of the mayor's last name in the firm's name is proper? A) No, because the firm is operating as a limited liability company B) No, because the mayor will not be practicing with the firm during her four-year term. C) Yes, because the firm has informed its clients and the public at large of the firm's plan to continue to use the mayor's name in the firm's name. D) Yes, because the mayor is taking a leave of absence, rather than resigning from the firm.

B) No, because the mayor will not be practicing with the firm during her four-year term. Answer choice B is correct. The name of a lawyer holding a public office is not permitted to be used in the name of a law firm, or in communications on its behalf, during any substantial period in which the lawyer is not actively and regularly practicing with the firm. While the mayor is limited to a four-year term, four years is a substantial period of time during which the mayor will not be actively and regularly practicing with the firm. Consequently, the firm cannot continue to use the mayor's last name in the firm's name, regardless of the mayor's consent. Answer choice A is incorrect because the restriction on the use of the name of a lawyer who is holding public office in the law firm's name applies to any law firm, regardless of the corporate form in which the firm is operating. Answer choice C is incorrect because, in order for a law firm to be able to continue to use the name of lawyer who has been elected to public office in the firm's name, the lawyer must actively and regularly practice with the firm. Notification of the firm's clients and the public at large does not permit the firm to continue to use the mayor's last name in the firm's name when she will not be actively and regularly practicing with the firm. Answer choice D is incorrect because, in order for a law firm to be able to continue to use the name of lawyer who has been elected to public office in the firm's name, the lawyer must actively and regularly practice with the firm. Here, even though the mayor has not resigned, she will not be actively and regularly practicing with the firm during her four-year term in office.

A client hired an attorney to sue a defendant in a business litigation, agreeing to pay a specified reasonable hourly fee which was due when the suit was over. Shortly after the suit was filed, the defendant unexpectedly impleaded a third party, and the increased complexity of the proceedings threatened to require much more of the attorney's time than the parties had originally expected. The attorney explained to the client that the attorney was willing to continue on an hourly fee, but offered to modify the fee agreement based on the recent change in circumstances. After the attorney explained in reasonable detail the risks and benefits of both hourly and contingency fee arrangements, the attorney and the client agreed to shift to a contingent-fee arrangement that was properly memorialized in writing and was reasonable in light of the expected complexity of the proceedings. Soon thereafter, the defendant made, and the client accepted, a large settlement offer. Under the new contingent fee arrangement, the attorney was entitled to recover a large contingent fee, although the client would have paid a much lower amount under the original hourly fee arrangement. Is the attorney subject to discipline with respect to the modified fee agreement? A) No, because changes in the basic nature of a fee arrangement are always permitted with the client's informed consent. B) No, because the modification was prompted by an unanticipated change in circumstances and was reasonable at the time of the modification. C) Yes, because an unexpected shift in workload is not a justified excuse for changing a fee arrangement. D) Yes, because the contingency fee exceeds what a lawyer would have received by charging a reasonable hourly rate.

B) No, because the modification was prompted by an unanticipated change in circumstances and was reasonable at the time of the modification. Answer choice B is correct. A fee agreement may be modified during a lawyer's representation of a client, but the modified agreement must be reasonable under the circumstances at the time of the modification. A change in the basic nature of a fee arrangement or a significant increase in the lawyer's compensation is generally unreasonable unless there has been an unanticipated change in circumstances. Here, the defendant unexpectedly impleaded a third party, and the proceedings threatened to require much more of the attorney's time than the parties had originally expected. As such, the attorney proposed that they switch from an hourly fee arrangement to a contingent fee arrangement. Therefore, the modification was not unreasonable. For this reason, answer choice C is incorrect. Answer choice A is incorrect because a change in the basic nature of a fee arrangement or a significant increase in the lawyer's compensation is not automatically proper when a client consents to the change. There must also be an unanticipated change in circumstances, as was the case here. Answer choice D is incorrect. Contingent fees, like any other fees, must be reasonable under the circumstances, but a contingency fee that exceeds what a lawyer would have received by charging a reasonable hourly rate is not per se unreasonable.

A niece and her uncle, who are both licensed attorneys, decided to rent an office space together in order to save money. They hired a receptionist who worked for both of them, and they shared the expenses involved with the daily operations of the legal office. Each attorney hired their own legal assistant and utilized separate telephone numbers and e-mail accounts. Neither had access to the other's client information, and no legal fees earned by the attorneys were shared. The niece worked on real estate transactions, and the uncle dealt with personal injury matters. Recently, one of the uncle's clients was injured during a bungee jumping accident and hired the uncle to bring a claim against the bungee jumping company. The bungee jumping company was owned by one of the niece's clients. The uncle filed a negligence action against the bungee jumping company, and the company's in-house defense counsel moved to disqualify the uncle due to his affiliations with the niece. Is the court likely to grant the motion to disqualify? A) No, because attorneys who do not share legal fees are not deemed to be associates for purposes of the conflict of interest rules. B) No, because the niece and the uncle have taken reasonably adequate measures to protect the confidential information of their respective clients. C) Yes, because attorneys who share office space and other practice expenses are deemed to be associated for purposes of the conflict of interest rules. D) Yes, because the uncle's representation of a party directly adverse to the niece's client presents an appearance of impropriety.

B) No, because the niece and the uncle have taken reasonably adequate measures to protect the confidential information of their respective clients. Answer choice B is correct. The test for determining if lawyers who share office facilities are considered associated for purposes of imputation of conflicts of interest is whether there are reasonably adequate measures to protect confidential client information so that it will not be available to other lawyers in the shared office. Here, the uncle and the niece have taken reasonably adequate measures to protect the confidential information of their respective clients, so there is no imputation of conflicts of interest between the uncle and the niece. Therefore, the court is unlikely to grant the motion to disqualify the uncle. Answer choice A is incorrect because the sharing of legal fees does not determine whether conflicts of interest are imputed between lawyers. Answer choice C is incorrect because the fact that the uncle and the niece share rent and other expenses is not determinative of whether the attorneys are considered associated for purposes of imputation of conflicts of interest. Answer choice D is incorrect because this is not a standard for assessing conflicts of interest. Additionally, although a lawyer related to another lawyer (e.g., parent, child, sibling, or spouse) may not represent a client in a matter in which the related lawyer is representing another party unless each client gives informed consent, this rule does not apply because the uncle and the niece are not representing adverse parties in the same matter.

During a phone call, a terminated corporate officer, who was a sophisticated business person, secured the representation of an attorney in a contractual dispute with the corporation over the officer's severance pay. The officer, assessing his chances of success as low, offered a reasonable contingency fee to the attorney, and the attorney accepted it. They also quickly arrived at an agreement as to how the fee was to be calculated and the expenses allocated. On the same day, the attorney reduced the terms of the fee arrangement to writing and sent the document to the officer for his signature. Despite several requests by the attorney, the officer never signed nor returned this document. Several months later, the attorney negotiated a sizeable settlement offer from the corporation, which the officer happily accepted. The corporation forwarded the settlement funds to the attorney. The attorney promptly notified the officer of her receipt of the funds and, after depositing the funds to which she was due under the fee agreement into her operating account, immediately forwarded the remainder to the officer with an accounting of the settlement funds and their distribution. Was the attorney's conduct proper? A) No, because the attorney acquired a proprietary interest in the officer's compensation dispute with the corporation. B) No, because the officer did not sign the written fee agreement despite several requests by the attorney. C) Yes, because the attorney was not required to advise the client to consult independent legal counsel before agreeing to a contingency fee. D) Yes, because the client accepted the settlement offer and received the proper amount due under the fee agreement.

B) No, because the officer did not sign the written fee agreement despite several requests by the attorney. Answer choice B is correct. A contingency fee agreement must be in writing and signed by the client. Here, despite the attorney's attempts to have her client sign the written agreement, the client did not sign the agreement. Answer choice A is incorrect because, while a lawyer generally must not obtain a proprietary interest in the cause of action or subject matter of litigation in which a client is represented, there is an exception when the lawyer contracts for a reasonable contingency fee, provided the case is not a criminal or matrimonial/domestic matter. Here, since the matter involves a contractual dispute, the attorney could enter into a contingency fee agreement with the officer. Answer choice C is incorrect because the ethical rules do not require an attorney to advise a client to consult independent legal counsel before agreeing to a contingency fee. Answer choice D is incorrect. A lawyer must generally communicate all bona fide offers of settlement to the client, and the ultimate decision as to whether to accept rests with the client. Here, the attorney complied with this ethical rule by notifying the officer of the settlement offer made by the corporation. It was the officer who made the decision to accept the offer. However, if there is a dispute as to the amount of the lawyer's fee, the lawyer is required to retain the disputed amount in a client trust account until the dispute is resolved. Here, the contingency fee agreement is not valid because it was not signed. As such, the attorney was required to place the amount due under the contingency fee into her client trust account, rather than her operating account.

A plaintiff was represented by an attorney in a breach of contract lawsuit against a defendant corporation. The plaintiff's claim was dismissed with prejudice by the court for failure to state a claim upon which relief could be granted. The corporation's lawyer reasonably believed that the plaintiff's attorney had not investigated the plaintiff's factual or legal claims with due diligence before filing the complaint. Under the rules of the applicable jurisdiction, the corporation could seek sanctions against the plaintiff's attorney to recover some of its legal expenses. However, the corporation could obtain a more valuable judgment by suing the plaintiff's attorney in a separate negligence action. Thus, instead of filing a motion for sanctions, the corporation's lawyer filed a negligence action against the plaintiff's attorney. The corporation's lawyer believes that she can prove that the plaintiff's attorney failed to use reasonable care to investigate the plaintiff's claim before filing a lawsuit against the corporation. Is the plaintiff's attorney likely to be subject to civil liability to the corporation in the negligence action? A) No, because the corporation could have pursued sanctions against the plaintiff's attorney in the breach of contract lawsuit. B) No, because the plaintiff's attorney owed no duty of care to the corporation. C) Yes, because the action filed by the plaintiff's attorney was dismissed for failure to state a claim upon which relief could be granted. D) Yes, because the plaintiff's attorney's failure to exercise reasonable care caused the corporation to incur unnecessary legal expenses.

B) No, because the plaintiff's attorney owed no duty of care to the corporation. Answer choice B is correct. A lawyer representing a party in litigation has no duty of care to the opposing party, and thus no liability for lack of care, except in unusual situations such as when a litigant has been invited to rely on the opinion or legal services of the lawyer (e.g., an opinion letter from opposing counsel as part of a settlement). Thus, the plaintiff's attorney is not likely to be subject to civil liability to the corporation for negligence. Answer choice A is incorrect. Litigation sanctions and civil liability are two distinct types of remedies. The phrase "subject to litigation sanction" asks whether the conduct would subject the lawyer or the lawyer's law firm to a fine, fee forfeiture, disqualification, punishment for contempt, or other sanction by a legal tribunal. The phrase "subject to civil liability," on the other hand, asks whether the conduct would subject the lawyer or the lawyer's law firm to civil liability, such as a claim arising from malpractice, misrepresentation, or breach of fiduciary duty. Although the corporation could have sought litigation sanctions against the plaintiff's attorney, that would not make the plaintiff's attorney subject to civil liability for negligence. Answer choice C is incorrect. The plaintiff's attorney owed no duty of care to the corporation, so the fact that the action filed by the plaintiff's attorney was dismissed for failure to state a claim upon which relief could be granted does not make the plaintiff's attorney subject to civil liability to the corporation for negligence. Answer choice D is incorrect. Even if the plaintiff's attorney had failed to exercise reasonable care in bringing the action, causing the corporation to incur unnecessary legal expenses, it would not make the plaintiff's attorney subject to civil liability to the corporation for negligence because the plaintiff's attorney had no duty to act with due care for the corporation's interests.

Two licensed attorneys, Alpha and Beta, were partners in the Alpha Beta law firm. Alpha met with a magician who had a potential breach of contract claim against a casino. Before disclosing any information about the contractual dispute, Alpha asked the magician to provide the name of the casino. Alpha immediately realized that Beta regularly represented the casino in a series of real estate transactions. For this reason, Alpha told the magician that she could not represent him, nor could she discuss the matter any further. Six months later, an attorney from another law firm filed a lawsuit against the casino on behalf of the magician. The casino asked Beta to defend it against the magician's lawsuit, and per the law firm's policy when taking on a new case, Beta passed around a conflict of interest questionnaire before accepting the representation. Alpha accurately reported what had transpired between her and the magician six months earlier. Based on this information, Beta did not screen Alpha from partaking in the litigation. Is Beta subject to discipline? A) No, because Alpha's conflict of interest was not imputed to Beta. B) No, because there was no conflict of interest since Alpha did not obtain any information from the magician that could be significantly harmful to him in the pending lawsuit. C) Yes, because the magician was a prospective client of Alpha in the same matter for which the casino sought Beta's representation. D) Yes, because Beta did not timely screen Alpha from partaking in the litigation.

B) No, because there was no conflict of interest since Alpha did not obtain any information from the magician that could be significantly harmful to him in the pending lawsuit. Answer choice B is correct. Model Rule 1.18(c) prohibits a lawyer from representing a client with interests materially adverse to those of a prospective client in a matter if the lawyer received information from the prospective client that could be significantly harmful to the prospective client in that matter. The facts indicate that Alpha limited the information obtained in the initial interview with the prospective client (the magician) to the name of the casino. Because Alpha did not obtain any significantly harmful information related to the magician's prospective engagement, there is no conflict of interest between Alpha and the magician. Therefore, since there is no conflict to be imputed to Beta, Beta is not prevented from representing the casino. Answer choice A is incorrect because there was no conflict of interest between Alpha and the magician to be imputed to Beta. Answer choice C is incorrect. Because Alpha did not obtain any significantly harmful information from the magician, there is no conflict to be imputed to Beta. Answer choice D is incorrect. No screening is necessary because Model Rule 1.18 requires screening and other precautions only when a lawyer receives significantly harmful information about a prospective client.

A sole practitioner who specialized in family law shared office space with a sole practitioner who specialized in criminal law. Since they frequently referred matters to one another, they decided to formalize this arrangement by entering into a reciprocal referral agreement under which they each agreed to refer matters that fit within their respective specialties to each other. They agreed that the arrangement would remain in effect for one year, at which point they would review it. The agreement did not prohibit either attorney from referring cases to other attorneys, and neither hesitated to recommend a different attorney whom they believed would better handle a particular case. When making referrals to each other, neither attorney informed clients of their agreement. Were the attorneys' actions regarding the referral agreement proper? A) No, because reciprocal referral agreements are prohibited by the Model Rules of Professional Conduct. B) No, because they did not inform clients of the arrangement. C) Yes, because the arrangement was not exclusive. D) Yes, because the arrangement did not provide for either attorney to be paid for referring clients.

B) No, because they did not inform clients of the arrangement. Answer choice B is correct. Under certain circumstances, an attorney may enter into a reciprocal referral agreement whereby each attorney agrees to refer clients to the other. The agreements must not be exclusive or of an indefinite duration, and the client must be informed of the nature and existence of the agreement. In this case, the clients were not informed of the agreement, and thus the attorneys did not act properly. Answer choice A is incorrect because reciprocal referral agreements are permitted by the Model Rules of Professional Conduct. Answer choice C is incorrect because although the agreement was not exclusive, the attorneys acted improperly when they failed to notify the clients of the agreement. Answer choice D is incorrect because although the attorneys did not pay each other for the referrals, they acted improperly by failing to inform clients of the agreement.

A real estate attorney was going for a drive in an isolated area in the mountains one evening when she happened to pass by a car that had just crashed into a tree. The attorney immediately pulled over in order to help the driver out and call for help. The attorney went to passenger side of the car and saw that the driver was still alive. The driver, who was severely injured, told the attorney that he was afraid that he was going to die, and that he did not even have a will so that he could leave his belongings to his children. The attorney told the driver that she happened to be an attorney and offered to write a will for him, even though she had never written a will before. The driver accepted the offer, and the attorney drafted a basic will for him, leaving the driver's belongings to his two children. Is the attorney subject to discipline? A) No, because attorneys are required to give legal assistance in emergency situations despite their lack of competence. B) No, because this was an emergency situation and consultation with another lawyer would have been impractical. C) Yes, because even in emergency situations, attorneys are not permitted to give assistance in matters in which they do not have the skill ordinarily required. D) Yes, because the attorney had never written a will before.

B) No, because this was an emergency situation and consultation with another lawyer would have been impractical. Answer choice B is correct. In an emergency, a lawyer may give advice or assistance in a matter in which the lawyer does not have the skill ordinarily required if referral to or consultation or association with another lawyer would be impractical. Assistance should be limited, however, to what is reasonably necessary under the circumstances. Here, there was an emergency because the driver was afraid that he was going to die, and he did not have a will so that he could leave his belongings to his children. It would have been impractical for the attorney to consult with a wills and estates lawyer, so it was permissible for the attorney, who had never written a will before, to write one in this situation without being subject to discipline. For this reason, answer choice C is incorrect. Answer choice A is incorrect because according to the Model Rules, attorneys are permitted, not required, to give legal assistance in emergency situations despite their lack of competence. Answer choice D is incorrect. Although the attorney had never written a will before, in this emergency situation, her lack of competence was excused, and it would have been impractical for her to consult with another attorney.

An attorney practices law in a state that has experienced a business recession and where several banks have failed and others are severely pressed to preserve their solvency. The attorney maintains a Clients' Trust Account in a bank and that account is insured by the Federal Deposit Insurance Corporation against losses up to $100,000. The attorney also maintains his regular office account in the same bank and that account is insured to $100,000. During a particularly busy time, the attorney's bookkeeper told the attorney that the balance in the Clients' Trust Account had increased to $150,000. The bookkeeper noted that the office account had a balance of $30,000. Which of the following courses of action by the attorney would be proper? A) Leave the Clients' Trust Account as is if the balance is likely to decrease to less than $100,000 within the next ten days. B) Open another Clients' Trust Account in another bank within the state, and transfer some funds to the second Clients' Trust Account to maintain a fully insured balance in both accounts. C) Open another Clients' Trust Account in a bank situated in another state, which is not experiencing bank failures, and transfer some funds to the second Clients' Trust Account to maintain a fully insured balance in both accounts. D) Temporarily transfer $50,000 from the Clients' Trust Account to the office account so the balance in both accounts is fully within insured limits.

B) Open another Clients' Trust Account in another bank within the state, and transfer some funds to the second Clients' Trust Account to maintain a fully insured balance in both accounts. Answer choice B is correct. It would be proper to transfer funds to another clients' trust account at another bank to ensure that all funds were properly insured by the Federal Deposit Insurance Corporation (FDIC). Under MRPC 1.15 (a), a lawyer must hold funds that are in a lawyer's possession in connection with a representation separate from the lawyer's own funds. Funds must be kept in a separate clients' trust account maintained in the state where the lawyer's office is situated (or elsewhere with the consent of the client or third person). Answer choice A is incorrect because, given the bank failures in the state, it would be improper for the attorney to leave funds that would exceed the amount insured by the FDIC in the original bank. Answer choice C is incorrect because clients' trust accounts must be maintained in the state in which the attorney's office is situated. Answer choice D is incorrect, as client funds must be kept separate from the lawyer's own funds, so it would be improper to transfer any of the funds to the office account.

An attorney represented a plaintiff in litigation that was settled, with the plaintiff's approval, for $25,000. The attorney received a check in that amount from the defendant, payable to the attorney's order. The attorney endorsed and deposited the check in the attorney's Clients' Trust Account. The attorney promptly notified the plaintiff and billed him $5,000 for legal fees. The plaintiff disputed the amount of the fee and wrote the attorney, stating, "I will agree to pay $3,000 as a reasonable fee for the work you did, but I will not pay anything more than that." Based on the above facts, which of the following actions is proper for the attorney to take? A) Retain the entire $25,000 in the attorney's Clients' Trust Account until the fee dispute is settled. B) Send the plaintiff $20,000, transfer $3,000 to the attorney's office account, and retain $2,000 in the attorney's Clients' Trust Account until the dispute is settled. C) Send the plaintiff $20,000 and transfer $5,000 to the attorney's office account. D) None of the above is proper.

B) Send the plaintiff $20,000, transfer $3,000 to the attorney's office account, and retain $2,000 in the attorney's Clients' Trust Account until the dispute is settled. Answer choice B is correct. The attorney is required to pay the plaintiff the $20,000 undisputed portion of the settlement, and may properly transfer the undisputed fee of $3,000 to the attorney's office account. The $2,000 that remains in dispute must remain in the Clients' Trust Account until the dispute is resolved. Answer choice A is not proper, as MRPC 1.15(e) requires the attorney to promptly pay the plaintiff the undisputed portion of the settlement ($20,000). Answer choice C is not proper, as $2,000 of the $5,000 fee remains disputed by the plaintiff and must remain in the Clients' Trust Account pursuant to MRPC 1.15.

A rising second-year law school student was employed as a summer intern at a law firm. The intern attended a deposition of a client who was represented by an associate attorney of the firm. The associate cautioned the intern not to discuss any aspect of the deposition with anyone. The intern discussed his own personal observations of the client's demeanor and his own conclusions about the client's truthfulness with his roommate, which did not result in any harm to the client. Of the following, which would likely provide the associate with the best defense to a disciplinary action based on the intern's actions? A) The intern only discussed his own personal observations and conclusions with his roommate. B) The associate cautioned the intern not to discuss any aspect of the deposition with anyone. C) The intern was not a licensed attorney. D) The client did not suffer harm as a consequence of the intern's conversation with his roommate.

B) The associate cautioned the intern not to discuss any aspect of the deposition with anyone. Answer choice B is correct. A lawyer must make reasonable efforts to prevent the inadvertent or unauthorized disclosure of information relating to the representation of a client by persons who are participating in the representation of the client or who are subject to the lawyer's supervision. The associate's warning to the intern not to discuss any aspect of the disposition with anyone likely constitutes reasonable precautions. Answer choice A is incorrect because the duty of confidentiality extends beyond communications made by a client and her agents to a lawyer or his staff to any information relating to the representation, and it includes personal observations. Answer choice C is incorrect because, although the intern is not subject to discipline under the Model Rules of Professional Conduct because he is not a lawyer, the associate is responsible for taking reasonable efforts to prevent the inadvertent or unauthorized disclosure of information relating to the representation and for taking reasonable precautions to safeguard such information from disclosure by non-lawyer employees and agents of the firm who are under the associate's supervision. Answer choice D is incorrect because a lawyer is subject to discipline for violation of a Model Rule of Professional Conduct even though the client does not suffer harm as a consequence of the violation.

Which of the following statements regarding a lawyer's professional competence is FALSE? A) Lawyers have a duty to be competent. B) The evidentiary standard for proving an ethical violation is a preponderance of the evidence. C) A lawyer must decline representation if the lawyer is not competent to represent the client or cannot become competent without unreasonable delay. D) A lawyer must inform the client when the lawyer has made an error.

B) The evidentiary standard for proving an ethical violation is a preponderance of the evidence.

Which of the following is NOT part of the test to determine whether a conflict exists between a current and former client? A) The scope of the prior representation B) The lawyer actually obtained information from the prior representation C) The lawyer could have obtained information from the prior representation D) The information would be useful to the current litigation

B) The lawyer actually obtained information from the prior representation

Which of the following statements regarding the ethical obligations under the Model Rules of Professional Conduct of a lawyer who represents a corporation is FALSE? A) The lawyer represents the corporation, not the corporation's constituents (e.g., officers, directors, employees). B) The lawyer cannot represent both the corporation and an employee. C) The lawyer who knows of a violation of the law that will likely result in substantial injury to the corporation may refer the matter to the highest authority of the corporation. D) If the organization does not correct a violation of the law, a lawyer is permitted to resign by "noisy withdrawal."

B) The lawyer cannot represent both the corporation and an employee.

Which of the following theories may NOT serve as a basis for a malpractice action against a lawyer? A) Breach of contract B) Violation of the Rules of Professional Conduct C) Breach of a fiduciary relationship D) Commission of an intentional tort

B) Violation of the Rules of Professional Conduct

Alpha and Beta practiced law under the firm name of Alpha & Beta. When Beta died, Alpha did not change the firm name. Thereafter, Alpha entered into an arrangement with another attorney, Gamma. Gamma pays Alpha a certain sum each month for office space, for use of Alpha's law library, and for secretarial services. Alpha and Gamma each have their own clients, and neither participates in the representation of the other's clients or shares in fees paid. On the entrance to the suite of offices shared by Alpha and Gamma are the words "Law Firm of Alpha, Beta & Gamma." Is Alpha subject to discipline? A) Yes, because Beta was deceased when Alpha made the arrangement with Gamma. B) Yes, because Gamma is not a partner of Alpha. C) No, because Alpha and Beta were partners at the time of Beta's death. D) No, because Gamma is paying a share of the rent and office expenses.

B) Yes, because Gamma is not a partner of Alpha. Answer choice B is correct. MRPC 7.5(d) prohibits lawyers from stating or implying that they practice in a partnership or other organization when that is not the fact. The sign at the entrance to the suite of offices represents to the public that Alpha and Gamma practice together in a single law firm and it is therefore prohibited by MRPC 7.5(d), as well as by MRPC 7.1, which prohibits false or misleading communications about a lawyer's practice. Answer choice A is incorrect, as MRPC 7.5 and its Comment permit a lawyer to use the name of a deceased member of a firm when there has been a continuing succession in the firm's identity, regardless of whether the deceased partner had given permission for the continuing use. Answer choice C is incorrect because Alpha was indeed permitted to continue to use Beta's name in his firm's name under MRPC 7.5. However, MRPC 7.5(d) prohibits him from stating or implying that he and Gamma practice together in a single law firm. Answer choice D is incorrect, as paying a share of the rent and other office expenses does not make Gamma a partner of Alpha, and MRPC 7.5(d) prohibits Alpha from stating or implying that he and Gamma practice together in a single law firm.

An attorney accepted a libel case, though he had no experience in that type of action and had little knowledge related to the issues involved. He informed his client of his inexperience but promised to work with a fellow competent attorney to handle the claim effectively. He received the client's written consent acknowledging this fact and allowing the attorney to proceed with the representation. The attorney then called his friend, a fellow attorney who had graduated at the top of her law school class but who likewise had no libel experience or knowledge herself, for advice on how to proceed. The friend provided good advice on general litigation techniques, and the attorney heeded this advice but took no other action. The attorney ultimately lost at trial. Would the attorney's representation of the client subject him to discipline? A) Yes, because he lacked the knowledge or experience to successfully represent the client. B) Yes, because he lacked the knowledge or experience to competently represent the client. C) No, because he consulted his friend on the matter. D) No, because the client consented in writing to the representation.

B) Yes, because he lacked the knowledge or experience to competently represent the client. Answer choice B is correct. An attorney lacking the necessary knowledge or experience must: (i) decline or withdraw from representing the client, (ii) become competent without unreasonable delay, or (iii) associate with competent counsel. An attorney should not accept representation in a matter unless it can be performed competently. An attorney who represents a client incompetently is subject to discipline, regardless of causation or any financial loss to the client. Here, the attorney was not knowledgeable in the area of libel, and advice on general litigation tactics would not provide him with the knowledge he needed to competently represent his client in that area. Answer choice A is incorrect. Because a lawyer who violates an ethics rule may also potentially be subject to civil liability for malpractice, the fact that the attorney lost at trial may subject him to civil liability for the client's damages. However, competent representation of a client does not require success at trial. An attorney does not have to win the case to avoid discipline under the ethical rules; he must merely be competent. Therefore, the mere fact that the attorney did not succeed at trial does not violate the ethical rules. Answer choice C is incorrect because the attorney's consultation with his friend would not constitute an association with competent counsel, as she had no knowledge or experience with libel cases either. Answer choice D is incorrect because a client cannot consent to incompetent representation.

An attorney litigated a non-jury case before a judge. After losing the case, the attorney filed an appeal arguing that the judge misapplied the law. After also losing the appeal, the attorney, attending a continuing legal education class, stood up and made a statement to everyone present that the judge, identifying her by name, "is crazy. She has some sort of mental disorder. She is not fit to sit on the bench." Is the attorney subject to discipline for this statement? A) Yes, because it was a statement affecting an individual's professional reputation that reflected a reckless disregard for the truth. B) Yes, because it reflected a reckless disregard for the truth about the qualifications or integrity of a judge. C) No, because the attorney was among fellow attorneys rather than the general population or the media. D) No, because his statement was based on the judge's misapplication of the law.

B) Yes, because it reflected a reckless disregard for the truth about the qualifications or integrity of a judge. Answer choice B is correct. An attorney must not make a statement that the attorney knows to be false or with reckless disregard as to its truth or falsity concerning the qualifications or integrity of a judge, adjudicatory officer, or public legal officer, or of a candidate for election or appointment to judicial or legal office. Here, the attorney made a very strong statement about the judge's mental health and qualification to remain on the bench without any basis of truth. Answer choice A is incorrect because a statement affecting another individual's professional reputation made with a reckless disregard for the truth might not automatically subject the attorney to discipline, but because the statement pertained to the integrity or qualifications of the judge, it would subject him to such discipline. Answer choice C is incorrect because the audience receiving the message is not relevant in determining whether the attorney's statement would subject him to discipline. Answer choice D is incorrect because, even if the judge had misapplied the law, the statement regarding the judge's mental health and ability to serve on the bench demonstrated a reckless disregard for the truth.

The state bar association has offered a judge and her spouse free transportation and lodging to attend its institute on judicial reform. The judge is expected to deliver a banquet speech. Is it proper for the judge to accept this offer? A) Yes, unless the value of the transportation and lodging exceeds $500. B) Yes, because the activity is devoted to the improvement of law. C) No, if members of the bar association regularly appear in the judge's court. D) No, because the bar association is offering free transportation to the judge's spouse.

B) Yes, because the activity is devoted to the improvement of law. Answer choice B is correct. Under CJC Rule 3.13(C), a judge may generally accept an invitation to the judge and the judge's spouse, domestic partner, or guest to attend without charge an event associated with a bar-related function or other activity relating to the law. A judge may also accept reimbursement of necessary and reasonable expenses for travel, food, lodging, or other incidental expenses, if the expenses or charges are associated with the judge's participation in extrajudicial activities permitted by the Code of Judicial Conduct. CJC Rule 3.14(A). Reimbursement of expenses for necessary travel, food, lodging, or other incidental expenses must be limited to the actual costs reasonably incurred by the judge and, when appropriate to the occasion, by the judge's spouse, domestic partner, or guest. CJC Rule 3.14(B). The judge must publicly report acceptance of such invitations and the amount of any reimbursement for attending. Here, the judge would be attending a state bar event devoted to improvement of the law, so it is proper to accept the offer. Answer choice A is not correct, as the CJC imposes no $500 limit on the food and lodging reimbursement the judge may seek. The only limit is that the amount be necessary and reasonable. Answer choice C is not correct, as there is no prohibition on attending a bar association-related event and receiving reimbursement when members of the bar association regularly appear in the judge's court. Answer choice D is not correct, as a judge's spouse may be included under CJC Rules 3.13 and 3.14.

An attorney and a restaurant owner entered into a reciprocal referral arrangement. The attorney agreed to prominently display ads for the restaurant in her office, and to mention the restaurant to all of her clients who requested a recommendation of a nearby place to eat. In return, the owner agreed to prominently display ads for the attorney's firm in the restaurant and to recommend the attorney to any of his customers who indicated a need for the services provided by the attorney. The reciprocal referral agreement was not exclusive, and the clients and customers would be informed of the existence and nature of the agreement. Is the attorney subject to discipline for entering into this agreement? A) Yes, because she asked the owner to place ads for the firm in the restaurant. B) Yes, because the agreement provided something of value to the restaurant owner in return for recommending the attorney's services. C) No, because she did not pay the restaurant owner for the referrals. D) No, because the agreement is not exclusive, and the clients and customers will be informed of the existence and nature of the agreement.

B) Yes, because the agreement provided something of value to the restaurant owner in return for recommending the attorney's services. Answer choice B is correct. MRPC 7.2(b) prohibits a lawyer from giving anything of value to a person for recommending the lawyer's services. Subsection (4) of that rule provides for an exception for reciprocal referral agreements with other lawyers or with non-lawyer professionals. However, even if the restaurant owner were considered a "professional" under this exception, this is not a mere reciprocal referral agreement. A reciprocal referral agreement would only require each professional to refer clients to the other professional. Here, the attorney's display of additional advertising is an additional exchange of value that goes beyond a mere referral. Therefore, the attorney is subject to discipline for this arrangement. Answer choice A is incorrect because the Model Rules do not prohibit a lawyer from asking others to post advertisements concerning the lawyer's services. Answer choice C is incorrect, because the attorney's promise to display the restaurant ads and to mention the restaurant to clients seeking a recommendation is something of value. Answer choice D is incorrect because, even if a reciprocal referral agreement would be permitted with the restaurant owner, the attorney's display of additional advertising is an additional exchange of value that goes beyond a mere referral. Therefore, the attorney is subject to discipline for this arrangement.

An attorney is widely regarded as an exceptionally competent practitioner in the field of criminal law. A client of the attorney became the subject of a grand jury investigation in a matter that could result in a felony indictment. The client lacked sufficient funds to pay for the attorney's services beyond the grand jury stage. He asked the attorney to provide limited representation for a flat fee. Under the arrangement he proposed, the attorney would advise the client concerning the grand jury investigation, but the representation would end when an indictment was returned or the grand jury decided not to indict. The attorney fully advised the client of the practical and legal aspects of the client's proposal. Is it proper for the attorney to accept this limited representation? A) Yes, because the client and not the attorney suggested this arrangement. B) Yes, because the attorney and the client may agree to limit the scope of the representation so long as the limitation is reasonable under the circumstances. C) No, because the attorney should not limit the scope of the representation based on the client's ability to pay. D) No, because the scope of the representation may not be limited in a criminal case.

B) Yes, because the attorney and the client may agree to limit the scope of the representation so long as the limitation is reasonable under the circumstances. Answer choice B is correct. A lawyer and client may agree to limit the scope of the lawyer's representation as long as the client has been fully informed. The limitation on the representation must be reasonable. MRPC 1.2(c). In this case, it is entirely permissible for the client and the lawyer to agree to a limited representation that covers only the client's involvement in the grand jury proceeding. Answer choice A is incorrect, as limitations on the scope of a representation will not be judged based upon whether they were suggested by the lawyer or the client. Answer choice C is incorrect, as lawyers do not have an obligation to provide free legal services to everyone. The ability to pay is one factor in crafting a limited representation agreement and, in this case, it puts the client in a better position than if he were not represented by the lawyer at all. Answer choice D is incorrect, as the Rules do not prohibit limited representation in criminal cases.

A client approached an attorney to represent her in a products liability case because the attorney claimed to be a specialist in products liability actions. He told the client that the National Council on Products Liability had certified him as a specialist in products liability resolution, even though he had lost half of the 100 products liability actions he had taken. The client consented to representation in writing after this discussion. The attorney lost the case on a technicality that would have been known to any specialist in products liability, though a general practice attorney would not have recognized the issue. The entire case was lost based on this error. After demonstrating that she suffered damages, would the client be likely to recover in a civil liability action against the attorney? A) Yes, because the attorney lost her case. B) Yes, because the attorney deviated from the applicable standard of care for products liability attorneys. C) No, because the attorney did not deviate from the applicable standard of care of a general attorney. D) No, because the client consented to the representation in writing after full disclosure.

B) Yes, because the attorney deviated from the applicable standard of care for products liability attorneys. Answer choice B is correct. An attorney owes a duty of care to the client. The duty of care is generally the competence and diligence exercised by attorneys of similar experience under similar conditions. If an attorney represents to the client that he has specialized expertise, then the attorney will be held to the standard of care of a specialist. Here, the standard of a specialist would apply, and the attorney likely deviated from this standard. Answer choice A is incorrect because losing a case alone is not a basis for civil liability. What is relevant is the standard of care that the attorney exercised in representing the client. Answer choice C is incorrect because while the attorney may not have deviated from a general attorney's standard of care, he did deviate from the standard of care of a specialist. Answer choice D is incorrect because if the attorney was negligent, then the client's consent would not mitigate his liability.

In accord with state law, parties to a civil nuisance action participated in a court-ordered mediation of their dispute. Both parties were non-lawyers and proceeded pro se, and together selected an attorney who agreed to serve as mediator. The attorney did not inform them that she did not represent either of them. The attorney does occasionally represent clients in nuisance actions, but she was otherwise unfamiliar with the parties or their specific complaint. As required by the state code of conduct for mediators, the attorney treated the parties with impartiality. Is the attorney subject to discipline? A) Yes, because the attorney may not serve as a mediator since she represents clients in nuisance actions. B) Yes, because the attorney failed to inform the parties that she did not represent them during the mediation. C) No, because the parties selected the attorney as their mediator with the knowledge that she was a lawyer. D) No, because the attorney treated the parties with impartiality as required by the state code of conduct for mediators.

B) Yes, because the attorney failed to inform the parties that she did not represent them during the mediation. Answer choice B is correct. Although the Model Rules recognize that a lawyer may serve as a mediator or other third-party neutral, the Model Rules specifically require a lawyer who does so to inform unrepresented parties that the lawyer is not representing them. MRPC 2.4(b). Answer choice A is incorrect. A lawyer may generally not represent anyone in connection with a matter in which the lawyer participated personally and substantially as a mediator or other neutral third party unless all parties to the proceeding give informed consent, confirmed in writing. However, the fact that the attorney's law practice involved the same area of the law from which the dispute in question arose does not give rise to a conflict of interests that would disqualify the attorney from serving as a mediator here. Answer choice C is incorrect. Even though the parties were aware that the attorney was licensed to practice law in the state, the attorney was nevertheless required to inform the unrepresented parties that she did not represent them. Answer choice D is incorrect because, although the attorney treated the parties with impartiality while serving as a mediator in accord with the state code of conduct for mediators, the attorney failed to inform the parties who were proceeding pro se that she did not represent them.

An attorney in his capacity as part-time assistant county attorney represented the county in a criminal non-support proceeding against a husband. This proceeding concluded with an order directing the husband to pay or be jailed. The husband refused to pay. The attorney, pursuant to applicable rules, is permitted to maintain a private law practice. The wife has discovered some assets of the husband. The attorney now has accepted employment from the wife to maintain a civil action against the husband to recover out of those assets arrearages due to the wife under the wife's support decree. The attorney did not obtain consent from the county attorney or from the husband to represent the wife in the civil action. Is the attorney subject to discipline for accepting employment in the wife's civil action against the husband? A) Yes, because the attorney did not obtain the husband's consent to the representation. B) Yes, because the attorney had personal and substantial responsibility in the first proceeding. C) No, because the attorney's responsibility in his public employment has terminated. D) No, because the attorney is representing the wife's interests in both the criminal and the civil proceedings.

B) Yes, because the attorney had personal and substantial responsibility in the first proceeding. Answer choice B is correct. Under MRPC 1.11, the attorney, who had personally and substantially participated in the first proceeding, could not accept the employment without the county's informed, written consent. Answer choice A is not correct, as the husband's consent is not needed. It is the county's consent that controls under MRPC 1.11. Answer choice C is not correct, as MRPC 1.11 imposes a duty to obtain informed, written consent from the county in order to represent the wife, even though the attorney's responsibility in his public employment has terminated. Answer choice D is not correct, as it does not matter that the attorney is representing the same side as both a county attorney and a private attorney. The county's informed, written consent is needed to permit the representation.

An attorney is a well-known, highly-skilled litigator. The attorney's practice is in an area of law in which the trial proceedings are heard by the court without a jury. In an interview with a prospective client, the attorney said, "I make certain that I give the campaign committee of every candidate for elective judicial office more money than any other lawyer gives, whether it's $500 or $5,000. Judges know who helps them get elected." The prospective client did not retain the attorney. Is the attorney subject to discipline? A) Yes, if the attorney's contributions are made without consideration of candidates' merits. B) Yes, because the attorney implied that she receives favored treatment from judges. C) No, if the attorney's statements were true. D) No, because the prospective client did not retain the attorney.

B) Yes, because the attorney implied that she receives favored treatment from judges. Answer choice B is correct. It is professional misconduct for a lawyer to state or imply an ability to improperly influence a government agency or official. MRPC 8.4(e). Here, the attorney's statements implied an ability to improperly influence judges. Answer choice A is not correct, as the contributions are irrelevant. The professional misconduct occurs as a result of the attorney's statements to the prospective client. Answer choice C is not correct, as the attorney would be subject to discipline even if the statements were true. It is professional misconduct for a lawyer to state or imply an ability to improperly influence a government agency or official. MRPC 8.4(e). Answer choice D is not correct, as the violation of MRPC 8.4(e) occurs regardless of whether or not the prospective client retains the lawyer.

A trust company entered into the following arrangement with an attorney, who was newly admitted to the bar. The trust company would provide the attorney with free office space in the building in which the company had its offices. If a customer of the trust company contacted it about a will, an officer of the company, who is not a lawyer, would advise the customer and help the customer work out the details of the will. The customer would be informed that the necessary documents would be prepared by the trust company's staff. The completed documents would be submitted by an officer of the company to the customer for execution. The attorney, in accordance with a memorandum from the company's trust officer detailing the plan, would prepare the necessary documents. The attorney would never meet with the customer and would not charge the customer for these services. The attorney would be free to engage in private practice, subject only to the limitation that the attorney could not accept employment adverse to the trust company. Is the attorney subject to discipline for entering into the arrangement with the trust company? A) Yes, because the attorney is restricting his right to practice. B) Yes, because the attorney is aiding the trust company in the practice of law. C) No, because the attorney is not charging the customer for his services. D) No, because the attorney is not giving advice to the company's customers.

B) Yes, because the attorney is aiding the trust company in the practice of law. Answer choice B is correct. A lawyer must not assist a person who is not admitted to practice in a jurisdiction in the unauthorized practice of law. MRPC 5.5(a). Here, the trust company is practicing law by providing legal advice on wills to its customers, and the attorney is assisting the company in this practice by preparing the necessary documents in accordance with a memorandum from the company's trust officer based on a non-lawyer's legal advice to the customer. The attorney would be subject to discipline for helping the company in the practice of law. Answer choice A is not correct because the limitation that the attorney could not accept employment adverse to the trust company merely reflects that the company refuses to waive any applicable conflicts of interest that would arise under the model rules if the attorney accepted such employment. In other words, the attorney is not restricting his right to practice by agreeing to avoid conflicts of interest that the Model Rules would already prohibit. Answer choice C is not correct, as it is irrelevant to the ethical violation of assisting a person who is not admitted to practice in the unauthorized practice of law whether or not the attorney is paid by the client. Answer choice D is not correct, as the violation here is assisting a person who is not admitted to practice in the unauthorized practice of law. It does not matter if the attorney gave advice to the trust company's customers, only that the attorney aided in the unauthorized practice by the trust company.

An attorney filed a medical malpractice action on behalf of a client against a hospital. At the time of filing the action, the attorney, having taken the case on a contingency fee basis, anticipated winning a sizable verdict. At the conclusion of discovery, the attorney had learned additional information that made the prospect of success unlikely. Shortly before trial, to the attorney's surprise, the hospital made a settlement offer. When the client rejected the offer, the attorney, wishing to devote himself to other cases that were more likely to prove profitable, told the client that he was withdrawing from the case. When the client balked at the attorney's withdrawal, the attorney promised the client that he would take steps to mitigate the material adverse effect of his withdrawal. However, the attorney acknowledged that he would not be able to eliminate all adverse effects of his withdrawal. Is the attorney likely subject to discipline for his withdrawal? A) Yes, because the attorney may not withdraw from a representation without the client's permission under any circumstances. B) Yes, because the attorney lacks good cause for withdrawing from the representation. C) No, because the attorney may ethically decline or withdraw from a representation at any time for any reason. D) No, because the attorney promised to take steps to mitigate the material adverse effects of his withdrawal.

B) Yes, because the attorney lacks good cause for withdrawing from the representation. Answer choice B is correct. There are limited situations in which a lawyer can withdraw from representation of client when doing so will have a material adverse effect on the client's interest. The attorney wishes to abandon the current lawsuit to devote his himself to other cases that are more likely to be profitable. However, the attorney has not indicated that the continuing the representation will impose an unreasonable financial burden on him. Also, although a lawyer is permitted to withdraw from a representation if "other good cause" exists, the facts do not articulate another basis for permitting the attorney to withdraw. Answer choice A is incorrect because a lawyer may withdraw from a representation without the client's permission in certain circumstances, even when the lawyer's withdrawal will have material adverse effect on the client's interest. However, none of those circumstances are likely present under these facts. Answer choice C is incorrect because, while a lawyer is generally free to decline a representation for any reason, a lawyer is not similarly free to withdraw from a representation unless certain specific circumstances exist. Answer choice D is incorrect because it is not enough that a lawyer who seeks to withdraw from a representation take steps to mitigate the material adverse effects of his withdrawal. When the withdrawal will have a material adverse effect on a client, there are limited circumstances in which a lawyer may withdraw. As discussed in detail with regard to answer choice B, none of those circumstances are likely present under these facts.

An attorney represented a landlord in a variety of matters over several years. An elderly widow living on public assistance filed suit against the landlord alleging that the landlord withheld without justification the security deposit on a rental unit that she vacated three years ago. She brought the action for herself, without counsel, in small claims court. The attorney investigated the claim and learned that it was legally barred by the applicable statute of limitations, although the plaintiff's underlying claim was meritorious. The attorney told the landlord of the legal defense, but emphasized that the plaintiff's claim was just and that, in all fairness, the security deposit should be returned to her. The attorney told the landlord: "I strongly recommend that you pay the plaintiff the full amount with interest. It is against your long-term business interests to be known in the community as a landlord who routinely withholds security deposits even though the tenant leaves the apartment in good condition. Paying the claim now will prevent future headaches for you." Was the attorney's conduct proper? A) Yes, if the landlord did not object to the attorney's advice and paid the plaintiff's claim. B) Yes, because the attorney may refer to both legal and non-legal considerations in advising a client. C) No, unless the attorney's engagement letter informed the landlord that his advice on the matter would include both legal and non-legal considerations. D) No, because in advising the landlord to pay the full claim, the attorney failed to represent zealously the landlord's legal interests.

B) Yes, because the attorney may refer to both legal and non-legal considerations in advising a client. Answer choice B is correct. In representing a client, a lawyer is required to exercise independent professional judgment and render candid advice. In rendering advice, a lawyer may refer not only to law, but also to other considerations such as moral, economic, social, and political factors that may be relevant to the client's situation. MRPC 2.1. Answer choice A is incorrect because it is irrelevant what the landlord chose to do in response to the attorney's properly given advice. Answer choice C is incorrect, as MRPC 2.1 permits a lawyer to refer to both legal and non-legal considerations when rendering advice, and there is no requirement that a lawyer advise a client of this in an engagement letter. Answer choice D is incorrect. Although a lawyer must be dedicated and committed to the interests of the client, a lawyer has a duty to exercise independent professional judgment and render candid advice, which is what the attorney did in this case.

A restaurant owned by a partnership had financial trouble and sought to attract some new investors. The partnership hired an attorney to help it negotiate a deal that would bring in the needed cash. During the negotiations, the investors asked the partnership to provide a lawyer-prepared evaluation of whether the restaurant had complied with local and state laws during the past five years. The attorney met with the partners and informed them that he could provide such an evaluation, but that it would be very damaging to their chances of obtaining the funds. The restaurant had been cited for some serious health code violations in the recent past, but the matter was skillfully settled without publicity. The attorney had not been involved in the health code matter. The attorney informed the partners that any evaluation would need to discuss this health code matter, and if the investors decided not to pursue the investment, the partners might have a difficult time selling the restaurant. The partners consented to the attorney's performance of the evaluation even though they understood the risks. The attorney prepared the evaluation and delivered it to the investors. The investors chose not to provide the cash to the partners, and rumors leaked about the health code issues. Was the attorney's conduct proper? A) Yes, because the attorney did not represent the partnership in the health code matter. B) Yes, because the attorney obtained the partners' informed consent to the evaluation and the disclosure. C) No, because the attorney's evaluation damaged the partners' reputation. D) No, because a lawyer who is negotiating a deal on behalf of a client cannot also prepare an evaluation of the client for another party to the potential transaction.

B) Yes, because the attorney obtained the partners' informed consent to the evaluation and the disclosure. Answer choice B is correct. A lawyer may provide an evaluation of a matter affecting a client for the use of someone other than the client if the lawyer reasonably believes that making the evaluation is compatible with other aspects of the lawyer's relationship with the client. If the lawyer knows or reasonably should know that the evaluation is likely to affect the client's interests materially and adversely, the lawyer is not permitted to provide the evaluation unless the client gives informed consent. Since in this instance the attorney secured the client's consent, the attorney's conduct was proper. Answer choice A is incorrect because regardless of whether the attorney had represented the partnership in the health code matter, the attorney may disclose such information with the informed consent of the client, which the attorney secured. Answer choice C is incorrect. When a lawyer knows that an evaluation is likely to materially and adversely affect the client's interests, the lawyer may nevertheless provide the evaluation if the attorney obtains the client's informed consent. Answer choice D is incorrect because a lawyer may provide an evaluation of a matter affecting a client for the use of a third party if the lawyer reasonably believes that making the evaluation is compatible with other aspects of the lawyer's relationship with the client. While the attorney was aware that the evaluation would be very damaging to the partners' chances of obtaining the funds from the prospective investors, since the partners knowingly accepted those risks, the lawyer could reasonably believe that making the evaluation was compatible with lawyer's task of negotiating a deal on behalf of the partnership with the potential investors.

A client telephoned an attorney who had previously represented him. The client described a problem on which he needed advice and made an appointment for the following week to discuss the matter with the attorney. Prior to the appointment, the attorney performed five hours of preliminary research on the client's problem. At the end of the appointment the client agreed that the attorney should pursue the matter and agreed to a fee of $100 per hour. The client then gave the attorney a check for $5,000 to cover the five hours already worked and as an advance on additional fees and expenses. The attorney gave the check to the office bookkeeper with directions to deposit the check into the client trust account and immediately transfer $3,000 to the general office account to cover the five hours of research already conducted plus the 25 additional hours she would spend on the matter the following week. At that time, the attorney reasonably believed that she would spend 25 additional hours on the case. The bookkeeper followed these directions. The next week, the attorney worked diligently on the matter for 23 hours. Reasonably believing that no significant work remained to be done on the matter, the attorney directed the bookkeeper to transfer $200 from the general office account to the client trust account. The attorney then called the client and made an appointment to discuss the status of the matter. Is the attorney subject to discipline? A) Yes, because the attorney accepted legal fees in advance of performing the work. B) Yes, because the attorney transferred funds for unearned fees to the general office account. C) No, because the attorney transferred the $200 owed to the client from the general office account to the client trust account. D) No, because the attorney reasonably believed that she would spend 25 additional hours on the case.

B) Yes, because the attorney transferred funds for unearned fees to the general office account. Answer choice B is correct. MRPC 1.15(c) requires a lawyer to "deposit into a client trust account legal fees and expenses that have been paid in advance, to be withdrawn by the lawyer only as fees are earned or expenses incurred." Since the attorney had earned only $500, for five hours of work, at the time of receiving the client's check, only $500 should have been withdrawn from the trust account and placed in the general office account. Answer choice A is incorrect, as an attorney may accept legal fees in advance of performing legal work, as long as the lawyer complies with the requirement of MRPC 1.15(c) that the advance fees be deposited in a client trust account until earned. Answer choice C is incorrect. Under MRPC 1.15(c), a lawyer may not hold fees paid in advance in the lawyer's general office account and later transfer unearned portions to the client trust account, as the attorney did here. Rather, the lawyer must deposit the advance payment in the client trust account and withdraw the payment for fees as they are earned. Answer choice D is incorrect. Regardless of whether a lawyer expects to earn fees paid in advance, MRPC 1.15(c) requires the lawyer to deposit the advance payment in the client trust account until the fees are actually earned.

An attorney is a long-time member of the state legislature and serves on the legislative budget committee that funds the local trial courts in the state. The attorney also maintains a part-time law practice, as is permitted in the state. An influential businessperson who regularly makes significant contributions to the attorney's political campaigns asked the attorney to help his uncle, who was involved in a bitter divorce. The attorney called the trial judge sitting on the uncle's case, a personal friend of the attorney. In discussing some upcoming votes of the budget committee with the judge, the attorney mentioned that the uncle was the type of solid citizen and influential person who could help garner support for the budget and thus ensure the economic health of the judicial system. Is the attorney subject to discipline? A) Yes, if the trial judge ruled in the uncle's favor. B) Yes, because the attorney used her public position to attempt to influence a tribunal in a pending matter. C) No, if the attorney called the trial judge in her capacity as a legislator and not as the uncle's lawyer. DNo, because members of the state legislature are permitted by law to engage in part-time legal practice.

B) Yes, because the attorney used her public position to attempt to influence a tribunal in a pending matter. Answer choice B is correct. Under MRPC 3.5(a), a lawyer must not seek to influence a judge by means prohibited by law. In addition, under MRPC 8.4, it is professional misconduct for a lawyer to engage in behavior prejudicial to the administration of justice. Here, the attorney was using her public position as a legislator with control over trial court funding to attempt to influence the judge on a pending matter, and she would therefore be subject to discipline. Answer choice A is not correct, as it does not matter whether the judge actually was influenced and did rule in the uncle's favor. What controls whether the attorney is subject to discipline is that the attorney was using her public position to attempt to influence the judge in a pending matter. Answer choice C is not correct, as MRPC 3.5 and 8.4 apply to a lawyer regardless of whether she is acting as an advocate for the person for whom the lawyer is attempting to influence the court. Answer choice D is not correct, because it is not relevant. The attorney is not subject to discipline for engaging in part-time practice while a legislator. The attorney is subject to discipline for improperly using her position to try to influence the court on a pending matter.

An attorney regularly represents a certain client. When the client planned to leave on a world tour, she delivered to the attorney sufficient money to pay her property taxes when they became due. The attorney placed the money in his clients' trust account. When the tax payment date arrived, the attorney was in need of a temporary loan to close the purchase of a new personal residence. Because the penalty for late payment of taxes was only 2 percent while the rate for a personal loan was 6 percent, the attorney withdrew the client's funds from the clients' trust account to cover his personal check for the closing. The attorney was confident that the client would not object. Ten days later, after the receipt of a large fee previously earned, the attorney paid the client's property taxes and the 2 percent penalty, fully satisfying the client's tax obligation. After the client returned, the attorney told her what he had done, and the client approved the attorney's conduct. Is the attorney subject to discipline? A) Yes, because the attorney failed to pay the client the 10 days of interest at the fair market rate. B) Yes, because the attorney used the client's funds for a personal purpose. C) No, because the client was not harmed and the attorney reasonably believed at the time he withdrew the money that the client would not object. DNo, because when the attorney told the client what he had done, the client approved his conduct.

B) Yes, because the attorney used the client's funds for a personal purpose. Answer choice B is correct. Client funds must be held for safekeeping in an account that is separate from the lawyer's funds and may be used only in accordance with the client's instructions, not for the lawyer's personal benefit. Further, the attorney was required to use the funds for the client's designated purpose, that is, for payment of the taxes when they became due, and for no other purpose. Answer choice A is incorrect because even if the lawyer had paid the higher rate of interest to the client, the lawyer's personal use of the client's funds and his failure to carry out the client's instructions would have been improper. Answer choice C is incorrect because whether or not the client might be expected to acquiesce, a lawyer may not violate the client's instructions about how to use funds given to the lawyer for safekeeping, nor may the lawyer use those funds for personal purposes. Answer choice D is incorrect because a client's later approval of the action does not excuse a lawyer's unethical use of client funds for personal purposes.

Although licensed to practice law, an attorney does not practice law but works as an investment broker. The attorney could have elected inactive status as a member of the bar, but chose not to do so. Recently, in connection with a sale of worthless securities, the attorney made materially false representations to an investment customer. The customer sued the attorney for civil fraud, and a jury returned a verdict in the customer's favor. The attorney did not appeal. Is the attorney subject to discipline? A) Yes, because the attorney was pursuing a non-legal occupation while an active member of the bar. B) Yes, because the attorney's conduct was fraudulent. C) No, because the attorney was not convicted of a crime. D) No, unless the standard of proof in the state is the same in lawyer disciplinary cases and civil cases.

B) Yes, because the attorney's conduct was fraudulent. Answer choice B is correct. Under MRPC 8.4(c), it is professional misconduct for a lawyer to engage in conduct involving dishonesty, fraud, deceit, or misrepresentation. Here, the attorney's conduct was fraudulent, and she is thus subject to discipline. Answer choice A is not correct, as a lawyer is subject to discipline for any fraudulent conduct, even if not committed in the lawyer's role as a lawyer. Answer choice C is not correct, as the fraudulent conduct need not be criminal in nature to subject a lawyer to discipline under MRPC 8.4(c). Answer choice D is not correct, as the standard of proof need not be the same in lawyer disciplinary cases and civil cases.

A client retained an attorney to recover for a personal injury. In the retainer agreement signed by the client and the attorney, the client agreed to cooperate fully and pay the attorney a contingent fee computed as a percentage of the amount of recovery after expenses: 25 percent if settled before trial, 30 percent if settled before verdict, 35 percent after verdict, and 40 percent after appeal. The attorney's representation of the client in the matter extended over a three-year period during which the attorney advanced a large amount for litigation expenses. After trial, the client obtained a jury verdict for an amount larger than either the attorney or the client had anticipated. However, the defendant filed an appeal based on questions of evidence and the measure of damages. Meanwhile, the defendant made an offer of settlement for approximately the amount the attorney had originally projected as reasonable to expect. The client, who was hard pressed financially, directed the attorney to accept the offer and settle. The attorney refused, because she was confident that there was no reversible error in the trial and that the appeal was without merit. The attorney reasonably believed that the appeal was filed solely to gain negotiating advantage in settlement negotiations. Is the attorney subject to discipline? A) Yes, because the attorney's percentage under the fee contract increased after appeal. B) Yes, because the client directed the attorney to accept the settlement offer. C) No, because the decision whether to settle or defend an appeal is a tactical matter for the attorney to determine. D) No, because evaluation of the merits of an appeal requires the exercise of independent professional judgment.

B) Yes, because the client directed the attorney to accept the settlement offer. Answer choice B is correct. Under MRPC 1.2(a), a lawyer must abide by a client's decision as to whether to settle a matter. Answer choice A is incorrect. Although the percentage increase gives the attorney an incentive to continue through the appeal, this incentive does not create an impermissible conflict of interest. Answer choice C is incorrect. Under MRPC 1.2(a), a lawyer must abide by a client's decision whether to settle a matter, regardless of the procedural status. Answer choice D is incorrect. Although it is true that evaluating the merits of an appeal requires the exercise of independent professional judgment, MRPC 1.2(a) confers upon the client the ultimate authority to determine the objectives of the representation and to make certain decisions, including a decision as to whether to settle a matter.

A recently graduated attorney began a plaintiffs' personal injury practice, but was having a difficult time attracting clients. The attorney hired an advertising agency to prepare a television commercial in which the attorney appeared to be arguing a case before a jury. In the commercial, the jury brought back a large award for the attorney's client. The voice-over stated that results would vary depending upon particular legal and factual circumstances. The attorney's only experience at the time the commercial was filmed was in moot court. As a result of airing the commercial, the attorney received several significant cases. Is the attorney subject to discipline? A) Yes, because the commercial created an unjustified expectation about the results that could be achieved in court. B) Yes, because the commercial implied that the attorney had successfully argued a case to a jury. C) No, because commercial speech is protected under the First Amendment. D) No, because the commercial contained an express disclaimer about the results a client could expect.

B) Yes, because the commercial implied that the attorney had successfully argued a case to a jury. Answer choice B is correct. MRPC 7.1 prohibits false or misleading statements in lawyer advertising. Although the advertisement does not expressly state that the attorney has successfully tried a case, the communication would lead a reasonable person to believe that he has. As a result, the communication is misleading and is prohibited by MRPC 7.1. Answer choice A is incorrect. An advertisement may truthfully report a lawyer's achievement on behalf of a former client unless it would lead a reasonable person to form an unjustified expectation that the same results could be obtained for other clients. The use of a prominent disclaimer, such as the voice-over in the ad, avoids such an expectation. But, the issue here is that the communication was misleading. Answer choice C is incorrect because, although it is true that commercial speech is protected under the First Amendment, the First Amendment does not prohibit states from banning false or misleading communications in advertisements by professionals such as lawyers. Answer choice D is incorrect; the voice-over disclaimer avoids unjustified expectations regarding the results a client could expect, but it does not avoid the misleading implication that the attorney had successfully argued a case to a jury.

An attorney has been hired to represent a client in a civil commitment proceeding initiated by the state. The client is now undergoing psychiatric evaluation to determine whether civil commitment should be ordered. The client told the attorney that she intends to commit suicide as soon as the tests are completed, and the attorney believes that the client will carry out this threat. Suicide and attempted suicide are crimes in the state. Is it proper for the attorney to disclose the client's intentions to the authorities? A) Yes, because the information concerns a future crime and is not protected by the attorney-client evidentiary privilege. B) Yes, because the information concerns a future crime that is likely to result in the client's imminent death. C) No, unless the attorney knows that the client has attempted suicide in the past. D) No, because disclosure would aid the state in its civil commitment case against the client.

B) Yes, because the information concerns a future crime that is likely to result in the client's imminent death. Answer choice B is correct. Under MRPC 1.6(b), a lawyer may reveal otherwise protected information relating to the representation of a client to the extent the lawyer reasonably believes necessary to prevent reasonably certain death or substantial bodily harm. Here, the attorney reasonably believes that disclosing the client's intention to commit suicide will save the client's life. Answer choice A is not correct. The attorney-client privilege, which covers all confidential communications between a client and his lawyer, only applies to protect information in the context of evidence or testimony by the lawyer and is not relevant to the question of whether the lawyer has violated the ethical duty of confidentiality under MRPC 1.6, which protects all information learned in the course of representing a client from disclosure. Answer choice C is not correct, as MRPC 1.6(b) allows the information to be revealed to the extent the lawyer reasonably believes necessary to prevent reasonably certain death or substantial bodily harm. Answer choice D is not correct, as MRPC 1.6(b) allows disclosure under these circumstances, even if it would aid the state in its civil commitment case against the client.

An attorney has been retained to defend an adult charged with a sex offense involving a minor. The attorney believes that, in order to win the case, she must keep parents of minor children off the jury. The attorney instructed her investigator as follows: "Visit the neighborhood of those prospective jurors on the panel with minor children. Ask the neighbors if they know of any kind of unusual sex activity of the prospective juror or any member of the family. This talk will get back to the prospective jurors, and they will think of excuses not to serve. But don't under any circumstances talk directly with any prospective juror or member of the family." Is the attorney subject to discipline for so instructing her investigator? A) Yes, unless the prospective jurors investigated are, in fact, selected to serve on the jury in the case. B) Yes, because the investigation is intended to harass prospective jurors and members of their families. C) No, if the matters inquired into might be relevant to a prospective juror's qualifications to serve in the case. D) No, because no prospective juror was directly contacted.

B) Yes, because the investigation is intended to harass prospective jurors and members of their families. Answer choice B is correct. Under MRPC 3.5(a), a lawyer shall not seek to influence a prospective juror by means prohibited by law. Here, the attorney has instructed her investigator to engage in conduct that would constitute intimidation and harassment of prospective jurors, which would subject the attorney to discipline for violating the Model Rules. Answer choice A is not correct, as the Model Rules apply to conduct directed at both jurors and prospective jurors. Answer choice C is not correct, as the facts demonstrate that the conduct here is intended to harass prospective jurors and convince them not to serve. Answer choice D is not correct, as the Model Rules prohibit efforts to influence a prospective juror whether or not the prospective juror is directly contacted.

A state probate court judge has been asked to serve on the board of directors of a public interest group seeking to improve the law primarily through litigation exclusively in federal courts. The judge is thoroughly familiar both with the group's objectives and with its means of achieving them. The judge has always considered these objectives and methods to be proper in all respects. The judge's service as director would not interfere with performance of the judge's judicial duties. Is it proper for the judge to accept a position on the board of directors of the group? A) Yes, because the judge has always considered the organization's objectives and means of achieving them to be proper in all respects. B) Yes, because the position would not interfere with performance of the judge's judicial duties. C) No, because the organization primarily advances its views through litigation. D) No, because judges may serve as directors of only educational or religious organizations.

B) Yes, because the position would not interfere with performance of the judge's judicial duties. Answer choice B is correct. A judge may serve on the board of an organization concerned with improving the law, as here, where the judge's service will not interfere with the performance of judicial duties and the organization will not engage in litigation before the judge or the judge's court. Answer choice A is incorrect. A judge may serve on the board of an organization concerned with improving the law only if the judge's service will not interfere with the performance of judicial duties and the organization will not engage in litigation before the judge or the judge's court. It is not sufficient that the judge considers the organization's objectives and means to be proper. Answer choice C is incorrect. A judge may serve on the board of an organization concerned with improving the law, even if it engages in litigation, if the litigation is unlikely to come before the judge or the judge's court. That is true here, where the judge is a state judge and the organization litigates exclusively in federal court. Answer choice D is incorrect because, while a judge may serve on the board of an organization involved in educational, religious, or charitable activities, a judge may also service on the board of an organization concerned with improving the law.

A prosecutor was assigned to try a criminal case against a defendant, who was charged with robbery of a convenience store. The defendant denied any involvement, contending he was home watching television with his mother on the night in question. At the trial, a customer at the convenience store testified that he had identified the defendant in a police lineup and provided other testimony connecting the defendant to the crime. In addition, the prosecutor entered into evidence a poor-quality videotape of the robbery as recorded by the store surveillance camera. The jury convicted the defendant of the crime charged. Unknown to the defendant's court-appointed lawyer, the customer had first identified another person in the police lineup and selected the defendant only after encouragement by the detective. The prosecutor was aware of these facts but did not notify the defendant's counsel, who made no pre-trial discovery request to obtain this information. Is the prosecutor subject to discipline? A) Yes, unless the jury could make its own identification of the defendant from the videotape. B) Yes, because this information tended to negate the defendant's guilt. C) No, because the defendant's counsel made no pre-trial discovery request to obtain this information. D) No, unless it is likely that the jury would have acquitted the defendant had it known that the customer first identified someone else.

B) Yes, because this information tended to negate the defendant's guilt. Answer choice B is correct. Under MRPC 3.8(d), a prosecutor must make timely disclosure to the defense of all evidence or information known to the prosecutor that tends to negate the guilt of the accused or mitigates the offense. Here, the information tended to negate the defendant's guilt and should have been disclosed. Answer choice A is not correct. Regardless of whether the jury could make its own identification of the defendant from the videotape, the prosecutor was required to timely disclose the facts and will be subject to discipline for not doing so. Answer choice C is not correct, as it is irrelevant whether the defendant's counsel had made a pre-trial request for the purposes of discipline under the Model Rules. MRPC 3.8(d) required the disclosure and controls the issue. Answer choice D is not correct. Regardless of the potential outcome of the case, the prosecutor was obligated under MRPC 3.8(d) to disclose the information.

An attorney represented both the owner of an art gallery and a publisher. The gallery owner and the publisher each made a practice of paying the attorney's fees in cash. The attorney received separate cash payments from the gallery owner and the publisher on the same day. Each payment consisted of ten $100 bills, which the attorney immediately deposited in her bank account. One week later, the attorney was contacted by United States Treasury agents, who informed her that four of the bills had been identified as counterfeit. The agents did not accuse the attorney of knowingly passing the counterfeit money but asked her who had given her the bills. The attorney was subpoenaed to testify before a grand jury and was asked who could have given her the counterfeit money. Is it proper for the attorney to provide the grand jury with the names of the gallery owner and the publisher? A) Yes, because negotiation of a counterfeit bill is a criminal act. B) Yes, because under the circumstances neither client's identity is privileged. C) No, because counterfeiting is not a crime that involves an imminent threat of death or serious bodily harm. D) No, because the attorney has no way of knowing which of the two clients gave her the counterfeit bills.

B) Yes, because under the circumstances neither client's identity is privileged. Answer choice B is correct. Under MRPC 1.6(B)(6), the attorney may provide client confidences in response to a grand jury subpoena unless the information may be protected by the attorney-client privilege, in which case the attorney must assert the privilege unless the client waives it. In this case, the clients' names are confidential under MRPC 1.6 but are not protected by the attorney-client privilege. The privilege covers only confidential communications between a client and a lawyer. It does not cover clients' identities unless disclosing their identities will implicitly reveal their confidential communications, which is not the case here. Answer choice A is incorrect. It is irrelevant that negotiating a counterfeit bill is a crime. If the attorney's information in this case was privileged, the attorney could not disclose it under the crime-fraud exception because the attorney does not know which client paid with counterfeit bills or whether the client who did so was aware that the bills were counterfeit. Answer choice C is incorrect. The fact that the information is confidential under MRPC 1.6(a) and not subject to the exception under MRPC 1.6(b)(1) that allows the disclosure of confidences to prevent reasonably certain death or substantial bodily harm is irrelevant to these facts. Answer choice D is incorrect. Under MRPC 1.6(B)(6), the attorney may provide client confidences in response to a grand jury subpoena unless the information may be protected by the attorney-client privilege, in which case the attorney must assert the privilege unless the client waives it. In this case, the clients' names are not protected by the attorney-client privilege, so the attorney may reveal their names. If the attorney's information in this case were privileged (which it isn't), then the attorney could not disclose it under the crime-fraud exception because the attorney does not know which client paid with counterfeit bills or whether the client who did so was aware that the bills were counterfeit. But because the information is not privileged, the crime-fraud exception is inapplicable.

An attorney was retained by a client to represent the client in defense of an action brought against the client by the plaintiff. In order to obtain ample time for settlement negotiations, the attorney immediately requested and obtained from opposing counsel a stipulation extending the client's time to answer the complaint until ten days after receipt of written demand from the opposing counsel. Four months later, no settlement had been reached, and on May 1, the opposing counsel wrote the attorney demanding that an answer be filed within ten days. When no answer was filed by May 15, the opposing counsel had a default judgment entered in favor of the plaintiff. The attorney was away on a two-month vacation when the opposing counsel's letter was received in her office. When the attorney returned on June 15, she promptly moved to have the default set aside, and her motion was granted. Is the attorney subject to discipline? A) Yes, unless she makes restitution to the client for any loss sustained by the client. B) Yes, if she did not make provision for the handling of her pending cases while she was away. C) No, because the default judgment was set aside. D) No, unless she knew that the opposing counsel had demanded that an answer be filed within ten days.

B) Yes, if she did not make provision for the handling of her pending cases while she was away. Answer choice B is correct. The attorney is required to control her workload to ensure that all matters are handled competently. MRPC 1.3, cmt 2. This would include ensuring that pending cases are addressed properly while she was away on vacation. Answer choice A is not correct, because even if the attorney makes restitution for any loss sustained by the client, it would not excuse a violation of the Model Rules for failure to control her workload. Answer choice C is not correct, because the fact that the default judgment was ultimately set aside does not excuse the attorney from complying with her responsibility under the Model Rules to control her workload to ensure that all matters are handled competently. Answer choice D is not correct, because even if the attorney did not specifically know of the opposing counsel's demand, the attorney knew of the agreement that had been made with the opposing counsel and was responsible to ensure that her client's interests were protected while she was away on vacation.

A state does not require lawyers to participate in continuing legal education courses. Three lawyers, recently admitted to practice, formed a law partnership in that state. As they considered what expenses the partnership would pay on behalf of each lawyer, a majority decided that the firm would not pay for continuing legal education courses since they were not required by their state. One of the lawyers, who wanted reimbursement for continuing legal education courses, angrily said, "Fine. I won't attend any continuing legal education courses." Is it proper for the lawyer to refuse to attend any continuing legal education courses? A) Yes, unless the state offers free continuing legal education courses. B) Yes, if the lawyer independently undertakes continuing study and education in the law. C) No, because the lawyer cannot maintain competence without attending continuing legal education courses. D) No, unless the lawyer obtains malpractice insurance.

B) Yes, if the lawyer independently undertakes continuing study and education in the law. Answer choice B is correct. A lawyer has a duty of competence. MRPC 1.1. To maintain the requisite knowledge and skill, a lawyer should make efforts to keep abreast of changes in the law and its practice, engage in continuing study and education, and comply with all continuing legal education requirements to which the lawyer is subject. MRPC 1.1, cmt 6. Here, the state has no continuing legal education requirement. The lawyer can properly maintain her knowledge and skills through her own independent study. Answer choice A is incorrect because there is no requirement under the state's law or the Model Rules for an attorney to attend even free continuing legal education courses. Answer choice C is incorrect because the lawyer can maintain her competence through self-study, without attending continuing legal education courses. Answer choice D is incorrect because, under the facts here, it would still be proper for the lawyer to refuse to attend any continuing legal education courses even if she obtained malpractice insurance. The purchase of malpractice insurance does not otherwise relieve an attorney of the duty of competence.

FILL IN THE BLANKS. A lawyer must generally report misconduct by another lawyer whenever the conduct is a ______________ and the reporting lawyer has __________________. A) minor violation, actual knowledge of the misconduct B) substantial violation, actual knowledge of the misconduct C) minor violation, a reasonable belief that the misconduct occurred D) substantial violation, a reasonable belief that the misconduct occurred

B) substantial violation, actual knowledge of the misconduct

Which of the following statements regarding a lawyer's representation of a client is TRUE? A) A client under a disability must always have a guardian. B) The scope of representation can always be limited by the lawyer as long as the limitation is conveyed to the client. C) A client and a lawyer may agree to limit the scope of the lawyer's representation. D) A lawyer cannot discuss any aspect of a client's intention to commit a future crime.

C) A client and a lawyer may agree to limit the scope of the lawyer's representation.

Which of the following statements regarding a lawyer's liability is FALSE? A) A lawyer can be liable to a third-party beneficiary. B) A lawyer can be liable to a third party for unauthorized acts. C) A lawyer can be liable for malicious prosecution even when the claim is supported by probable cause. D) A lawyer can limit malpractice liability through settlement if the client is represented by independent counsel.

C) A lawyer can be liable for malicious prosecution even when the claim is supported by probable cause.

Which of the following statements regarding a prosecutor's duties is FALSE? A) A prosecutor may not prosecute a charge that the prosecutor knows is not supported by probable cause. B) A prosecutor must not subpoena a lawyer for the purpose of circumventing the attorney-client privilege. C) A prosecutor need not remedy a conviction when presented with new exculpatory evidence that is clear and convincing. D) A prosecutor must disclose exculpatory information during trial.

C) A prosecutor need not remedy a conviction when presented with new exculpatory evidence that is clear and convincing.

Which of the following may a court order an attorney to disclose? A) Any information protected by the duty of confidentiality B) Any information subject to the attorney-client privilege C) Any information protected by the duty of confidentiality that is not subject to the attorney-client privilege D) Any information subject to the attorney-client privilege that is not subject to the duty of confidentiality

C) Any information protected by the duty of confidentiality that is not subject to the attorney-client privilege

Which of the following is a disciplinary sanction that a lawyer may face for conduct that violates the rules of professional conduct? A) Civil liability B) Criminal liability C) Disbarment D) Disqualification as a counsel in a civil or criminal matter

C) Disbarment

An attorney, who was recently admitted to the bar, has been appointed by the court as counsel for an indigent defendant charged with a felony. After consulting with the defendant and attempting for two days to prepare the case for trial, the attorney became convinced that he lacked the knowledge and experience to represent the defendant effectively. Which of the following would NOT be proper for the attorney? A) Request permission of the court to withdraw from representing the defendant because the attorney knows that he is not competent to handle the case. B) Request the court to appoint experienced co-counsel and grant a continuance to enable co-counsel to prepare the case. C) Explain the circumstances to the defendant and, if the defendant consents, proceed to represent him alone to the best of his ability. D) None of the above would be proper.

C) Explain the circumstances to the defendant and, if the defendant consents, proceed to represent him alone to the best of his ability. Answer choice C is correct because it is the only improper course of action. A lawyer's duty of competence may not be waived by the client. Answer choice A is incorrect, because, as an appointed lawyer who knows that he is not competent to represent a client, it is proper for the attorney to attempt to obtain the court's permission to withdraw from representation. Answer choice B is incorrect because it is proper for the attorney to seek to provide competent representation through the association with a lawyer of established competence in the field. MRPC 1.1, cmt 2. Answer choice D is incorrect because options A and B are proper courses of action for the attorney to take.

A new client has asked his attorney to write a letter recommending his nephew for admission to the bar. The client has told his attorney that he has no direct contact with his nephew, but that his sister (the nephew's mother) has assured him that the nephew is industrious and honest. Which of the following would be proper for the attorney? A) Write the letter on the basis of the client's assurance. B) Write the letter on the basis of the client's assurance if the attorney has no unfavorable information about the nephew. C) Make an independent investigation and write the letter only if the attorney is thereafter satisfied that the nephew is qualified. D) All of the above would be proper.

C) Make an independent investigation and write the letter only if the attorney is thereafter satisfied that the nephew is qualified. Answer choice C is correct. Answer choices A and B are incorrect, as it would be improper for the attorney to recommend an applicant to the bar without independently assuring herself that the applicant is fit to practice law. The attorney may not properly rely on the client's assurance or upon her own lack of unfavorable information about the nephew. Answer choice C is correct, as it provides for an independent investigation without reliance on the client's assurances.

Attorneys Alpha and Beta had been political opponents. Alpha was elected to the state legislature after a bitter race in which Beta had managed the campaign of Alpha's opponent. Alpha had publicly blamed Beta at that time for what Alpha reasonably believed were illegal and unethical campaign practices and later had publicly objected to Beta's appointment as a judge. Alpha represented a client in a widely publicized case tried in Judge Beta's court. At the conclusion of the trial, Beta ruled against Alpha's client. Alpha then held a press conference and said, "All that you reporters have to do is check your files and you will know what I think about Judge Beta's character and fitness." Is Alpha subject to discipline for making this statement? A) Yes, if Alpha's statement might lessen confidence in the legal system. B) Yes, because Alpha's past accusations were unrelated to Beta's legal knowledge. C) No, because Alpha reasonably believed that the statements about Beta were true. D) No, if Beta had equal access to the press.

C) No, because Alpha reasonably believed that the statements about Beta were true. Answer choice C is correct. Under MRPC 8.2(a), a lawyer must not make a statement that the lawyer knows to be false or with reckless disregard as to its truth or falsity concerning the qualifications or integrity of a judge. Here, Alpha told the reporters to check their files, referring to his prior statements about what he reasonably believed were illegal and unethical campaign practices by Beta. As he reasonably believed that these statements were true, Alpha is not subject to discipline. Answer choice A is not correct, as Alpha's reasonable belief in the truth of his statements renders them permissible under the Model Rules. Answer choice B is not correct, as it is irrelevant that Alpha's past accusations were unrelated to Beta's legal knowledge. To be subject to discipline for a statement about a judge's integrity, a lawyer must know the statement to be false or make the statement with reckless disregard for its truth. Answer choice D is not correct, as it is not necessary for Beta to have had equal access to the press for Alpha not to be subject to discipline.

An attorney experienced several instances when clients failed to pay their fees in a timely manner, but it was too late in the representation to withdraw without prejudicing the clients. To avoid a recurrence of this situation, the attorney has drafted a stipulation of consent to withdraw if fees are not paid according to the fee agreement. She proposes to have all clients sign the stipulation at the outset of the representation. Is it proper for the attorney to use the stipulation to withdraw from representation whenever a client fails to pay fees? A) Yes, because a lawyer may withdraw when the financial burden of continuing the representation would be substantially greater than the parties anticipated at the time of the fee agreement. B) Yes, because the clients consented to the withdrawal in the stipulation. C) No, because a client's failure to pay fees when due may be insufficient in itself to justify withdrawal. D) No, unless clients are provided an opportunity to seek independent legal advice before signing the stipulation.

C) No, because a client's failure to pay fees when due may be insufficient in itself to justify withdrawal. Answer choice C is correct. While in some cases failure to pay fees may be sufficient to justify a withdrawal under MRPC 1.16, in other cases it will not be sufficient. Under MRPC 1.16(b)(5), a lawyer may withdraw if the client refuses to fulfill an obligation regarding the representation, which can include nonpayment of fees. In such cases, however, the client must first be given reasonable warning and an opportunity to satisfy the obligation. The use of an advance-consent stipulation allowing withdrawal in all circumstances in which fees are not paid immediately would not allow for a proper warning or provide an opportunity for the client to satisfy her obligation as required by the rule. Answer choice A is not correct. While a lawyer may seek to withdraw when the representation will result in an unreasonable financial burden on the lawyer, this stipulation attempts to allow the attorney to withdraw whenever fees are not paid according to the fee agreement. A court may determine that one instance of nonpayment does not necessarily constitute an unreasonable burden. Answer choice B is not correct, as even if a client consents to a withdrawal, a court may nevertheless order a lawyer to continue representation. Answer choice D is not correct, as the requirement for seeking independent legal advice applies to the settlement of malpractice claims, not in this context.

In an attorney's closing statement to the court in a bench trial, the attorney said, "Your honor, I drive on the street in question every day and I know that a driver cannot see cars backing out of driveways as the one did in this case. I believe that my client was not negligent, and I ask you to so find." Was the attorney's closing argument proper? A) Yes, if the attorney was speaking truthfully and not trying to deceive the court. B) Yes, because the rules of evidence are very liberal when the trial is before a judge without a jury. C) No, because the attorney asserted his personal knowledge of facts in issue. D) No, if there is no other evidence in the record about the facts asserted by the attorney.

C) No, because the attorney asserted his personal knowledge of facts in issue. Answer choice C is correct. Under MRPC 3.4(e), a lawyer is prohibited from asserting personal knowledge of facts in issue, except when testifying as a witness. Thus, the attorney's closing argument, which asserted personal knowledge of facts at issue, was improper. Answer choice A is not correct, as regardless of the truth of what the attorney said and his motive, MRPC 3.4(e) controls and prohibits such assertions. Answer choice B is not correct, as MRPC 3.4(e) is what controls the ethics question (not the rules of evidence) and prohibits the assertion of personal knowledge of facts in issue under the circumstances. Answer choice D is not correct, as regardless of whether there is evidence that would support the attorney's assertion of facts, MRPC 3.4(e) prohibits the attorney from asserting personal knowledge of facts in issue.

An attorney was engaged under a general retainer agreement to represent a corporation involved in the uranium industry. Under the agreement, the attorney handled all of the corporation's legal work, which typically involved regulatory issues and litigation. The corporation told the attorney that a congressional committee was holding hearings concerning the extent of regulation in the copper industry. Because the corporation was considering buying a copper mine during the next fiscal year, the corporation wanted the attorney to testify that the industry was overregulated. The attorney subsequently testified before the relevant congressional committee. The attorney registered his appearance under his own name and did not disclose that he was appearing on behalf of a client. Afterward, the attorney billed the corporation for fees and expenses related to his testimony. Was the attorney's conduct proper? A) Yes, because the duty of confidentiality prevented the attorney from disclosing the identity of his client. B) Yes, because the attorney-client evidentiary privilege prevented disclosure of the identity of his client in this context. C) No, because the attorney failed to disclose that he was appearing and testifying in a representative capacity. D) No, because the attorney accepted compensation in return for his testimony.

C) No, because the attorney failed to disclose that he was appearing and testifying in a representative capacity. Answer choice C is correct. Under MRPC 3.9, a lawyer who represents a client before a legislative body must disclose that the appearance is in a representative capacity. As the attorney did not do that here, his conduct was improper. Answer choice A is not correct, as MRPC 3.9 requires such disclosure. Answer choice B is not correct, as the attorney-client evidentiary privilege is irrelevant. There is no indication in the facts that the client wanted the attorney not to reveal the representation. Even if that was the case, a lawyer may not mislead a legislative body regarding the fact that the lawyer appears in a representative capacity. To comply with the rule, the lawyer could simply disclose that he is appearing on behalf of another, whose identity is privileged. Answer choice D is not correct, as a lawyer may accept compensation for assisting a client in trying to influence legislative action. The lawyer must disclose the fact that he is appearing in a representative capacity, however, to be in compliance with the Model Rules.

An attorney is employed by a client who is a fugitive from justice under indictment for armed robbery. The attorney, after thorough legal research and investigation of the facts furnished by the client, reasonably believes the indictment is fatally defective and should be dismissed as a matter of law. The attorney advised the client of his opinion and urged the client to surrender. The client told the attorney that she would not surrender. The attorney informed the district attorney that he represented the client and that he had counseled her to surrender but that she refused to follow his advice. The attorney has not advised his client on how to avoid arrest and prosecution and does not know where she is hiding. Is the attorney subject to discipline if he continues to represent the client? A) Yes, because the client is engaging in continuing illegal conduct. B) Yes, because the client refused to accept the attorney's advice and surrender. C) No, because the attorney is not counseling the client to avoid arrest and prosecution. D) No, because the attorney reasonably believes the indictment is defective.

C) No, because the attorney is not counseling the client to avoid arrest and prosecution. Answer choice C is correct. Under MRPC 1.2(d), a lawyer may not knowingly counsel or assist a client in illegal conduct, but in this case, the lawyer has not done so. Answer choice A is incorrect because, although it is true that the client is engaging in continuing illegal conduct, the client is nevertheless entitled to the advice of a lawyer. So long as the lawyer does not knowingly counsel or assist the client in the illegal conduct (MRPC 1.2(d)), the lawyer may continue to represent the client. Answer choice B is incorrect because, although MRPC 1.16(b)(4) grants a lawyer permission to withdraw if the client insists on action that the lawyer believes is repugnant or with which the lawyer fundamentally disagrees, the lawyer is not required to do so. Answer choice D is incorrect because even if the indictment is defective, the client's conduct would still be illegal. Nevertheless, the lawyer may continue to represent the client so long as the lawyer does not knowingly counsel or assist the client in the illegal conduct.

An attorney who had represented a client for many years prepared the client's will and acted as one of the two subscribing witnesses to its execution. The will gave 10 percent of the client's estate to her housekeeper, 10 percent to her son and sole heir, and the residue to charity. Upon the client's death one year later, the executor named in the will asked the attorney to represent him in probating the will and administering the estate. At that time, the executor informed the attorney that the son had notified him that he would contest the probate of the will on the grounds that the client lacked the required mental capacity at the time the will was executed. The attorney believes that the client was fully competent at all times and will so testify, if called as a witness. The other subscribing witness to the client's will pre-deceased the client. Is it proper for the attorney to represent the executor in the probate of the will? A) Yes, because the attorney is the sole surviving witness to the execution of the will. B) Yes, because the attorney's testimony will support the validity of the will. C) No, because the attorney will be called to testify on a contested issue of fact. D) No, because the attorney will be representing an interest adverse to the client's heir at law.

C) No, because the attorney will be called to testify on a contested issue of fact. Answer choice C is correct. MRPC 3.7(a) states that a lawyer may not serve as an advocate at a trial in which the lawyer is likely to be a necessary witness relating to a contested issue. Answer choice A is incorrect because the fact that the attorney is the sole surviving witness to the will's execution means that the attorney is likely to be a necessary witness regarding the client's mental capacity when she executed the will. The attorney, therefore, may not also represent the executor in connection with the will contest, per MRPC 3.7(a). Answer choice B is incorrect because, under MRPC 3.7(a), a lawyer may not represent a party at a trial in which the lawyer is likely to be a necessary witness relating to a contested issue, even if the testimony supports the party's position. Answer choice D is incorrect, as the fact that the attorney would be representing a party (the executor) who is adverse to the deceased client's heir is irrelevant. But for the fact that the attorney is a likely witness, the representation would be proper.

An attorney and a client entered into a written retainer and hourly fee agreement that required the client to pay $5,000 in advance of any services rendered by the attorney and that required the attorney to return any portion of the $5,000 that was not earned. The agreement further provided that the attorney would render monthly statements and withdraw her fees as billed. The agreement was silent as to whether the $5,000 advance was to be deposited in the attorney's Clients' Trust Account or in a general account. The attorney deposited the entire fund in her Clients' Trust Account, which also contained the funds of other persons, which had been entrusted to the attorney. Thereafter, the attorney rendered monthly progress reports and statements for services to the client after services were rendered, showing the balance of the client's fee advance. However, the attorney did not withdraw any of the $5,000 advance until one year later, when the matter was concluded to the client's complete satisfaction. At that time, the attorney had billed the client reasonable legal fees of $4,500. The attorney wrote two checks on her Clients' Trust Account: one to herself for $4,500, which she deposited in her general office account, and one for $500 to the client. Was the attorney's conduct proper? A) Yes, because the attorney deposited the funds in her Clients' Trust Account. B) Yes, because the attorney rendered periodic and accurate billings. C) No, because the attorney's failure to withdraw her fees as billed resulted in an impermissible commingling of her funds and the client's funds. D) No, because the attorney required an advanced payment against her fee.

C) No, because the attorney's failure to withdraw her fees as billed resulted in an impermissible commingling of her funds and the client's funds. Answer choice C is correct. Under the fee agreement, the attorney agreed to withdraw her fees as billed. The fact that she failed to withdraw her fees as billed over the year meant that she commingled her funds with the client's because all the money remained in a Clients' Trust Account. MRPC 1.15(a) prohibits commingling of a lawyer's personal funds with client funds. Answer choice A is not correct, as the attorney was required to withdraw her fees as billed. She could not leave them in the Clients' Trust Account. Answer choice B is not correct because the fact that the billing was accurate is not at issue. The attorney commingled personal and client funds by not withdrawing her fees as billed. Answer choice D is not correct, as advance payment of fees is not improper so long as the lawyer deposits the funds into a Clients' Trust Account to be withdrawn by the lawyer only as fees are earned or expenses incurred in the representation. MRPC 1.15(c).

An attorney represented a baker in a claim involving a breach of the baker's employment contract. The case was settled without suit being filed. The proceeds of the settlement were paid directly to the baker, who subsequently paid the attorney in full for the attorney's fee and expenses. Thereafter, the attorney did no other work for the baker. The baker is now being audited by the Internal Revenue Service (IRS). The IRS has asked the attorney for details of the settlement, including the amount claimed for each item of damage and the amounts paid for the items. The attorney reported the request to the baker, who told the attorney not to provide the information to the IRS. Is it proper for the attorney to furnish the information to the IRS? A) Yes, if the information does not involve the attorney's work product. B) Yes, because the attorney no longer represents the baker. C) No, because the baker told the attorney not to provide the information. D) No, unless the attorney believes the disclosure would be beneficial to the baker.

C) No, because the baker told the attorney not to provide the information. Answer choice C is correct. The ethical duty of confidentiality under MRPC 1.6 prohibits the attorney from furnishing the information in the absence of the baker's consent. While a lawyer may reveal confidential information concerning the representation of a client to the extent the lawyer reasonably believes it necessary to comply with other law or a court order, here there is no indication of any law or court order requiring such disclosure. Moreover, unless the client provides informed consent to do otherwise, the lawyer must assert on behalf of the client all non-frivolous claims that the order is not authorized by other law or that the information sought is protected against disclosure by lawyer-client privilege or other applicable law. Answer choice A is not correct, as all information from the representation of the baker, not merely attorney work product, must be kept confidential in the absence of the baker's consent, pursuant to MRPC 1.6. Answer choice B is not correct, as the ethical duty of confidentiality continues even after the representation ends. Answer choice D is not correct, because even if the attorney believes the disclosure would be beneficial to the baker, the attorney is not permitted to provide the information in the absence of the baker's consent, pursuant to MRPC 1.6.

An attorney represents a client in commercial litigation that is scheduled to go to trial in two months. Over the past several weeks, the client has disagreed with almost every tactical decision that the attorney has made. Frustrated, the attorney finally said to the client that if she didn't like the way he was handling the lawsuit, perhaps she should get another lawyer. The client was upset at the suggestion and accused the attorney of trying to get out of the case. Reasonably believing that he could no longer work effectively with the client, the attorney sought the client's permission to withdraw from the representation, and the client reluctantly agreed. After giving the client sufficient notice to obtain replacement counsel, the attorney requested the court's permission to withdraw from the litigation, but the court denied the request. May the attorney withdraw from the representation? A) Yes, because the client agreed, and the attorney gave the client sufficient notice to obtain replacement counsel. B) Yes, because the client had made it unreasonably difficult for the attorney to carry out the representation effectively. C) No, because the court denied the attorney's request to withdraw. D) No, because the attorney's withdrawal would cause material prejudice to the client, and the client's agreement was not voluntary.

C) No, because the court denied the attorney's request to withdraw. Answer choice C is correct. MRPC 1.16(c) states that the lawyer must continue the case if ordered to do so by the court. Answer choice A is incorrect because in all court proceedings, the judge must consent to the withdrawal and substitution of counsel. Sufficient notice and client consent are not enough for withdrawal if the court does not agree. MRPC 1.16(c) states that the lawyer must continue the case if ordered to do so by the court. Answer choice B is incorrect; the client's conduct is a reason for the lawyer to move for withdrawal, but the lawyer must get the court's permission before withdrawal would be proper. MRPC 1.16(c) states that the lawyer must continue the case if ordered to do so by the court. Answer choice D is incorrect because there is no evidence that the client would be prejudiced by the withdrawal. Withdrawal is improper because the court refused to grant permission to withdraw.

An attorney is representing the plaintiff in a personal injury case on a contingent fee basis. The client is without resources to pay for the expenses of the investigation and the medical examinations necessary to prepare for trial. The client asked the attorney to pay for these expenses. The attorney declined to advance the funds but offered to guarantee the client's promissory note to a local bank in order to secure the funds needed to cover those expenses. The client has agreed to reimburse the attorney in the event she incurs liability on the guaranty. Is the attorney subject to discipline if she guarantees the client's promissory note? A) Yes, because the attorney is lending her credit to the client. B) Yes, because the attorney is lending her credit to the client. C) No, because the funds will be used for trial preparation. D) No, because the attorney took the case on a contingent fee basis.

C) No, because the funds will be used for trial preparation. Answer choice C is correct. While financial assistance to a client is generally prohibited with respect to pending or planned litigation, a lawyer is permitted to advance litigation costs (including the expenses of medical examination and the costs of obtaining and presenting evidence) to the client under MRPC 1.8(e). The attorney will therefore not be subject to discipline for guaranteeing the client's promissory note. Answer choices A and B are not correct, as MRPC 1.8(e) allows a lawyer to advance litigation costs to the client. Answer choice D is not correct, as MRPC 1.8(e) allows the repayment of litigation costs to be contingent on the outcome of the matter.

An attorney served two four-year terms as the state's governor immediately prior to reopening his law office in the state. The attorney printed and mailed an announcement of his return to private practice to members of the bar, previous clients, and personal friends whom he had never represented. The printed announcement stated that the attorney had reopened his law office, gave his address and telephone number, and added that he had been the state's governor for the past eight years. Is the attorney subject to discipline for the announcement? A) Yes, because the mailing included persons who had not been his clients. B) Yes, because his service as governor is unrelated to his ability as a lawyer. C) No, because the information in the announcement was true. D) No, because all of the information was already in the public domain.

C) No, because the information in the announcement was true. Answer choice C is correct. The communication is an advertisement. MRPC 7.1, which covers lawyer advertising, forbids only false or misleading communications, and this communication was neither false nor misleading. Answer choice A is incorrect because although MRPC 7.3 prohibits lawyers from making in-person, live telephone, or real-time electronic contact to solicit for pecuniary gain, this communication does not fall within the banned behavior. It is an advertisement governed by MRPC 7.1 and 7.2. As a result, the communication is permissible so long as it does not contain any false or misleading statements, which this announcement did not. Answer choice B is incorrect, as MRPC 7.1 forbids only false or misleading communications in lawyer advertising. A truthful statement about the attorney's prior service as governor is not likely to create any unjustified expectations and is therefore not misleading. Answer choice D is also incorrect. Information in the public domain can still be either false or misleading. As such, communication of that information might be prohibited under MRPC 7.1, which forbids false or misleading communications in lawyer advertising.

A judge has recently resigned from the state trial court bench. When she was a judge, she supervised activity in cases pending before another judge while he was on vacation. In one pending case, she entered an administrative order changing the courtroom in which a particular case was to be tried. After trial and appeal, the case was remanded for a new trial. The plaintiff in that case decided to change lawyers and has asked the recently resigned judge to try the case. Will the judge be subject to discipline if she tries this case on behalf of the plaintiff? A) Yes, because the judge acted officially as a judge with respect to an aspect of the case. B) Yes, because the judge would try the case before a judge of the court on which she previously sat. C) No, because the judge did not act as a judge with respect to a substantial matter in or the merits of the case. D) No, because any information that the judge learned about the case while acting as a judge was a matter of public record.

C) No, because the judge did not act as a judge with respect to a substantial matter in or the merits of the case. Answer choice C is correct. Under MRPC 1.12(a), the former judge may represent the plaintiff because she did not participate "substantially" in the matter while she was an acting judge. She merely entered an incidental administrative order unrelated to the merits of the lawsuit. Answer choice A is incorrect because, although the judge acted officially in the lawsuit before resigning from the bench, she was not restricted by MRPC 1.12(a) because she did not participate "substantially" in the matter when she entered an administrative order changing the courtroom. Comment [1] to MRPC 1.12 provides that "the fact that a former judge exercised administrative responsibility in a court does not prevent the former judge from acting as a lawyer in a matter where the judge had previously exercised remote or incidental administrative responsibility that did not affect the merits." Answer choice B is incorrect because no rule prohibits a former judge from appearing on behalf of a party before a judge of a court on which she previously sat. Answer choice D is incorrect because the fact that information learned by the former judge in her administrative capacity is a matter of public record would not be enough in itself to allow the former judge to represent a party in a matter in which she had previously participated as a judge. If the judge's personal participation had been substantial, which it was not in this case, then MRPC 1.12(a) would forbid her later representation of the plaintiff.

A manufacturer sued a company for its breach of warranty regarding machine components it furnished. A judge, who presided at the nonjury trial, sent her law clerk to the manufacturer's plant to observe the machine that was malfunctioning due to the allegedly defective parts. The clerk returned and told the judge that the machine was indeed malfunctioning and that the engineer, an employee of the manufacturer, had explained to the clerk how the parts delivered by the company caused the malfunction. There was testimony at the trial that supported what the clerk learned on his visit. The judge rendered a judgment for the manufacturer. Was the judge's conduct proper? A) Yes, because the judge's judgment was supported by evidence at the trial. B) Yes, because the judge has the right to gather facts concerning the trial. C) No, because the judge has engaged in ex parte contacts that might influence the outcome of litigation. D) No, unless the engineer was a witness at the trial and subject to cross-examination by the company.

C) No, because the judge has engaged in ex parte contacts that might influence the outcome of litigation. Answer choice C is correct. A judge is required not to initiate, permit, or consider ex parte communications or other communications made to the judge outside the presence of the parties or their lawyers, with regard to a pending or impending matter. A judge is required to make reasonable efforts, including providing appropriate supervision, to ensure that this rule is not violated by court staff, court officials, and others subject to the judge's direction and control. CJC Rule 2.9(D). Here, the statement made to the clerk constituted an ex parte communication and was improper for the judge to consider. Answer choice A is not correct because regardless of whether the judgment was supported by evidence at trial, the communication and consideration of the communication was improper under the Code of Judicial Conduct. Answer choice B is not correct, as a judge is not permitted to gather facts through improper ex parte communication. Answer choice D is not correct, because even if the engineer was a witness subject to cross-examination by the company, the Code of Judicial Conduct only permits the judge to consider evidence from the engineer that is provided in court.

A law firm, after an exploration of its options, contracted with an independent company, which the firm thoroughly vetted, to store the firm's electronically maintained records, including client files, at a distant location. The company maintains reasonable procedures to prevent unauthorized access to the records. Despite those procedures, a hacker succeeded in breaching the company's security and gained access to the law firm's records. There is no evidence that a client of the law firm has been harmed by this breach. Are the managing partners of the law firm subject to discipline? A) Yes, because the managing partners are responsible for the conduct of non-lawyers who are retained by the law firm. B) Yes, because a lawyer owes a duty of confidentiality with regard to information relating to the representation of a client. C) No, because the law firm took reasonable measures to prevent disclosure of the firm's client records. D) No, because there is no evidence that a client of the law firm has been harmed by the breach in the company's security.

C) No, because the law firm took reasonable measures to prevent disclosure of the firm's client records. Answer choice C is correct. Although a lawyer does generally owe a duty of confidentiality with regard to information relating to the representation of a client, a lawyer is not an insurer of the nondisclosure of such information. Instead, a lawyer must make reasonable efforts to prevent the inadvertent or unauthorized disclosure of, or unauthorized access to, information relating to the representation of a client. Here, since the law firm made such efforts, the managing partners of the firm are not subject to discipline. Answer choice A is incorrect. A lawyer who has managerial authority over the professional work of a law firm, such as a partner,, must make reasonable efforts to ensure the firm has measures in place that give reasonable assurance that the conduct of all lawyers in the firm, as well as that of non-lawyers employed by, retained by, or associated with the firm, conforms to the Model Rules. Here, the law firm conducted a thorough exploration of its options and selected an independent company that maintained reasonable security procedures to prevent unauthorized access to the firm's client records. Consequently, the managing partners have not breached their duty. Answer choice B is incorrect because, although a lawyer does generally owe a duty of confidentiality with regard to information relating to the representation of a client, the law firm fulfilled its duty by making reasonable efforts to prevent inadvertent or unauthorized disclosure of client information. Answer choice D is incorrect because, unlike a liability action by a client against a law firm based on the negligent disclosure of the client's information, a lawyer's ethical duty of confidentiality can be breached, even though the client does not suffer harm as a consequence. ?

An attorney is skilled in trying personal injury cases. The attorney accepted the representation of a plaintiff in a personal injury case on a contingent fee basis. While preparing the case for trial, the attorney realized that the direct examination and cross-examination of the medical experts would involve medical issues with which the attorney was not familiar and, as a consequence, the attorney might not be able to represent the plaintiff competently. Without informing the plaintiff, the attorney consulted a second attorney, who is both a lawyer and a medical doctor and who is a recognized specialist in the care and treatment of injuries of the type sustained by the plaintiff. The two attorneys agreed that the second attorney would participate in the trial to the limited extent of conducting the direct examination and cross-examination of the medical experts and that the original attorney would divide the fee in proportion to the services performed and the responsibility assumed by each. Was the arrangement between the two attorneys proper? A) Yes, because the fee to be paid by the plaintiff was not increased by reason of the second attorney's association. B) Yes, because the fee would be divided in proportion to the services performed and the responsibility assumed by each. C) No, because the plaintiff was not advised of the association of the second attorney. D) No, unless, upon conclusion of the matter, the original attorney provides the plaintiff with a written statement setting forth the method of determining both the fee and the division of the fee with the second attorney.

C) No, because the plaintiff was not advised of the association of the second attorney. Answer choice C is correct. For the two attorneys to split a fee, MRPC 1.5 requires that: (i) the fee be in proportion to the services rendered by each lawyer or that joint responsibility is assumed for the representation, (ii) the client agree in writing to the fee-splitting arrangement, and (iii) the total fee charged be reasonable such that a client is not charged more just because an additional lawyer is working on the case. Here, there is no indication that the plaintiff agreed in writing to the fee-splitting arrangement, so it would be improper. Answer choices A and B are not correct, as each represents only one of the three requirements for a fee-splitting arrangement. Since the plaintiff did not agree in writing, the arrangement is still improper. Answer choice D is not correct, as the plaintiff's consent was required prior to the two attorneys entering into their association on the plaintiff's case.

A plaintiff, who is not a lawyer, is representing himself in small claims court in an action to recover his security deposit from his former landlord. The plaintiff told an attorney, a close friend who lived near him, about this case, but did not ask the attorney for any advice. The attorney said, "I'll give you some free advice. It would help your case if the new tenants would testify that the apartment was in good shape when they moved in, and, contrary to the allegation of your former landlord, it was not, in fact, repainted for them." The plaintiff followed the attorney's advice and won his case. Is the attorney subject to discipline for assisting the plaintiff in preparing for his court appearance? A) Yes, because the attorney assisted the plaintiff in the practice of law. B) Yes, because the attorney offered unsolicited, in-person legal advice. C) No, because the plaintiff was representing himself in the proceedings. D) No, because the attorney was not compensated for his advice.

C) No, because the plaintiff was representing himself in the proceedings. Answer choice C is correct. A lawyer may counsel a non-lawyer who wishes to proceed pro se in a legal matter. MRPC 5.5, cmt 3. Thus, the attorney would not be subject to discipline for assisting the plaintiff. Answer choice A is not correct, as a lawyer may assist a non-lawyer who is representing himself in a legal matter. Answer choice B is not correct, as there was a close personal relationship between the plaintiff and the attorney, and the attorney was not speaking to the plaintiff out of a desire for pecuniary gain. Answer choice D is not correct, as the attorney would not be subject to discipline for assisting the plaintiff even if he was compensated for his advice.

A seller of an apartment building hired an attorney to handle the transaction. Under state law, a seller of commercial real estate is required to provide the buyer with a summary of the leases in effect at the time of the sale. The summary must be completed by a lawyer and must include the lawyer's opinion as to whether the leases are in good standing at the time the summary is prepared. After reviewing the leases, the attorney reasonably concluded that there was no problem with them and that there was no significant risk to providing the buyer with the lease summaries. Without first asking the seller, the attorney then prepared a summary of the leases and provided it to the buyer, with a copy to the seller. The summary included a statement that all of the leases were in good standing. After the buyer reviewed the attorney's summary, the buyer called the seller and asked several questions about the existing leases, which the seller answered. Is the attorney subject to discipline? A) Yes, because the attorney disclosed confidential information of the seller to the buyer without the seller's informed consent. B) Yes, because the attorney failed to advise the seller not to speak with the buyer unless the attorney was present. C) No, because the preparation of the lease summary and evaluation was impliedly authorized by the seller. D) No, because the attorney did not disclose any confidential information in the evaluation.

C) No, because the preparation of the lease summary and evaluation was impliedly authorized by the seller. Answer choice C is correct. When an evaluation prepared by a lawyer and provided to a third party presents no significant risk to the client, the lawyer may be impliedly authorized to disclose information to carry out the representation. This is clearly the case here. Answer choice A is incorrect because, while generally a lawyer is prohibited from the disclosure of a client's confidential information without the client's informed consent, there is an exception when a lawyer prepares an evaluation of a matter affecting a client for the use of someone other than the client when the evaluation presents no significant risk to the client. The lawyer is impliedly authorized to disclose information to carry out the representation. Answer choice B is incorrect because there is no requirement under the Model Rules or other law that a lawyer must advise clients not to speak with other parties outside the presence of the lawyer. Answer choice D is incorrect. The information contained in the leases was information relating to the representation that was protected by confidentiality. However, when an evaluation prepared by a lawyer and provided to a third party presents no significant risk to the client, the lawyer may be impliedly authorized to disclose information, including information otherwise protected by the confidentiality rule, to carry out the representation, which is the case here.

A prosecutor, in accord with state law and with a reasonable belief that probable cause existed, filed an indictment charging another attorney with felony theft. The attorney was arrested. At his bail hearing, the attorney elected, with the court's approval, to proceed pro se. The prosecutor asked the attorney to waive his right to a preliminary hearing, but the attorney refused. At the preliminary hearing, the judge determined that probable cause did not exist to prosecute the attorney and dismissed the charge. Is the prosecutor subject to discipline? A) Yes, because the judge found that probable cause did not exist to prosecute the attorney for felony theft. B) Yes, because the prosecutor asked the accused to waive his right to a preliminary hearing. C) No, because the prosecutor did not know that the felony theft was not supported by probable cause. D) No, because the attorney refused to waive his right to a preliminary hearing.

C) No, because the prosecutor did not know that the felony theft was not supported by probable cause. Answer choice C is correct. A prosecutor must not prosecute a charge that the prosecutor knows is not supported by probable cause. Since here the prosecutor had a reasonable belief that the felony theft charge was supported by probable cause, the prosecutor did not run afoul of this prohibition. Answer choice A is incorrect because, even though the judge found that probable cause did not exist, the prosecutor had a reasonable belief that the felony theft charge was supported by probable cause. Answer choice B is incorrect because, while a prosecutor must generally not attempt to obtain a waiver of important pre-trial rights from an unrepresented accused, such as the right to a preliminary hearing, this prohibition does not apply to an accused appearing pro se with the approval of the tribunal. Answer choice D is incorrect. A prosecutor must not attempt to obtain a waiver of important pre-trial rights from an unrepresented accused, such as the right to a preliminary hearing. However, this rule does not apply to an accused appearing pro se with the approval of the tribunal. The fact that the accused did not waive the right to a preliminary hearing would not have protected the prosecutor had the accused not being proceeding pro se.

An experienced prosecutor asks a defense witness about having been charged with a misdemeanor assault for purposes of impeaching the witness. Although the witness was charged with this crime, the charges were dismissed. The prosecutor knows that the answer to the question would not be admissible under the jurisdiction's evidentiary rules because the crime does not go to the witness's character for truthfulness. Is the prosecutor's conduct proper? A) Yes, because the witness had been charged with the assault. B) Yes, because the prosecutor's purpose was proper. C) No, because the question violated the jurisdiction's evidentiary rules. D) No, because prosecutors are subject to heightened ethics rules.

C) No, because the question violated the jurisdiction's evidentiary rules. Answer choice C is correct. A lawyer must not knowingly disobey an obligation under the rules of a tribunal except for an open refusal based on an assertion that no valid obligation exists. A lawyer also must not allude to matters that are irrelevant or are not supported by evidence that the lawyer reasonably believes will be admissible. Here, the prosecutor knowingly violated the rules of evidence by attempting to impeach the witness with a conviction that would not be admissible for this purpose. Answer choice A is incorrect. Even though it was true that the witness had been charges with assault, the prosecutor knew that this fact could not be supported by admissible evidence. Therefore, the question was improper. Answer choice B is incorrect because, despite having the proper purpose of impeaching the witness, the prosecutor chose to do so by alluding to evidence that could not be admitted under the applicable rules of evidence. This was a knowing violation of an obligation under the rules of the tribunal, and was therefore improper. Answer choice D is incorrect because, while certain special ethical rules apply to prosecutors, they do not apply in this instance. Instead, the prosecutor has violated an ethics rule that applies to any lawyer.?

A criminal defendant who was to be a witness at his own trial told his publicly provided attorney that he intended to testify that he was not at the scene of the crime. The attorney, without knowledge of the defendant's whereabouts at the time of the crime but reasonably believing, based on other evidence, that such testimony would be a lie, counseled the defendant about his obligation to testify truthfully. At trial, even though the defendant indicated that he had not had a change of heart, the attorney called the defendant as a witness, and the defendant testified that he was not at the scene of the crime. Is the attorney subject to discipline for permitting the defendant to testify about his whereabouts at the time of the crime? A) Yes, because the attorney reasonably believed that the client was going to lie. B) Yes, because the attorney has a duty of candor toward the tribunal. C) No, because the witness was a criminal defendant. D) No, because the attorney was publicly provided.

C) No, because the witness was a criminal defendant. Answer choice C is correct. An attorney for a criminal defendant cannot refuse to offer the defendant's testimony when the attorney reasonably believes, but does not know, that the testimony will be false. Consequently, the attorney in this question was required to permit the defendant to testify, and therefore he is not subject to discipline for doing so. Answer choice A is incorrect because, while an attorney may generally refuse to offer evidence that the attorney reasonably believes is false, an attorney is subject to discipline only when the attorney knowingly offers false evidence. Answer choice B is incorrect because, although an attorney has a duty of candor toward the tribunal, the attorney does not breach this duty by offering evidence that the attorney reasonably believes, but does not know, is false. Answer choice D is incorrect because a publicly provided attorney is not subject to special rules with regard to the duty of candor.

An attorney represents the holder of a trademark in a cancellation proceeding before a federal administrative board. The petitioner has alleged that the holder's trademark is confusingly similar to the petitioner's trademark. At the request of the holder, the attorney has posted a link on the attorney's website to an online survey on the holder's website that asks viewers to indicate an opinion as to whether the two trademarks are confusingly similar. The holder wants to introduce the survey results as evidence at the cancellation proceeding. Is it likely that the attorney's posting of this link violates the attorney's ethical duty regarding publicity? A) Yes, because the survey constitutes an extrajudicial statement disseminated by means of a public communication. B) Yes, because the attorney is assisting the client in doing what the attorney cannot ethically do. C) No, because there is no substantial likelihood of materially prejudicing the proceeding in the matter. D) No, because the matter is being tried in an administrative proceeding instead of a judicial proceeding.

C) No, because there is no substantial likelihood of materially prejudicing the proceeding in the matter. Answer choice C is correct. A lawyer who is participating in the investigation or litigation of a matter is not permitted to make an extrajudicial statement that the lawyer knows or reasonably should know will be disseminated by means of public communication and will have a substantial likelihood of materially prejudicing an adjudicative proceeding in the matter. While the posting of a link on the attorney's website to the holder's online survey constitutes an extrajudicial statement disseminated by means of public communication, it is unlikely this statement would materially prejudice the cancellation proceeding. One key factor in determining prejudice can be the nature of the proceeding involved. A criminal jury trial will be most sensitive to extrajudicial speech, while a civil trial may be less sensitive. A non-jury hearing or arbitration proceeding may be even less sensitive. In addition, the purpose of this survey is to generate evidence to be introduced at the proceeding rather than to influence the proceeding with information that would be inadmissible at the proceeding. Answer choice A is incorrect because, although an extrajudicial statement that is disseminated by means of a public communication is subject to the ethical rule on trial publicity, in order to violate this rule, the statement must have a substantial likelihood of materially prejudicing an adjudicative proceeding in the matter. As discussed with regard to answer choice C, it is unlikely this statement would have that effect. Answer choice B is incorrect because, while a lawyer may not knowingly assist or induce another to violate the Model Rules or to do so through the acts of another, even if the attorney were to post the survey directly on the attorney's own website, the attorney's action is unlikely to violate the attorney's ethical duty regarding publicity. Answer choice D is incorrect because, although Model Rule 3.6 is labeled "Trial Publicity," its restrictions apply to a lawyer who is participating or has participated in the investigation or litigation of a matter that is the subject of any adjudicative proceeding, not just a trial.

An attorney represents the holder of a trademark in a cancellation proceeding before a federal administrative board. The petitioner has alleged that the holder's trademark is confusingly similar to the petitioner's trademark. At the request of the holder, the attorney has posted a link on the attorney's website to an online survey on the holder's website that asks viewers to indicate an opinion as to whether the two trademarks are confusingly similar. The holder wants to introduce the survey results as evidence at the cancellation proceeding. Is it likely that the attorney's posting of this link violates the attorney's ethical duty regarding publicity? A) Yes, because the survey constitutes an extrajudicial statement disseminated by means of a public communication. B) Yes, because the attorney is assisting the client in doing what the attorney cannot ethically do. C) No, because there is no substantial likelihood of materially prejudicing the proceeding in the matter. D) No, because the matter is being tried in an administrative proceeding instead of a judicial proceeding..

C) No, because there is no substantial likelihood of materially prejudicing the proceeding in the matter. Answer choice C is correct. A lawyer who is participating in the investigation or litigation of a matter is not permitted to make an extrajudicial statement that the lawyer knows or reasonably should know will be disseminated by means of public communication and will have a substantial likelihood of materially prejudicing an adjudicative proceeding in the matter. While the posting of a link on the attorney's website to the holder's online survey constitutes an extrajudicial statement disseminated by means of public communication, it is unlikely this statement would materially prejudice the cancellation proceeding. One key factor in determining prejudice can be the nature of the proceeding involved. A criminal jury trial will be most sensitive to extrajudicial speech, while a civil trial may be less sensitive. A non-jury hearing or arbitration proceeding may be even less sensitive. In addition, the purpose of this survey is to generate evidence to be introduced at the proceeding rather than to influence the proceeding with information that would be inadmissible at the proceeding. Answer choice A is incorrect because, although an extrajudicial statement that is disseminated by means of a public communication is subject to the ethical rule on trial publicity, in order to violate this rule, the statement must have a substantial likelihood of materially prejudicing an adjudicative proceeding in the matter. As discussed with regard to answer choice C, it is unlikely this statement would have that effect. Answer choice B is incorrect because, while a lawyer may not knowingly assist or induce another to violate the Model Rules or to do so through the acts of another, even if the attorney were to post the survey directly on the attorney's own website, the attorney's action is unlikely to violate the attorney's ethical duty regarding publicity. Answer choice D is incorrect because, although Model Rule 3.6 is labeled "Trial Publicity," its restrictions apply to a lawyer who is participating or has participated in the investigation or litigation of a matter that is the subject of any adjudicative proceeding, not just a trial.

Attorney, recently admitted to practice, opened an office near a residential neighborhood and published the following advertisement in the local newspaper. COUPON Get Acquainted With Your Neighborhood Lawyer A. Attorney Suite 2 - 1100 Magnolia Avenue Sunshine City, State 01000 Telephone: (555) 555-5555 In order to acquaint you with our services, we are offering a one-hour consultation to review your estate plans, including your wills, trusts, and similar documents, all at the nominal cost of $25 to anyone presenting this coupon. Call now for an appointment. Is Attorney subject to discipline? A) Yes, because Attorney is soliciting business from persons with whom Attorney had no prior relationship. B) Yes, because Attorney requires the use of a coupon. C) No, if Attorney provides the services described for the fee stated. D) No, unless Attorney is seeking business from persons who are already represented by a lawyer.

C) No, if Attorney provides the services described for the fee stated. Answer choice C is correct. A lawyer is generally permitted to advertise her services so long as the communications regarding such services are not false or misleading in violation of MRPC 7.1 or in violation of the rules against solicitation of clients under MRPC 7.3. MRPC 7.2(a). Answer choice A is not correct, as this is advertising, not solicitation under the Model Rules. "Solicitation" refers to narrower, targeted communications directed at a small group identified as requiring a specific service (such as letters sent to accident victims, in-person solicitation of employment, or real-time electronic contact). "Advertising" refers to widely distributed, public statements about the services available from a lawyer or law firm (such as phone book listings, or newspaper or television ads). Answer choice B is not correct, as there is nothing in the Model Rules prohibiting the use of a coupon as part of a lawyer's advertisement. Answer choice D is not correct, as the Model Rules permit advertisements, so long as they are not false or misleading or in violation of the rules against solicitation.

The law firm of Alpha and Beta has a radio commercial which states: "Do you have a legal problem? Are you being sued? Consult Alpha and Beta, licensed attorneys at law. Initial conference charge is $25 for one hour. Act now and protect your interests. Call at 1234 Main Street; telephone area code (101) 123-4567." Are Alpha and Beta subject to discipline for the commercial? A) Yes, because the qualifications of the lawyers are not stated. B) Yes, because the radio broadcast may encourage litigation. C) No, if all the statements in the radio broadcast are true. D) No, unless the radio broadcast is heard outside the state in which they are licensed.

C) No, if all the statements in the radio broadcast are true. Answer choice C is correct. A lawyer is generally permitted to advertise his services through public media, including a radio commercial, so long as the communications regarding such services are not false or misleading or in violation of the rules against solicitation of clients. MRPC 7.2(a). Any advertisement must include the name and office address of at least one lawyer or law firm responsible for its content. MRPC 7.2(c). Here, the answer choice indicates that all of the statements in the commercial are true. The name and office address of the lawyers are included. Thus Alpha and Beta would not be subject to discipline. Answer choice A is incorrect, as there is no requirement under the rules that a lawyer list his credentials in an advertisement. Answer choice B is incorrect, as the Model Rules do permit advertising, so long as the advertisement is not false or misleading or in violation of the rules against solicitation. Answer choice D is incorrect, as the Model Rules do not prohibit lawyers from using a radio commercial that might be heard outside of the state in which the lawyers are licensed.

A defendant, who has been indicted for auto theft, is represented by an attorney. The prosecutor reasonably believes that the defendant committed the offense, but, because of the defendant's youth, it is in the interest of justice to permit the defendant to plead guilty to the lesser offense of "joy-riding" in return for an agreement by the prosecutor to recommend probation. The prosecutor has so advised the attorney, but the attorney told the prosecutor she would not plea bargain and would insist on a jury trial. The attorney informed the defendant of the prosecutor's offer and advised the defendant not to accept it. The defendant followed the attorney's advice. The attorney is a candidate for public office, and the prosecutor suspects that the attorney is insisting on a trial of the case to secure publicity for herself. Assume for the purposes of this question that the defendant was tried, convicted, and sentenced to prison for two years. Must the prosecutor report to the disciplinary authority his suspicions about the attorney's conduct in the case? A) Yes, because the defendant suffered a detriment from the attorney's refusal to plea bargain. B) Yes, if the attorney in fact received widespread publicity as a result of the trial. C) No, unless the prosecutor has knowledge that the attorney's refusal to plea bargain was due to personal motives. D) No, if the attorney zealously and competently represented the defendant at the trial.

C) No, unless the prosecutor has knowledge that the attorney's refusal to plea bargain was due to personal motives. Answer choice C is correct. Under MRPC 8.3, a lawyer generally must report misconduct by another lawyer to the appropriate professional authority when the lawyer has actual knowledge of the misconduct. Only if the prosecutor knew that the attorney's refusal to accept the plea bargain was based on an improper motive would the prosecutor be required to report the matter. Answer choice A is not correct, as misconduct must only be reported when there is actual knowledge of the misconduct. The fact that the defendant merely suffered a detriment is not enough to require reporting. Answer choice B is not correct, as only actual knowledge of misconduct imposes a duty to report. The fact that the attorney received widespread publicity does not, by itself, indicate that the attorney did anything wrong and does not require a report to the disciplinary authority. Answer choice D is not correct, as it is irrelevant to the question asked. The only issue is whether the prosecutor has actual knowledge of misconduct that he is required to report.

Under the ABA Model Code of Judicial Conduct, a judge is always prohibited from doing which of the following? A) Testifying as a character witness B) Accepting appointment as a fiduciary C) Running a non-family business D) Practicing law pro se

C) Running a non-family business

An attorney, with her client's approval, settled a claim against a defendant for $60,000. The settlement agreement provided that one-half would be paid by the defendant's primary insurance carrier, and one-half by a co-insurer. The attorney's agreed fee was 30% of the amount of the settlement. The attorney received the primary insurance carrier's check for $30,000 and a letter from the co-insurer advising that its check would be sent in two weeks. The attorney promptly advised the client and deposited the $30,000 in her Clients' Trust Account. The client demanded that the attorney send him the entire $30,000 and take her fee out of the funds to be received from the co-insurer. Which of the following would not be proper for the attorney? A) Send the client $30,000. B) Send the client $21,000 and retain $9,000 in her Clients' Trust Account. C) Send the client $21,000 and transfer $9,000 to her personal account. D) All of the above would be proper.

C) Send the client $21,000 and transfer $9,000 to her personal account. Answer choice C is correct. The key to this question is to determine what funds are disputed, and what the attorney's options are for dealing with these funds. The attorney was entitled to 30% of the amount of the settlement, or $18,000. However, the settlement amount will be paid by two sources, each sending a check for half the funds. When the attorney received the first check for $30,000, a dispute arose regarding whether the attorney is entitled to 30% of each check for half the settlement, which would entitle her to take the first $9,000 of her fee out of this first check, or whether it is proper for the client to demand that the attorney wait and take her full $18,000 fee out of the second check. In either case, the first $21,000 of the first settlement check is undisputed and should be distributed to the client. However, because there is some dispute as to whether the last $9,000 should be treated as part of the lawyer's fee, it would be improper to transfer $9,000 to the attorney's personal account. Since the call of the question asks which option would NOT be a proper course of action, and answer choice C is the only choice that improperly deposits the disputed $9,000 into the attorney's personal account, answer choice C is the correct answer here. Answer choice A is incorrect because it is proper. The attorney can follow the client's instructions and take her fee from the payment to be made by the co-insurer. Answer choice B is incorrect because it is proper. The attorney has a claim to the $9,000 that is disputed by the client; under such circumstances, she may properly keep the $9,000 in the Clients' Trust Account until the claim is resolved, but she must send any undisputed funds to the client. Answer choice D is incorrect because, as described above, answer choice C would not be proper.

An attorney has a highly efficient staff of paraprofessional legal assistants, all of whom are graduates of recognized legal assistant educational programs. Recently, the statute of limitations ran against a claim of a client of the attorney's when a legal assistant negligently misplaced the client's file and suit was not filed within the time permitted by law. Which of the following correctly states the attorney's professional responsibility? A) The attorney is subject to civil liability and is also subject to discipline on the theory of respondeat superior. B) The attorney is subject to civil liability or is subject to discipline at the client's election. C) The attorney is subject to civil liability but is NOT subject to discipline unless the attorney failed to supervise the legal assistant adequately. D) The attorney is NOT subject to civil liability and is NOT subject to discipline if the attorney personally was not negligent.

C) The attorney is subject to civil liability but is NOT subject to discipline unless the attorney failed to supervise the legal assistant adequately. Answer choice C is correct. Under the doctrine of respondeat superior, as the supervisor of the legal assistant, the attorney would be subject to civil liability for malpractice for allowing the statute of limitations to expire. Unless the attorney failed to adequately supervise the legal assistant, however, the attorney would not be subject to discipline under the Model Rules. Under MRPC 5.3(c), a lawyer will be responsible for the conduct of a non-lawyer assistant that would be a violation of the Model Rules if: (i) the lawyer orders or, with the knowledge of the specific conduct, ratifies the conduct involved, or (ii) the lawyer is a partner or has comparable managerial authority in the law firm in which the non-lawyer assistant is employed, or has direct supervisory authority over the person, and knows of the conduct at a time when its consequences can be avoided or mitigated but fails to take reasonable remedial action. Answer choice A is not correct, as the Model Rules do not subject an attorney to discipline on the theory of respondeat superior. A lawyer is subject to discipline only if the lawyer orders or specifically ratifies the conduct. Answer choice B is not correct because even though a client can make a complaint seeking disciplinary action, only the jurisdiction's disciplinary authority can subject a lawyer to discipline. Answer choice D is not correct, as the attorney would be subject to civil liability for malpractice under the doctrine of respondeat superior.

Which of the following is NOT a circumstance in which a lawyer is permitted to reveal confidential information? A) The client consents B) To prevent death or substantial bodily injury C) To prevent the client's future criminal act unrelated to the lawyer's services D) To settle a dispute between the lawyer and client

C) To prevent the client's future criminal act unrelated to the lawyer's services

A plaintiff sued a defendant in a negligence action regarding a car accident, and during the trial, a witness for the plaintiff testified that she had observed the defendant running a red light which caused the collision. Upon the conclusion of the trial in the plaintiff's favor, but before the time for review on appeal had passed, the witness contacted the plaintiff's attorney and confessed that she had falsely testified about seeing the defendant run the red light. The witness stated that she felt sorry for the plaintiff and had wanted to bolster her case, but now regretted her false testimony and wanted to tell the truth. Does the attorney have a duty to remedy the witness's false testimony? A) No, because the false testimony was given by a witness and not by the attorney's client. B) No, because the trial has concluded. C) Yes, because a lawyer's duty to remedy false testimony continues until the time for review on appeal has passed. D) Yes, because the witness has told the attorney that she wants to tell the truth.

C) Yes, because a lawyer's duty to remedy false testimony continues until the time for review on appeal has passed. Answer choice C is correct. If a lawyer, the lawyer's client, or a witness called by the lawyer, has offered material evidence and the lawyer comes to know of its falsity, Model Rule 3.3 requires the lawyer to take reasonable remedial measures. This obligation continues through the conclusion of the proceeding, which is defined as when a final judgment in the proceeding has been affirmed on appeal or the time for review has passed. The facts state that the time for appellate review has not passed, so the proceeding has not concluded. As a result, the attorney's duty to take reasonable remedial measures applies. Answer choice A is incorrect. A lawyer may not offer any evidence that the lawyer knows to be false, and if the lawyer comes to know of the falsity of testimony of a witness called by the lawyer before the conclusion of the proceeding, the lawyer must take reasonable remedial measures. Answer choice B is incorrect because the time for appellate review has not passed. Therefore, the proceeding has not concluded, and the attorney's duty to take reasonable remedial measures still applies. Answer choice D is incorrect. The fact that the witness told the attorney that she wants to tell the truth is irrelevant to the attorney's duty of candor under Rule 3.3. The rule requires a lawyer to take reasonable remedial measures if the lawyer comes to know that a witness called by the lawyer offered material false testimony, regardless of whether the witness wants to correct the testimony.

After undergoing numerous corrective surgeries to fix a botched abdominoplasty performed on her by a plastic surgeon, a patient retained an attorney in order to file a medical malpractice claim against the plastic surgeon. The abdominoplasty was performed three years earlier, but the statute of limitations to file a medical malpractice claim in the jurisdiction was two years. Knowing that the time to timely file the claim had passed, the attorney filed the claim and did not mention the statute of limitations. The plastic surgeon's defense attorney did not object. Is the attorney subject to discipline? A) No, because an attorney who voluntarily offers adverse facts will be subject to discipline for breaching the duty of loyalty to his client. B) No, because the attorney was not required to disclose to the tribunal that there was legal authority directly adverse to the position of the client. C) Yes, because an attorney must disclose legal authority known to him to be directly adverse to the client and that was not disclosed by opposing counsel. D) Yes, because the attorney owed a duty of fairness to opposing counsel.

C) Yes, because an attorney must disclose legal authority known to him to be directly adverse to the client and that was not disclosed by opposing counsel. Answer choice C is correct. A lawyer must disclose to the tribunal legal authority in the controlling jurisdiction known to the lawyer to be directly adverse to the position of the client and that was not disclosed by opposing counsel. Here, the attorney knew that the statute of limitations to file a medical malpractice claim was two years, and thus he must disclose this legal authority known to him that was not disclosed by opposing counsel. Therefore, the attorney is subject to discipline. For this reason, answer choice B is incorrect. Answer choice A is incorrect. An attorney who voluntarily offers adverse facts may be subject to discipline for breaching the duty of loyalty to his client. However, when legal authority is involved, an attorney may be required to disclose such legal authority to the tribunal known to the lawyer to be directly adverse to the position of the client. Answer choice D is incorrect. Although in general an attorney does owe a duty of fairness to opposing counsel, in this case the attorney owed a duty of candor to the tribunal.

An attorney represented his client in a products liability action against the manufacturer of a malfunctioning blender which caused the client injuries totaling over $750,000. Prior to trial, the two parties entered into settlement negotiations, and the manufacturer offered the client $500,000 to settle the case. However, as part of the settlement, the attorney had to agree not to file any further products liability actions against the manufacturer regarding the malfunctioning blender on behalf of any future plaintiffs who were injured by it. The attorney wanted to get the best possible settlement offer for his client, and he did not have time in his practice for another case such as this one. For this reason, the attorney agreed to the condition, and the client, who was happy with the settlement amount, accepted the settlement offer. Is the attorney subject to discipline? A) No, because the attorney did not have time to litigate another products liability action against the manufacturer. B) No, because the attorney was acting in the best interests of his client and with his client's consent by agreeing to the terms of the settlement agreement. C) Yes, because attorneys cannot make settlement agreements restricting their right to practice in the future. D) Yes, because the attorney accepted a settlement for significantly less than the value of the client's damages.

C) Yes, because attorneys cannot make settlement agreements restricting their right to practice in the future. Answer choice C is correct. A lawyer must not make or offer an agreement in which a restriction on the lawyer's right to practice is part of the settlement of a client controversy. Here, the attorney agreed not to file any further products liability actions against the manufacturer regarding the malfunctioning blender on behalf of any future plaintiffs who were injured by it. This is not permissible under the Model Rules. Therefore, the attorney is subject to discipline. Answer choice A is incorrect because even an agreement that only prevents the lawyer from accepting representation in cases that the lawyer would not otherwise have accepted is a restriction on the lawyer's right to practice. Answer choice B is incorrect. Although the attorney may have been acting in the best interests of his client by agreeing to the terms of the settlement agreement, he cannot make settlement agreements restricting his right to practice in the future. Answer choice D is incorrect because the ultimate decision as to whether to accept a settlement agreement rests with the client, regardless of the value of the settlement.

A bicyclist who was injured when he hit a pothole in a sidewalk wanted to bring a civil claim for damages against the municipal agency responsible for repairing the pothole. As permitted by local law, the bicyclist and the agency mutually decided to pursue mediation instead to limit their expenses. The agency contacted a state court judge who presided in a county that was located a significant distance away from the municipality where the bicyclist was injured. The agency asked the judge if she would serve as a mediator in the dispute. Reasonably believing that she would never be assigned to any lawsuit involving the municipal agency or the bicyclist, and after receiving the informed written consent of the bicyclist and the head of the municipal agency, the judge agreed to serve as a mediator in the dispute. The mediation occurred only on the weekends, so it did not interfere with the timely completion of the judge's assigned judicial duties. In addition, the judge did not charge a fee for mediating the dispute. Is the judge subject to discipline? A) No, because the judge reasonably believed that she would never be assigned to any lawsuit involving the municipal agency, and the parties consented to the mediation. B) No, because the mediation did not interfere with the timely completion of the judge's assigned judicial duties, and she did not charge a fee C) Yes, because judges are not permitted to act as mediators apart from their assigned judicial duties. D) Yes, because judges are not permitted to act as mediators.

C) Yes, because judges are not permitted to act as mediators apart from their assigned judicial duties. Answer choice C is correct. Model Rule 3.9 of the Code of Judicial Conduct states that a judge shall not act as an arbitrator or a mediator or perform other judicial functions apart from the judge's official duties unless expressly authorized by law. The rule does not prohibit a judge from participating in arbitration, mediation, or settlement conferences performed as part of assigned judicial duties. Here, the facts describe a mediation unrelated to the judge's assigned duties. Therefore, the judge is subject to discipline. Answer choice A is incorrect. Although the judge reasonably believed that she would never be assigned to any lawsuit involving the parties, a judge cannot act as an arbitrator apart from her assigned judicial duties, even with the consent of the parties. Answer choice B is incorrect. Although the mediation did not interfere with the timely completion of the judge's assigned judicial duties, and she did not charge a fee, a judge cannot act as an arbitrator apart from her assigned judicial duties. Answer choice D is incorrect because it overstates the limitation on a judge's ability to serve as mediators. Judges may serve as mediators in connection with the judges' official duties.

A sole practitioner represented a client in a contract involving the sale of his shoe manufacturing business. Due to his relative inexperience, the sole practitioner decided to work on the sale with a more experienced attorney at another law firm. The attorney agreed to perform 70% of the legal work in the matter in exchange for 50% of the total fee. The total fee paid by the client was reasonable, and the sole practitioner and the attorney each agreed to assume joint responsibility for the representation. The client agreed to the terms of the agreement, but the agreement was not confirmed in writing. Upon the conclusion of the sale, the attorney received 50% of the total fee. Is the sole practitioner subject to discipline? A) No, because the client agreed to the terms of the agreement and the total fee was reasonable. B) No, because the sole practitioner and the attorney agreed to assume joint responsibility for the representation. C) Yes, because the agreement was not confirmed in writing. D) Yes, because the division of the fee was not in proportion to the services performed by the sole practitioner and the attorney.

C) Yes, because the agreement was not confirmed in writing. Answer choice C is correct. Model Rule 1.5(e) states that a division of a fee between lawyers who are not in the same firm may be made only if (i) the fee is in proportion to the services rendered by each lawyer or joint responsibility is assumed for the representation, (ii) the client agrees in writing to the fee splitting arrangement, and (iii) the total fee charged is reasonable. Here, the agreement was not confirmed in writing. Therefore, the sole practitioner is subject to discipline. Answer choice A is incorrect because although the total fee was reasonable, the agreement was not confirmed in writing. Answer choice B is incorrect because although the attorneys agreed to assume joint responsibility for the representation, the agreement was not confirmed in writing. Answer choice D is incorrect. Under the Model rules, a fee splitting agreement between lawyers in different firms is not required to be in proportion to the services performed by each lawyer as long as each lawyer assumes joint responsibility. Here, each lawyer agreed to assume joint responsibility, so the agreement need not split the fee in proportion to their efforts to comply with Rule 1.5. However, the attorney is still subject to discipline because the agreement was not confirmed in writing.

An indigent defendant was accused of robbery of a woman at gunpoint, and a trial judge appointed a criminal defense attorney to represent him. Two years ago, the defense attorney had been held at gunpoint by a robber. Although that robber had been convicted and sent to prison, the defense attorney was still severely disturbed by the incident and refused to represent any defendant accused of robbery in her private practice. However, because of the court appointment, the attorney agreed to meet with the indigent defendant. After the interview, the attorney surmised that she would be able to competently represent the defendant, but that her repugnance towards him would most likely impair their attorney-client relationship. For this reason, the attorney sought to decline the court appointment, citing her reasons for doing so to the court without violating any of the indigent defendant's confidences. Was the attorney's conduct proper? A) No, because the attorney had declined to represent any defendant accused of robbery in her private practice. B) No, because the attorney sought to avoid the court appointment even though she believed that she could competently represent the defendant. C) Yes, because the attorney believed that her repugnance toward the defendant was likely to impair the attorney-client relationship. D) Yes, because the attorney was not required to accept a client whom she considered repugnant.

C) Yes, because the attorney believed that her repugnance toward the defendant was likely to impair the attorney-client relationship. Answer choice C is correct. Under Model Rule 6.2, a lawyer has a duty to accept court appointments, so long as no good cause exists to decline. Good cause to decline representation exists if: (i) the lawyer could not handle the matter competently; (ii) if the representation would result in an improper conflict of interest; or (iii) if acceptance would be unreasonably burdensome. One example of an improper conflict of interest would arise when the client or the cause is so repugnant to the lawyer as to be likely to impair the client-lawyer relationship or the lawyer's ability to represent the client. Here, the attorney had good cause to decline the court appointment because she believed that her repugnance toward the defendant was likely to impair the attorney-client relationship. Answer choice A is incorrect. With the exception of court appointments, a lawyer is not under a duty to accept representation of any client. A lawyer is generally not required to have a good cause for refusing to accept representation of a client, or even to tell a potential client the lawyer's reasons for declining to represent a client. Therefore, it was not improper for the attorney to decline to represent defendants accused of robbery in her private practice. Answer choice B is incorrect. Even though the attorney believed that she could competently represent the indigent defendant, she was nevertheless permitted to avoid the appointment for good cause based on her conflict of interest that would impair the attorney-client relationship. Answer choice D is incorrect. The attorney could be required to accept a client whom she considered repugnant unless good cause existed. However, because the attorney believed that her repugnance toward the defendant was likely to impair the attorney-client relationship, there is good cause for the attorney to decline this appointment.

A corporation that owned oil tankers hired an attorney to represent its interests at a legislative committee during a session of the state legislature. The legislative committee was investigating the environmental impact of crude oil spills by oil tankers upon the ocean. The attorney agreed to testify before the committee. To prepare, the attorney acquired data from the corporation regarding the number and magnitude of oil spills that had occurred from the corporation's oil tankers over the past ten years. The attorney prepared a written report and included the corporation's data in an appendix. She also testified and answered questions about the corporation's oil spills. Before the committee had proposed any new legislation, the attorney learned that the corporation had severely underreported the number of oil spills that had occurred from its oil tankers over the past ten years. The attorney counseled the corporation to correct the inaccurate data, but the corporation told the attorney not to reveal the correct numbers to the legislative committee. Believing that she was obligated to follow the corporation's directions, the attorney did not correct the data she had provided to the committee. Is the attorney subject to discipline? A) No, because the attorney believed that the data was accurate when she submitted it to the legislative committee. B) No, because the corporation refused to reveal the correct data to the legislative committee even after the attorney counseled it to do so. C) Yes, because the attorney did not correct the data she had provided to the legislative committee. D) Yes, because the attorney did not withdraw from the representation of the corporation when the corporation refused to correct the misinformation.

C) Yes, because the attorney did not correct the data she had provided to the legislative committee. Answer choice C is correct. A lawyer who represents a client before a legislative body must disclose that she is there in a representative capacity and must generally conform to the Model Rules with regard to candor toward the entity. The duty of candor under Model Rule 3.3 requires a lawyer who has offered material evidence and later comes to know of its falsity to take reasonable remedial measures, including, if necessary, disclosure to the tribunal (or legislative body). Thus, the attorney was obligated to correct the false information she provided to the legislative committee. Answer choice A is incorrect. The fact that the attorney believed the data to be accurate when it was submitted did not remove the duty to take reasonable remedial measures once the attorney came to know of the data's falsity. Answer choice B is incorrect. The corporation's objection to remedial measures, including disclosure, did not excuse the attorney's duty to take reasonable remedial measures. Answer choice D is incorrect. The attorney's withdrawal from the representation of the corporation would not be sufficient to correct the false data provided to the legislative committee.

An attorney represented a criminal defendant in a murder case. The attorney's client told the attorney that he wanted to testify in his own defense. The attorney tried to dissuade his client from testifying, in part because although he had no evidence to support it, he suspected that the client's version of the events was fabricated. The attorney also believed that the jury would not believe his client. The client insisted, however, that he testify. The attorney told his client that he would call the client to the stand but reminded him that he was under oath and that it was imperative that he answer all questions truthfully. At trial, the client testified in his own defense, and the prosecution subsequently impeached the defendant's testimony. Was the attorney's action in allowing his client to testify proper? A) No, because the attorney reasonably believed that his client's testimony would be false. B) No, because the attorney was required to take remedial measures. C) Yes, because the attorney did not know that the client's testimony would be false. D) Yes, because an attorney is required to respect a criminal defendant's decision to testify

C) Yes, because the attorney did not know that the client's testimony would be false. -Read the call of the question first. This is an issue of CANDOR TO THE COURT because this is an issue about whether or not the client should be allowed to testify. The rule is: even with the attorney's advice, the client ultimately MAKES the DECISION ON WHETHER TO TESTIFY OR NOT and if the ATTORNEY MUST HAVE ACTUAL KNOWLEDGE that the client committing perjury, REASONABLY BELIEF IS NOT ENOUGH. -A) is wrong because it is not in compliance with the rule -B) is wrong because you're only required to take remedial measures AFTER the PERJURY HAS BEEN COMMITTED -C) is correct because the lawyer wasn't sure, and HE REASONABLY BELIEVED IT. The lawyer had no KNOWLEDGE about the testimony being false. -D) is wrong even though it's right, IT IS NOT the issue here. The issue here is whether it's proper to put up evidence an attorney REASONABLY BELIEVES might be FALSE.

A farmer asked his attorney to draft a letter in connection with a loan transaction. The letter issued by the attorney stated that the farm equipment to be pledged by the farmer to the lender to secure the loan was not subject to prior liens. The letter did not indicate that the attorney had relied on the farmer's statement to that effect and had not performed, as custom would require, a search of the relevant public records. Such a search would have revealed that the farmer's statement was false. At the farmer's direction, the attorney, unaware that the farmer had misinformed him as to the existence of the liens, sent the letter to the lender, who relied on it in making the loan. Subsequently, the farmer defaulted on the loan and, due to the prior liens, the lender was unable to recoup the outstanding balance owed by the farmer on the loan. Is the lawyer likely subject to civil liability to the lender? A) No, because the lender was not the attorney's client. B) No, because the attorney relied on his client's statement. C) Yes, because the attorney failed to comply with his duty of care with regard to the letter. D) Yes, because, by issuing the letter, the attorney became a guarantor of the loan.

C) Yes, because the attorney failed to comply with his duty of care with regard to the letter. Answer choice C is correct. While generally a lawyer does not owe a duty to a non-client, a lawyer can owe a duty of care to a non-client when the non-client has been invited to rely on the opinion or legal services of the lawyer, and the non-client does so. This is particularly true when the lawyer is aware that his opinion is being used with regard to a specific transaction and is being supplied to a particular person, rather than the public in general. In this instance, since the attorney failed to search publicly available records, as custom would require, he violated that duty of care. Moreover, the lender was harmed by the attorney's breach of this duty. Answer choice A is incorrect because, although generally a lawyer does not owe a duty to a non-client, a lawyer may owe a duty to a non-client when the non-client has been invited to rely on the opinion or legal services of the lawyer, and the non-client does so to her detriment. Answer choice B is incorrect because, although a lawyer who relies on information provided by a client is generally not liable to the client for doing so, such reliance may not protect the lawyer from liability to a third party when, in so relying, the lawyer fails to exercise reasonable care. Answer choice D is incorrect because the attorney, in issuing an opinion, did not assume an obligation with respect to the loan itself.

A patient contacted an attorney to represent her in a negligence action against her plastic surgeon after he accidentally made a hole in her septum while performing a cosmetic procedure on her nose. In accordance with the patient's instructions, the attorney attempted to negotiate a settlement with the surgeon prior to filing a lawsuit. The patient had required additional surgery to repair her septum. During the settlement negotiations, the attorney provided the surgeon with the patient's medical records and a calculation of the costs related to that reparative surgery. After some time, the surgeon made a settlement offer which the attorney relayed to the patient and the patient approved. However, prior to the finalization of the agreement, the patient unexpectedly died. Without informing the surgeon of the patient's death, the attorney signed the agreement on behalf of the patient. Is the attorney subject to discipline? A) No, because the fact of the patient's death was information relating to the representation. B) No, because the surgeon could have discovered the patient's death through due diligence. C) Yes, because the attorney failed to inform the surgeon of the patient's death. D) Yes, because the attorney provided the patient's medical records to the surgeon.

C) Yes, because the attorney failed to inform the surgeon of the patient's death. Answer choice C is correct. A lawyer generally has no affirmative duty to inform an opposing party of relevant facts. However, a lawyer has a duty to inform opposing counsel of the death of a client in the first communication after the lawyer has learned of that fact. By purporting to settle the matter on the patient's behalf after learning of the patient's death, without disclosing the death to the surgeon, the attorney misrepresented a material fact in violation of Model Rule 4.1 because the attorney no longer had a client and therefore had no actual authority to act on the client's behalf. Answer choice A is incorrect. A lawyer is impliedly authorized by Model Rule 1.6(a) to disclose client information necessary to carry out the representation. In this situation, the disclosure of the patient's death was necessary to avoid a misrepresentation by omission. Answer choice B is incorrect. That the surgeon could have learned of the patient's death through other means does not remove the attorney's duty to inform opposing counsel of the death of a client in the first communication after the lawyer has learned of that fact. Answer choice D is incorrect. The attorney's disclosure of the patient's medical records to the surgeon, before the patient's death, was impliedly authorized to carry out the representation and therefore permitted by Model Rule 1.6(a). ?

An attorney worked for a government agency that was involved in the maintenance of public roadways. While there, the attorney spent all of his time working on an ongoing dispute concerning the re-pavement of a specific highway. A short time later, the attorney left his government job in order to pursue a career as a sole practitioner. Six months after leaving his government post, the attorney was approached by a private client who was seeking representation with regards to the same dispute concerning the highway that the attorney had worked on while at the government agency. Because the private client took the same position as the government agency with regards to the dispute, the attorney had a reasonable belief that there would not be a conflict of interest between the two parties. He sent a letter notifying the government agency that he was representing the private client in the dispute and gave the agency 45 days to object to the representation. After 45 days had passed with no reply from the government agency, the attorney took on the representation of the private client. Is the attorney subject to discipline? A) No, because the attorney gave the government agency notice of the potential representation and an opportunity to object to it. B) No, because the new client's interests were not adverse to the government agency. C) Yes, because the attorney should have gotten the government agency's informed consent, confirmed in writing, to the representation. D) Yes, because there is a substantial risk that the attorney obtained confidential factual information while working for the government agency that would materially advance the client's position in this matter.

C) Yes, because the attorney should have gotten the government agency's informed consent, confirmed in writing, to the representation. Answer choice C is correct. The Model Rules prohibit a former government lawyer from representing a client in a matter in which the lawyer participated personally and substantially as a government lawyer, unless the appropriate government agency gives its informed consent, confirmed in writing, to the representation. Here, the attorney spent all of his time as a government attorney working on the ongoing dispute concerning the re-pavement of a highway. Therefore, he participated personally and substantially in the matter. As such, he was prohibited from representing the private client in the matter unless he obtained the government agency's informed consent, confirmed in writing. Answer choice A is incorrect because it is not enough that the attorney gave the government agency notice of the potential representation and an opportunity to object to it. He also needed to get the government agency's informed consent, confirmed in writing, to the representation. Answer choice B is incorrect. The prohibition applies regardless of whether the new client's interests are adverse to the government. Thus, a lawyer who has pursued a claim on behalf of the government may not pursue the same claim on behalf of a subsequent private client after the lawyer has left government service, except when the government agency gives its informed, written consent. Answer choice D is incorrect because a former government lawyer's conflicts of interest are triggered whenever they participated in a specific matter personally and substantially, not when they may have merely obtained confidential factual information while working for the government agency.

An attorney represented the defendant in an aggravated assault action. While the client was in jail, he was attacked by another inmate and received a concussion. The attorney had his client immediately evaluated by a physician to ensure that the concussion did not impact the defendant's ability to recount the events of the assault. Hours before the client was to testify, the physician called the attorney to tell him that the concussion did not cause any memory problems but that a secondary psychiatric opinion noted that the defendant did have a personality disorder. The psychiatrist noted that the disorder would not affect the client's decision-making process or ability to recount events, but it should be treated with intensive, in-patient therapy as soon as possible to improve quality of life. The attorney believed that the trial would end in the next day or two if they proceeded, and because he had a trial-heavy few months after the trial in question, he did not want to delay the trial further by immediately sending his client for treatment. He was confident that delaying treatment for a few days would not harm his client and that the client was competent to testify, as the trial was nearly over. The attorney therefore waited to tell his client about the diagnosis until after the trial was complete. Is the attorney subject to discipline for withholding this information from the client? A) No, because the attorney did not believe that the delay would harm the client. B) No, because the attorney followed the correct process for evaluating a client's mental status. C) Yes, because the attorney withheld the information to serve his own interests. D) Yes, because the attorney withheld information relating to a psychiatric diagnosis.

C) Yes, because the attorney withheld the information to serve his own interests. Answer choice C is correct. An attorney may be justified in delaying transmission of information to a client when the client would be likely to react imprudently to an immediate communication. Thus, an attorney might withhold a psychiatric diagnosis of a client when the examining psychiatrist indicates that such disclosure would harm the client. An attorney may not, however, withhold information to serve the attorney's own interests or convenience or the interests or convenience of another person. Here, the attorney withheld the information because a delay in the trial would be inconvenient for his own schedule, and thus, his actions are improper. Answer choice A is incorrect because the attorney's motivation in withholding the information was related to his own interests rather than concern over harming the client. Answer choice B is incorrect because the attorney's actions following his receipt of the results of the client's evaluation did not follow the correct process. Answer choice D is incorrect because an attorney may, in some circumstances, withhold certain information from a client, including information about a psychiatric diagnosis. In this case, however, the attorney's motivation for withholding it would subject him to discipline.

An attorney was approached by a husband and wife who decided to jointly execute a will. In addition to some jointly owned property that the couple agreed to devise to their only child, the wife mentioned that she was interested in devising some jointly owned stock to her sister. The husband expressed his disagreement with this plan. Despite this disagreement, the attorney believed that he could still provide competent and diligent representation to both the husband and the wife, so he agreed to jointly represent them. The spouses gave their informed consent to joint representation, but the attorney did not obtain a written confirmation of the couple's informed consent. At the next meeting with the attorney, the husband stated that the stock was to be devised to the couple's child. The wife was in attendance at the meeting, and she did not express her disagreement with the plan. The attorney subsequently drafted a will devising the stock to the child. Is the attorney subject to discipline? A) No, because the attorney reasonably believed he could provide both parties with diligent and competent representation. B) No, because the wife did not express her disagreement with the plan to devise the stock to the couple's child. C) Yes, because the couple did not give informed consent, confirmed in writing. D) Yes, because the husband's and wife's interests were directly adverse.

C) Yes, because the couple did not give informed consent, confirmed in writing. Answer choice C is correct. A lawyer must not represent a client if doing so would be directly adverse to the interests of another current client or there is a significant risk that the representation of the client will be materially limited by the lawyer's responsibilities to the current client, unless: (i) the lawyer reasonably believes that the lawyer will be able to provide competent and diligent representation to each affected client; (ii) the representation is not prohibited by law; (iii) the representation does not involve the assertion of a claim by one client against another client represented by the lawyer in the same litigation or other proceeding before a tribunal; and (iv) each affected client gives informed consent, confirmed in writing. Here, the husband and wife had interests that were directly adverse to one another because the wife wanted to devise the jointly owned stock to her sister, and the husband wanted to devise it to the couple's only child. Although the attorney believed that he would be able to provide competent and diligent representation to the clients, neither the husband nor the wife gave informed consent, confirmed in writing. Therefore, the attorney is subject to discipline. For this reason, answer choice A is incorrect. Answer choice B is incorrect. Regardless of whether the wife voiced her disagreement with the plan to devise the stock to the couple's child at the subsequent meeting, the attorney failed to obtain her informed consent, confirmed in writing, to the joint representation. Therefore, the attorney is subject to discipline. Answer choice D is incorrect. It is permissible for the husband's and wife's interests to be directly adverse as long as they are not opposing parties represented by the same attorney in the same lawsuit and the enumerated elements above are satisfied. Here, had each affected client gives informed consent, confirmed in writing, the representation would have been permitted.

During a conversation among friends in his home, an attorney called into question whether a candidate for the office of attorney general met the statutory requirements. Specifically, the attorney stated that he had heard a rumor that the candidate had not been licensed to practice law in the state for the requisite number of years mandated by statute. When asked whether he had investigated this matter, the attorney replied that he didn't need to check the facts, it had to be true. The candidate had, in fact, been licensed to practice law for the statutorily required time period. Is the attorney subject to discipline? A) No, because the statement did not concern a current member of the judiciary or a candidate for judicial office. B) No, because the statement was not made in a public forum. C) Yes, because the false statement was made with a reckless disregard for its truth or falsity. D) Yes, because the statement concerned a candidate for public office.

C) Yes, because the false statement was made with a reckless disregard for its truth or falsity. Answer choice C is correct. False statements by a lawyer can unfairly undermine public confidence in the administration of justice. The prohibition on such statements applies to the qualifications or integrity of not only a judge or a candidate for judicial office, but also a public legal officer, such as an attorney general, or a candidate for such office. Consequently, a lawyer must not make a statement that the lawyer knows to be false or with reckless disregard as to its truth or falsity (i.e., a false statement made with actual malice) concerning the qualifications or integrity of a judge, adjudicatory officer, or public legal officer, or of a candidate for election or appointment to judicial or legal office. Because the attorney evidenced a reckless disregard for the truth or falsity of his statement regarding the failure of the attorney general candidate to meet the statutory requirements, the attorney is subject to discipline. Answer choice A is incorrect because, as noted with regard to answer choice C, the prohibition on a false statement made with actual malice about the qualifications or integrity of an individual applies not only to a judge or a candidate for judicial office, but also to a public legal officer, such as an attorney general, or a candidate for such office. Answer choice B is incorrect because the prohibition applies to false statements made with actual malice, regardless of whether the statements are made in private or in public. Answer choice D is incorrect because the prohibition on false statements does not apply to any candidate for a public office, but is limited to candidates for judicial or legal offices, such as judges, attorney generals, prosecuting attorneys, and public defenders.

A plaintiff was standing on the sidewalk next to an open parking spot where a car was about to parallel park using its automatic parking feature. The plaintiff was injured when the automatic parking system malfunctioned, and the car jumped the curb and struck the plaintiff, causing her serious injuries. The plaintiff subsequently filed a products liability action against the manufacturer of the automatic parking system under a design defect theory of recovery. Because the judge knew relatively little about the technology surrounding automatic parking systems, he decided to consult with an outside expert on the subject matter. The judge notified both parties in writing of his intention to consult with an expert who was not related to any party or other expert in the litigation. A description of the subject matter of the consultation with the expert was included in the written notice, as well as a two-week window within which the parties had to object and respond to the consultation. The written notice also stated that the parties would have 45 days to respond to any information provided to the judge by the expert. Was the judge's conduct proper? A) No, because the judge's consultation would be an impermissible ex parte communication with an expert. B) No, because the judge was required to obtain written consent from the parties prior to consulting with an outside expert. C) Yes, because the judge gave the parties advance notice of his intention to consult with the outside expert, notice of the subject matter to be solicited, and an opportunity to object and respond to the notice and information received. D) Yes, because the judge notified the parties of his intention to consult with the outside expert, which is all that was required.

C) Yes, because the judge gave the parties advance notice of his intention to consult with the outside expert, notice of the subject matter to be solicited, and an opportunity to object and respond to the notice and information received. Answer choice C is correct. Under Rule 2.9(A)(2) of the Code of Judicial Conduct, a judge is permitted to obtain the written advice of a disinterested expert on the law applicable to a proceeding before the judge, if the judge gives advance notice to the parties of the person to be consulted and the subject matter of the advice to be solicited, and gives them a reasonable opportunity to object and respond to the notice and to the advice received. Here, the judge met the conditions in the rule: the judge gave the parties advance notice indicating the person to be consulted and the subject matter of the information to be solicited and provided the parties with an opportunity to object to and respond to the notice and the information received. Answer choice A is incorrect. The Code of Judicial Conduct does not prohibit judges from all ex parte consultations with disinterested experts. Answer choice B is incorrect. Written consent of the parties is not required when the judge informs parties of the judge's intention to consult with an expert. Answer choice D is incorrect. In addition to notifying the parties of his intention to consult with the outside expert, the judge must inform the parties of the subject matter of the information to be solicited and notify parties of their opportunity to object to and respond to the notice and the information received.

The chair of one of a state's political parties approached a law firm's senior partner, who was very active within that party, about representing the political party in litigation opposing a ballot measure backed by the state's opposing party. A junior partner at the firm was an active member of the opposing party and had contributed substantial time and money toward getting the measure on the ballot. The firm had never previously represented any political party or organization. After discussing the matter with the firm's management, who were all aware of the junior partner's involvement with getting the measure on the ballot, the senior partner decided to represent the political party. Is the senior partner's representation of the state's political party proper? A) No, because one of the firm's partners has a conflict of interest. B) No, because the junior partner's conflict of interest would be imputed to the other partners. C) Yes, because the junior partner's conflict of interest would not be imputed to the other partners. D) Yes, because there is no conflict of interest.

C) Yes, because the junior partner's conflict of interest would not be imputed to the other partners. Answer choice C is correct. In general, if one attorney in a firm is prohibited by the conflict-of-interest rules from representing a client, that prohibition applies to all other attorneys in the firm. If the prohibition is based on a personal interest of the disqualified attorney and does not present a significant risk of materially limiting the representation of the client by the remaining attorneys in the firm, then the disqualified attorney's conflict will not be imputed to the firm. In this case, although the junior partner had a conflict of interest given his involvement in getting the measure on the ballot, this conflict would be based on his personal interest. Because there is no indication that the junior partner's interest would materially limit the representation of the client by the remaining attorneys in the firm, the conflict would not be imputed to the other attorneys. Answer choice A is incorrect because although the junior partner likely would have a conflict of interest, this would not prevent another attorney from representing the client unless the conflict was imputed to the other attorneys. Answer choice B is incorrect because the conflict would not be imputed to the other attorneys. Answer choice D is incorrect because the junior partner had a conflict of interest based on his personal involvement in placing the measure on the ballot.

Over a period of many years, a small company that sold artisanal sodas was represented by a partner in a law firm in its business transactions. The company's president recently discovered that one of the firm's associate lawyers was counseling a soda conglomerate about the prospect of suing the company in a trademark infringement action. The president immediately asked the law firm to cease representing the conglomerate in connection with the potential lawsuit because she was worried that the associate counseling a soda conglomerate might also have access to the company's confidential information. The law firm refused to withdraw from its representation of the conglomerate. However, it did offer to screen the associate to avoid access to or misuse of the company's confidential information. Dissatisfied with the law firm's offer, the president retained independent counsel and sought an injunction in a civil action to bar the law firm from its continued representation of the conglomerate in connection with the potential trademark infringement action. Is the court likely to grant the requested injunctive relief? A) No, because the law firm has not committed any professional negligence. B) No, because the proper remedy for the law firm's breach of its duty is a disciplinary action. C) Yes, because the law firm is breaching its fiduciary duty to the small soda company. D) Yes, because the law firm's violation of the conflict of interest rule gives rise to civil liability per se.

C) Yes, because the law firm is breaching its fiduciary duty to the small soda company. Answer choice C is correct. A lawyer is subject to an injunction to the same extent provided by generally applicable law, and an injunction is appropriate to prevent a breach of a fiduciary duty. Here, the law firm's ongoing representation of the company was materially limited by the firm's associate's advice to the conglomerate and the possibility that the company's confidential information might "leak" to the conglomerate. The conflict could most likely be consented to by the company, but because the company refused to consent, the law firm should have immediately sought to withdraw from representing the conglomerate. Because it did not, it breached its fiduciary duty to the company. Answer choice A is incorrect. Because a lawyer is subject to an injunction to the same extent provided by generally applicable law, injunctive relief may be available against a lawyer for reasons other than professional negligence. Therefore, it is incorrect to state that the injunction will be denied because the law firm is not committing an act of professional negligence. Since an injunction is appropriate to prevent a breach of a fiduciary duty, this injunction is likely to be granted, even without a showing of professional negligence. Answer choice B is incorrect. One consequence of violating a rule of professional conduct could be professional discipline. However, professional discipline is not the exclusive remedy for conduct that violates a professional rule. To the extent that conduct also satisfies the elements in a cause of action for negligence, breach of fiduciary duty, breach of contract, or the violation of some other common-law obligation, a lawyer may be subject to that liability as well. Answer choice D is incorrect. Violation of a rule of professional conduct does not always give rise to civil liability, although the same facts underlying a violation of the rules of professional conduct may also constitute a civil wrong. It is incorrect, however, to state that a violation of a professional rule always results in civil liability.

An attorney represented a client who the attorney reasonably believed had a severe mental disorder affecting her ability to make rational personal and business decisions. Rather than consulting a physician or other mental healthcare professional about how to deal with his client, the attorney sought the advice of another attorney, who specialized in representing other attorneys in malpractice actions, about his professional obligations to his client. During his discussions with the other attorney, the attorney disclosed information that the client had revealed to him. Was the attorney's disclosure of information learned from his client proper? A) No, because the disclosed information was acquired during the course of the attorney's representation of the client. B) No, because the lawyer failed to consult a physician or mental healthcare professional first. C) Yes, because the lawyer may disclose this information to obtain advice regarding his professional obligations. D) Yes, because the lawyer for a client under a disability must disclose information necessary for protective action.

C) Yes, because the lawyer may disclose this information to obtain advice regarding his professional obligations. Answer choice C is correct. An attorney's duty of confidentiality does not prohibit an attorney from obtaining confidential legal advice about the attorney's responsibility to comply with the Model Rules of Professional Conduct. Generally, disclosing information to obtain such advice would be impliedly authorized for the attorney to carry out the representation. Even if not impliedly authorized, the Model Rules of Professional Conduct allow such disclosure. Answer choice A is incorrect because even if the information was acquired in the course of the representation, the attorney may still disclose the information to obtain confidential legal advice. Answer choice B is incorrect because the attorney does not have such an obligation before securing confidential legal advice on how to proceed in accordance with the Model Rules of Professional Conduct. Answer choice D is incorrect because the attorney does not have to reveal information about a client under a disability in order to take protective measures for that client. However, the Rules permit the attorney to make such disclosures to the extent reasonably necessary to protect the client's interests.

A defendant was charged with involuntary manslaughter after he allegedly murdered his wife upon discovering that she was having an affair with his best friend. Two days after the charges were filed against the defendant, the prosecutor assigned to the case contacted the defendant in an attempt to reach a plea bargain. The prosecutor did not advise the defendant of his right to and the procedure for obtaining counsel, nor did she provide him with any time to hire an attorney. The prosecutor encouraged the defendant to waive his right to a preliminary hearing and accept a guilty plea in exchange for a lighter sentence. The defendant waived his right to counsel, and the defendant and prosecutor entered into a plea agreement. The plea agreement was then submitted to a judge for approval. Upon determining that the defendant had entered the plea of guilty knowingly, voluntarily and intelligently, the judge accepted the plea agreement. Is the prosecutor subject to discipline? A) No, because the defendant waived his right to counsel. B) No, because the plea agreement was accepted by the judge. C) Yes, because the prosecutor did not make reasonable efforts to assure the defendant was advised of his right to and the procedure for obtaining counsel. D) Yes, because the prosecutor encouraged the defendant to accept a guilty plea in exchange for a lighter sentence.

C) Yes, because the prosecutor did not make reasonable efforts to assure the defendant was advised of his right to and the procedure for obtaining counsel. Answer choice C is correct. A prosecutor must make reasonable efforts to assure defendants are advised of their right to and the procedure for obtaining counsel and have been given reasonable opportunity to obtain counsel. Here, the prosecutor contacted the defendant only two days after he was charged with second-degree murder and did not advise him of his right to and the procedure for obtaining counsel or provide him with any time to hire counsel. Thus, the prosecutor did not make reasonable efforts to assure that the defendant was advised of his right to obtain counsel, and she did not give him any opportunity to obtain counsel. For this reason, she is subject to discipline. Answer choice A is incorrect. The fact that the defendant waived his right to counsel does not release the prosecutor of liability under the Model Rules. Answer choice B is incorrect. Although the judge accepted the plea agreement, the prosecutor was still under an obligation to make reasonable efforts to assure that the defendant was advised of his right to and the procedure for obtaining counsel. Answer choice D is incorrect. The goal of plea bargaining is to encourage a criminal defendant to plead guilty, and in exchange for pleading guilty, the criminal defendant may receive a lighter sentence or have the charges reduced. Thus, it was not improper for the prosecutor to encourage the defendant to accept a guilty plea in exchange for a lighter sentence.

An attorney who maintained a solo law practice entered into substantive discussions with a law firm about joining the firm. As part of these discussions, the attorney revealed the identities of her clients, their status as current or former clients, and the matters for which she represented them. The purpose of these revelations, which did not compromise the attorney-client privilege or otherwise prejudice the clients, was to detect conflicts of interest that would arise from the attorney joining the firm. The attorney did not seek the informed consent of her clients before making these revelations. Due to an inability to resolve a potential conflict of interest, the attorney did not join the law firm. Were the attorney's revelations proper? A) No, because the attorney did not seek the informed consent of her clients before making these revelations. B) No, because the attorney did not join the law firm. C) Yes, because the purpose of the revelations was to detect conflicts of interest that would arise from the attorney joining the law firm. D) Yes, because the duty of confidentiality does not apply to disclosures to another attorney of information relating to the representation of a client.

C) Yes, because the purpose of the revelations was to detect conflicts of interest that would arise from the attorney joining the law firm. Answer choice C is correct. Although an attorney has a duty of confidentiality with respect to information relating to the representation of a client, there is an exception for such disclosures that are made to detect and resolve conflicts of interest that occur when a lawyer is engaged in substantive discussions about changing employment or there is substantive discussion about a change in the composition or ownership of a firm. Answer choice A is incorrect because an attorney seeking to associate with a firm may make limited disclosure of confidential information without first seeking the informed consent of her clients unless the revelations would compromise the attorney-client privilege or otherwise prejudice the clients. Because these limitations did not apply here, the attorney did not need the informed consent of her clients before making the disclosures. Answer choice B is incorrect because the "detection of conflicts of interest" exception to the duty of confidentiality applies even when the attorney does not change employment, provided the disclosures are not made until substantive discussions regarding the new relationship have occurred. Answer choice D is incorrect because there is no blanket exception to the duty of confidentiality for disclosures regarding a client made by an attorney to another attorney. In fact, most such disclosures would violate this duty.

A woman sustained serious injuries while parachuting when her equipment malfunctioned, and her parachute did not open properly. In anticipation of litigation, the parachuting company interviewed a man who had been waiting for the woman to land. The man was the only witness to the accident. After the man gave an account of what had happened, the company's attorney prepared a written statement recording the man's account of the accident, and the man signed it. Two years later, the woman filed a negligence suit against the parachuting company. The woman's attorney interviewed the man who had witnessed the accident, but he could no longer remember the details of the accident. The attorney served a discovery request upon the parachuting company to obtain any documents concerning the man's former statements regarding the accident. The parachuting company's attorney refused to produce the written statement, claiming that it was protected by the work-product doctrine and the attorney-client privilege. Is the court likely to require the parachuting company's attorney to produce the written statement? A) No, because the attorney-client privilege protects the written statement. B) No, because the work-product doctrine protects the written statement. C) Yes, because the woman has a substantial need for the written statement and cannot obtain a substantial equivalent by any other means. D) Yes, because the written statement does not contain statements made to the parachuting company's attorney by a client or a client representative.

C) Yes, because the woman has a substantial need for the written statement and cannot obtain a substantial equivalent by any other means. Answer choice C is correct. The parachuting company's attorney's written statement is work product, which consists of tangible material prepared by a lawyer for current or reasonably anticipated litigation and is ordinarily protected from discovery. It is discoverable here, however, because (i) the woman has a substantial need for the material and (ii) is unable without undue hardship to obtain the substantial equivalent by other means since the man could no longer remember the details of the accident and he was the only witness to the accident. For this reason, answer choice B is incorrect. Answer choice A is incorrect. The written statement of the man's account of the accident is not privileged. The attorney-client privilege protects only communications involving "privileged persons," including a lawyer and the lawyer's client. Here, the witness interviewed (the man) was not the parachuting company's attorney's client. Answer choice D is incorrect. Although the parachuting company's attorney's written statement is not a privileged communication, it is work product, which ordinarily would not be discoverable unless an exception applies.

Which of the following is the key to the existence of a lawyer-client relationship? A) Payment of a fee by the client B) Entering into an agreement between the lawyer and the client C) Explicit acceptance of the client by the lawyer D) A reasonable belief by the client that a lawyer-client relationship exists

D) A reasonable belief by the client that a lawyer-client relationship exists

Which of the following statements regarding a lawyer's responsibility for misconduct by others is FALSE? A) A partner in a firm must ensure that the firm has in place measures to prevent violation of the rules of professional conduct. B) A supervisory lawyer who learns of a violation of the rules of professional conduct by a subordinate lawyer after it has occurred has an obligation to take remedial action. C) A subordinate lawyer can follow a supervisory lawyer's reasonable resolution of an ethical issue without violation of rules of professional conduct. D) A supervisory lawyer is not responsible for the conduct of non-professional employees.

D) A supervisory lawyer is not responsible for the conduct of non-professional employees.

Which of the following statements regarding impartiality is FALSE? A) A judge must make reasonable efforts to ensure that her staff does not violate the rule against ex parte communication. B) A judge generally must not publicly comment on a case pending in another jurisdiction. C) A judge must not conduct an independent investigation. D) The rule against ex parte communication generally does not apply to a judge.

D) The rule against ex parte communication generally does not apply to a judge.

An attorney is a candidate for a judicial office that has been occupied by an incumbent for six years. The attorney has conducted a thorough investigation of the incumbent's personal and professional life. Assuming that all factual statements are accurate, which of the following statements is improper for the attorney to make during the campaign? A) "The incumbent has been reversed by the appellate courts more than any other judge in the state during the preceding two years." B) "The incumbent was publicly censured by the state Judicial Qualification Commission on one occasion for his overbearing conduct in court." C) "The incumbent was given a poor rating for judicial temperament in a county bar association poll." D) "During the previous year, the average sentence in armed robbery cases tried in the incumbent's court was 3.5 years, and in murder cases was 8.2 years. If I am elected, I won't be soft on crime."

D) "During the previous year, the average sentence in armed robbery cases tried in the incumbent's court was 3.5 years, and in murder cases was 8.2 years. If I am elected, I won't be soft on crime." Answer choice D is correct. The statement is improper, as it appears to be a pledge or commitment by the attorney, as candidate for judicial office, to take a position on cases that are likely to come before the court, in violation of CJC Rule 4.1. Answer choices A, B, and C are all proper, as they are true factual statements.

Which of the following statements regarding a lawyer's political activity is TRUE? A) A lawyer running for judge is not subject to the judicial ethics rules until the lawyer wins the election. B) A lawyer must not make a disparaging statement that the lawyer knows to be true about the qualifications of a person who is seeking appointment as a judge. C) A lawyer is prohibited from soliciting contributions for her own campaign while actively engaging in litigation. D) A lawyer is prohibited from making contributions in exchange for favor in a lawsuit.

D) A lawyer is prohibited from making contributions in exchange for favor in a lawsuit.

In which of the following circumstances may a lawyer represent opposing parties in the same lawsuit? A) Both parties give their informed consent. B) The lawyer believes that his representation of each party will not be adversely affected. C) There is no risk of revealing confidential information. D) A lawyer may never represent opposing parties in the same lawsuit.

D) A lawyer may never represent opposing parties in the same lawsuit.

Alpha and Beta are members of the bar in the same community but have never practiced together. Beta is a candidate in a contested election for judicial office. Beta is opposed by Delta, another lawyer in the community. Alpha believes Beta is better qualified than Delta for the judiciary and is supporting Beta's candidacy. Which of the following would be proper for Alpha to do? A) Solicit public endorsements for Beta's candidacy by other attorneys in the community who know Beta, including those who are likely to appear before Beta if Beta becomes a judge. B) Solicit contributions to Beta's campaign committee from other attorneys in the community, including those who are likely to appear before Beta if Beta becomes a judge. C) Publicly oppose the candidacy of Delta. D) All of the above would be proper.

D) All of the above would be proper. Answer choice D is correct. Answer choice A is proper, as a lawyer may summon endorsements for a candidate for judicial office. Answer choice B is proper, as lawyers may contribute to a judicial office candidate's campaign through a campaign committee authorized by CJC Rule 4.4. Answer choice C is proper, as a lawyer may publicly oppose a candidate for judicial office. Therefore, answer choice D is the best choice, as it includes all of the proper courses of action.

An attorney represents a well-known contractor before a state administrative agency. The agency has ordered the client to show cause why the client's license as a contractor should not be revoked for violation of agency regulations. In a newspaper interview prior to the administrative hearing, the attorney made a variety of statements. Which of these statements, if any, would be improper? A) "My client denies the charge made by the agency that she engaged in conduct constituting grounds for revocation of her license as a contractor." B) "The next step in the administrative process is the administrative hearing; if the agency is successful, we will appeal, and the agency still cannot revoke my client's license until a court affirms the finding for the agency." C) "My client needs witnesses who are aware of the incidents that are the subject of the hearing." D) All of these statements are proper

D) All of these statements are proper Answer choice D is correct. Statement A is proper, as a public statement of the defense involved is permissible under MRPC 3.6(b)(1). Statement B is proper, as a statement of the schedule of the litigation is permissible under MRPC 3.6(b)(4). Statement C is also proper, as a request for assistance in obtaining evidence and information necessary to the case is permissible under MRPC 3.6(b)(5). Therefore, answer choice D is correct, as it includes all three statements.

Attorney Alpha, a member of the bar, placed a printed flyer in the booth of each artist exhibiting works at a county fair. The face of the flyer contained the following information: "I, Alpha, am an attorney, with offices in 800 Bank Building, telephone (555) 555-5555. I have a J.D. degree from State Law School and an M.A. degree in fine arts from State University. My practice includes representing artists in negotiating contracts between artists and dealers and protecting artists' interests. You can find me in the van parked at the fair entrance." All factual information on the face of the flyer was correct. There was a retainer agreement on the back of the flyer. At the entrance to the fair, Alpha parked a van with a sign that read "Alpha - Attorney at Law - 800 Bank Building." For which, if any, of the following is Alpha subject to discipline? A) Placing copies of the flyer in the booth of each artist. B) Including a retainer agreement on the back of the flyer. C) Parking the van with the sign on it at the fair entrance. D) Alpha is not subject to discipline for any of the above.

D) Alpha is not subject to discipline for any of the above. Answer choice D is correct because answer choices A, B, and C are all proper. Answer choice A is proper because a lawyer is permitted to advertise his services through written communication, so long as the communication regarding such services is not false or misleading in violation of MRPC 7.1, or in violation of the rules against solicitation of clients under MRPC 7.3. MRPC 7.2(a). In addition, any advertisement must include the name and office address of at least one lawyer or law firm responsible for its content. MRPC 7.2(c). Here, the flyer is a written communication that is not false or misleading, and it included the attorney's name and office address. The attorney was not engaging in solicitation, because unless a prospective client has made known to the lawyer a desire not to be solicited, the lawyer may target the client with written communications so long as the communications satisfy the Model Rules with regard to advertising and communications by lawyers. Answer choice B is proper, as a lawyer may use a preprinted retainer form. Answer choice C is proper, as there is no prohibition against the use of a billboard or outdoor sign as advertising, so long as it meets the requirements of MRPC 7.2.

An attorney represents a plaintiff in a personal injury action. An eyewitness to the accident lives about 500 miles distant from the city where the case will be tried. The attorney interviewed the witness and determined that the witness's testimony would be favorable for the client. The witness asked the attorney to pay him, in addition to the statutory witness fees while attending the trial, certain other expenses. If the attorney agrees to pay the witness the following, for which, if any, is the attorney subject to discipline? A) Reimbursement for lodging expenses while attending the trial. B) Reimbursement for lost wages while present at the trial. C) Reimbursement for personal meals while attending the trial. D) An amount equal to 5% of any recovery in the matter.

D) An amount equal to 5% of any recovery in the matter. Answer choice D is correct. Paying a witness a portion of the amount recovered by the plaintiff would constitute an inducement to a witness that is prohibited by law, pursuant to MRPC 3.4(b). Answer choices A, B, and C all include reasonable expenses that a lawyer may reimburse to a non-expert witness in return for testifying.

Which of the following statements regarding a contingent fee for a lawyer's services is FALSE? A) The contingency fee arrangement must be in writing. B) The contingency fee arrangement must be signed by the client. C) The contingency fee arrangement must include the calculation methodology for the fee and expenses. D) Contingency fee arrangements are permitted in all cases.

D) Contingency fee arrangements are permitted in all cases.

Which of the following does NOT constitute misconduct for which a lawyer can be subject to discipline? A) Violation of a rule of professional conduct B) A conviction for bribing a juror C) Fraud committed in a personal transaction D) Conviction for minor traffic offense

D) Conviction for minor traffic offense

An attorney worked for a state agency that set hotel rates in various cities within the state. Hotel establishments were required to submit information about their expenditures and management to the agency, which the agency was prohibited from disclosing to the public. Two years ago, the attorney left the state agency and took a job with a private law firm. One of the partners in the law firm intends to accept representation of a peer-to-peer vacation rental company that is in direct competition with hotel establishments in one of the cities. The attorney who had worked at the state agency was aware that the vacation rental company could use information he remembered that the competing hotels had reported to the agency about their expenditures and management practices to the disadvantage of those hotels. What must the partner or the attorney do to permit the law firm to represent the vacation rental company? A) Obtain the informed consent, confirmed in writing, of the hotel establishments to the possible adverse use of the confidential information they provided to the state agency. B) Obtain the informed consent, confirmed in writing, of the state agency to the representation of the vacation rental company by the attorney's firm. C) Provide written notice to the state agency of the attorney's association with the law firm and the firm's representation of the vacation rental company. D) Timely screen the attorney from participation in the representation of the vacation rental company and ensure that the attorney receives no fee from the matter.

D) Timely screen the attorney from participation in the representation of the vacation rental company and ensure that the attorney receives no fee from the matter. Answer choice D is correct. Except as the law may otherwise expressly permit, a government lawyer who acquires information that the lawyer knows is confidential government information about a person may not later represent a private client whose interests are adverse to that person in a matter in which the information could be used to that person's material disadvantage. Confidential government information is information that has been obtained under governmental authority and which the government is prohibited by law from disclosing to the public. In this case, the former government attorney knows that the vacation rental company could use confidential government information he remembered about the competing hotels to the disadvantage of those hotels. However, a firm with which a former government lawyer is associated may undertake or continue representation in a matter in which that lawyer is disqualified due to possession of confidential government information if the disqualified lawyer is timely screened from any participation in the matter and gets no part of the fee from the representation. Therefore, the former government attorney's law firm may represent the vacation rental company if the attorney is screened and apportioned no fee. Answer choice A is incorrect. The law firm with which the attorney is associated may represent the vacation rental company if the attorney is screened and apportioned no fee, but informed consent will not cure this conflict, regardless of whether the consent is in writing. Answer choice B is incorrect because the government agency's consent is not required. Model Rule 1.11(c) protects the individuals and companies to whom confidential government information pertains. It is not aimed at protecting the government agency itself. Answer choice C is incorrect. A firm with which a former government lawyer is associated may undertake or continue representation in a matter in which that lawyer is disqualified due to possession of confidential government information if the disqualified lawyer is timely screened from any participation in the matter and gets no part of the fee from the representation. Written notice of the attorney's association with the law firm is not required, and would not excuse the firm from the requirements to screen the former government attorney and ensure that he gets no part of the fee.

An attorney is defending a client who has been indicted for burglary. During an interview, the client stated to the attorney that before he had consulted her, he had committed perjury while testifying before the grand jury that indicted him. The attorney is subject to discipline if she: A) Continues to represent the client. B) Continues to represent the client unless he admits his perjury. C) Does not inform the authorities of the perjury. D) Informs the authorities of the perjury.

D) Informs the authorities of the perjury. Answer choice D is correct. The attorney learned the information about the perjury during the course of representing the client; therefore it is protected by the ethical duty of confidentiality under MRPC 1.6, and the attorney will be subject to discipline if she informs the authorities of the perjury. Answer choice A is not correct, as the attorney may continue to represent the client even though she knows of the client's past perjury. Only if the client attempts to commit perjury as part of evidence being offered by the attorney in the future would the attorney have a potential obligation to withdraw from representation. Answer choice B is not correct, as the perjury here was committed in the past, before the attorney was involved in the case. The client does not have to reveal the past perjury for the attorney to continue representation. Only if the client attempts to commit perjury as part of evidence being offered by the attorney in the future would the attorney have a potential obligation to withdraw. Answer choice C is not correct, as the attorney is obligated not to inform the authorities because of the ethical duty of confidentiality under MRPC 1.6.

Which of the following is NOT a source of professional responsibility law tested on the MPRE? A) Constitutional law issues B) Evidentiary rules C) ABA Model Code of Judicial Conduct D) Model Penal Code

D) Model Penal Code

Attorney Alpha, a sole practitioner, recently suffered a heart attack and was advised that she could not return to work for six months. Alpha delivered all of her clients' files to Attorney Beta, who is also a sole practitioner. Beta agreed to review each client's file promptly, take any action necessary to protect each client's interests, and treat the information in the files as confidential. Alpha then wrote her clients, advising each client that the client's file had been delivered to Beta for review and for any action necessary to protect the client's interest, and that the client was free to select another lawyer. Alpha knows that Beta is a competent attorney. Beta did not accept the file of any person whose interests were, or could be, adverse to the interests of any of Beta's own clients. Was it proper for Alpha to deliver the files to Beta for review? A) Yes, because Alpha knows that Beta is competent to protect the clients' interests. B) Yes, because Beta agreed to treat the information in the files as confidential. C) Yes, because given her medical condition, Alpha's delivery of the files was necessary to protect the clients' interests. D) No, because Alpha did not obtain the prior consent of each client whose file was delivered to Beta.

D) No, because Alpha did not obtain the prior consent of each client whose file was delivered to Beta. Answer choice D is correct. In general, MRPC 1.6(a) prohibits a lawyer from revealing information relating to the representation of a client without client consent. A lawyer practicing in a law firm is impliedly authorized to disclose client confidences to other lawyers within the law firm, see MRPC 1.6, cmt. 5, but not to lawyers outside the firm. There is an exception when a lawyer seeks legal advice from another lawyer about how to comply with the ethics rules, but that exception does not apply here. There is also an exception that allows sole practitioners to create a plan in advance of their death or incapacity that will protect their clients' interests in the event that the sole practitioner is no longer available to handle their own cases. But here, Attorney A had no plan in advance of her illness that would automatically share her case files with Attorney B. She only decided to hand her cases over to Attorney B after she fell ill, so the sole practitioner contingency plan exception does not apply. Therefore, the lawyer needed client consent. Answer choice A is incorrect because even if Beta is competent, Alpha needs a client's consent to engage Beta in the representation and to disclose the client's confidences to Beta. The disclosure to Beta was not "impliedly authorized in order to carry out the representation," because a client would not anticipate that his or her lawyer, who is a sole practitioner, would transfer the client's matters to a different lawyer. Answer choice B is incorrect because even if Beta agrees to keep the client's information confidential, the client's consent is needed to engage Beta and to share the client's confidences with Beta. Subject to exception, MRPC 1.6(a) generally prohibits a lawyer from revealing information relating to the representation of a client without client consent. No exception applies here. Answer choice C is incorrect. If Alpha had first secured the clients' consent, it would have been permissible to deliver the files to Beta. But, absent client consent, the disclosure of clients' confidences to Beta was prohibited by MRPC 1.6(a).

An attorney is a well-known tax lawyer and author. During congressional hearings on tax reform, the attorney testified to her personal belief and expert opinion on the pending reform package. She failed to disclose in her testimony that she was being compensated by a private client for her appearance. In her testimony, the attorney took the position favored by her client, but the position was also one that the attorney believed was in the public interest. Was it proper for the attorney to present this testimony without identifying her private client? A) Yes, because the attorney believed that the position she advocated was in the public interest. B) Yes, because Congress is interested in the content of the testimony and not who is paying the witness. C) No, because a lawyer may not accept a fee for trying to influence legislative action. D) No, because a lawyer who appears in a legislative hearing should identify the capacity in which the lawyer appears.

D) No, because a lawyer who appears in a legislative hearing should identify the capacity in which the lawyer appears. Answer choice D is correct. MRPC 3.9 requires a lawyer representing a client before a legislative body to disclose that the appearance is in a representative capacity. Answer choice A is incorrect, as the attorney's conduct was improper because MRPC 3.9 requires a lawyer representing a client before a legislative body to disclose that the appearance is in a representative capacity. Answer choice B is incorrect because the congressional members at the hearing would be very interested in knowing whether a witness is possibly biased in her testimony; therefore, MRPC 3.9 requires a lawyer appearing in a representative capacity to disclose that fact to the legislative body. Answer choice C is incorrect; lawyers frequently serve as paid lobbyists. This activity is permissible so long as the lawyer-lobbyist complies with applicable rules, such as the requirement to disclose her representative capacity when appearing before a legislative body.

A developer wanted to acquire land on which to build a shopping mall. The developer contacted an attorney to represent the developer in buying the land from a farmer. To prevent knowledge of the developer's plans for the land from driving up its price, the developer instructed the attorney to disclose neither the developer's plan for the land nor the identity of the purchaser. The attorney complied with the developer's instructions. The farmer did not inquire as to whether the attorney was purchasing the land for herself or someone else or the use that the buyer intended to make of the land. Is the attorney subject to discipline? A) Yes, because the omission of a relevant fact can be the equivalent of an affirmative false statement. B) Yes, because the attorney failed to disclose to the farmer the identity of the purchaser of the land and the reason for its acquisition. C) No, because the attorney was complying with the developer's instructions. D) No, because an undisclosed principal is generally not a material fact that must be disclosed to a third person.

D) No, because an undisclosed principal is generally not a material fact that must be disclosed to a third person. Answer choice D is correct. While a lawyer does not generally have a duty to inform an opposing party of relevant facts, the lawyer is not permitted to misrepresent facts. Under generally accepted conventions in negotiation, certain types of statements, such as the existence of an undisclosed principal, are not generally taken as statements of material fact. Therefore, since her nondisclosure does not rise to the level of fraud, the attorney is not subject to discipline on these facts. Answer choice A is incorrect because, while an omission of a relevant fact can be the equivalent of an affirmative false statement, under generally accepted conventions in negotiation, certain types of statements are not generally taken as statements of material fact. Among such statements is the existence of an undisclosed principal. Answer choice B is incorrect because, in representing a client, a lawyer does not generally have a duty to inform an opposing party of relevant facts. Answer choice C is incorrect because, while a lawyer is generally required to adhere to a client's instructions in representing the client in negotiations with a third party, a lawyer must not knowingly make a false statement of a material fact or law to a third person, regardless of the client's instructions.

An attorney was engaged to create a trust for a client. The attorney had taken several advanced tax law classes and had acquired an LL.M. in taxation. The attorney did not advertise her advanced knowledge of the tax law because she had not been certified as a tax specialist by a state-approved or ABA-accredited organization, and the client was not aware that the attorney had any training in tax law. The trust created by the attorney violated tax provisions that a lawyer with her special knowledge of the tax law should have been able to avoid. As a consequence, the client incurred a tax liability that he otherwise would not have incurred. The client has brought a malpractice claim against the attorney based on negligence. May the attorney be subject to civil liability to the client? A) Yes, because the attorney could not disclaim her possession of specialized knowledge of the tax law. B) Yes, because the attorney failed to exercise the special skill in tax law that she possessed. C) No, because the client was not aware and therefore did not rely on the attorney's special tax training. D) No, because the attorney did not hold herself out as a tax specialist.

D) No, because the attorney did not hold herself out as a tax specialist. Answer choice B is correct. In determining a lawyer's duty of care, the lawyer's special experience with or knowledge of the law is taken into account, regardless of whether the client knows that the lawyer has such knowledge. Because the attorney should have been able to avoid these adverse tax consequences by using her special knowledge of the tax law, the attorney has breached her duty of care and may be liable for the tax liability incurred by the client. Answer choice A is incorrect because, at least in some circumstances, a lawyer may disclaim competency in excess of the ordinary or the possession of specialized knowledge. However, no disclaimer was made here. Answer choice C is incorrect because the lawyer's special experience with or knowledge of the law is taken into account when determining her duty of care in a malpractice claim, regardless of whether the client knows that the lawyer has such knowledge. Answer choice D is incorrect because, although the attorney properly did not hold herself as a tax specialist, the attorney was required to use the specialized knowledge that she possessed, even if the client did not know she had such knowledge.

Several attorneys decided to host a dinner honoring a judge who recently retired and was not subject to recall as a judge. Each of the attorneys had successfully tried at least one case before the judge. One of the attorneys, who was a sole practitioner, collected money from the other attorneys, which he deposited in his office account that contained the attorney's personal funds, without notating the source of the money or its purpose. The attorney expended the money collected from the other attorneys as well as his own money in connection with the dinner. Is the attorney subject to discipline for his handling of the money received from the other attorneys? A) Yes, because the attorney failed to maintain adequate records of the money received from the other attorneys. B) Yes, because the attorney commingled the money received from the other attorneys with his own money. C) No, because the rule requiring a lawyer to safeguard money does not apply to money received from another lawyer. D) No, because the attorney did not receive the money in connection with the representation of a client.

D) No, because the attorney did not receive the money in connection with the representation of a client. Answer choice D is correct. In general, the same rules regarding a client's property in the possession of a lawyer apply to the property of a third party received by a lawyer in connection with representation of a client. Here, however, the attorney did not receive the money from the other attorneys in connection with the representation of a client. Consequently, the prohibition on commingling as well as the recordkeeping, notice, delivery, and accounting requirements that apply to a third person's property being held by the lawyer in connection with the representation of client do not apply. Answer choice A is incorrect because, although a lawyer is required to maintain adequate records with regard to the property of a third person that is held by the lawyer in connection with the representation of a client, this requirement does not apply in this case since the money in question was not received by the attorney in connection with the representation of a client. Answer choice B is incorrect because, although a lawyer generally must not commingle a client's funds that are in the lawyer's possession in connection with a representation with the lawyer's own funds, this prohibition does not apply in this case since the money in question was not received by the attorney in connection with the representation of a client. Answer choice C is incorrect because property received by a lawyer from another lawyer is not exempt from the duty to safeguard the property belonging to others that is received in connection with the representation of a client.

An attorney met with a client on a Monday morning about the client's pending divorce. The client told the attorney that if his wife did not agree to a property settlement by the end of the week, he "was going to kill [her]." The attorney thought his client was simply being dramatic but was concerned because he knew that his client already owned a gun and had made similar statements before. The attorney worried that the client might be serious about the threat and decided that, if the case had not settled by Friday, he would call the client's wife and warn her. The client shot his wife on Thursday. Is the attorney subject to discipline for not disclosing the client's threat to kill his wife? A) Yes, because the attorney reasonably believed that the client was going to harm his wife if the case did not settle. B) Yes, because the attorney owed a duty of fairness to the wife as the other party in the civil action. C) No, because the threat was not imminent since the client claimed he would kill his wife at the end of the week if a settlement was not reached. D) No, because the attorney had no obligation to disclose information he thought was necessary to prevent future bodily harm.

D) No, because the attorney had no obligation to disclose information he thought was necessary to prevent future bodily harm. Answer choice D is correct. An attorney may reveal confidential information concerning the representation of a client to the extent the attorney reasonably believes disclosure is necessary to prevent reasonably certain death or bodily harm. The attorney is not obligated to do so, however. Here, the attorney thought his client was simply being dramatic, and the client had not taken steps to harm his wife despite making similar statements in the past. Answer choice A is incorrect because the fact that the belief was reasonable does not require the attorney to disclose the information: it simply permits him to do so. Answer choice B is incorrect because although an attorney does owe a duty of fairness to an opposing party, this duty does not require the attorney to disclose the information learned as a consequence of representing his client. Answer choice C is incorrect because the attorney was not obligated to disclose the information even if the threat was imminent. He would have the option of revealing the information, however.

A client hired an attorney to represent him in a contractual dispute. Because the client had just relocated to the state and did not have a bank account, he and the attorney agreed that the client would give the attorney a number of bond certificates as a retainer, and the attorney would return any unearned bond certificates at the conclusion of the representation. The attorney placed all of the bonds in a safe-deposit box at the bank where he maintained his client trust account. At the end of the trial, the attorney and the client disputed the number of bonds owed to the attorney, although both the attorney and the client agreed that some of the bonds belonged to the attorney while many belonged to the client. Because they could not agree as to the exact amount owed to each, however, the attorney retained all of the bonds in the safe-deposit box pending resolution of the dispute. Because the attorney admitted that many of the bonds were in fact owed to the client, the client was upset that the attorney retained all of the bonds. Is the attorney subject to discipline for his actions regarding the bonds? A) Yes, because the attorney retained all of the bonds in the safe-deposit box. B) Yes, because the attorney did not return any of the bonds to the client. C) No, because the rules do not govern the payment of bonds in satisfaction of a debt to an attorney. D) No, because the attorney maintained possession of all of the bonds.

D) No, because the attorney maintained possession of all of the bonds. Answer choice D is correct. If, as a part of the representation of a client, an attorney is in possession of property in which two or more persons (one of whom may be the attorney) claim interests, then the property must be kept separate by the attorney until the dispute is resolved. The attorney is required to promptly distribute all portions of the property as to which the interests are not in dispute. Unlike a cash retainer, bond certificates aren't themselves valuable, but merely represent intangible rights to underlying capital. If the attorney had a cash retainer instead of the bonds, each unit of the cash would be treated as interchangeable, and as long as the attorney and client could agree to some sum to which the client was entitled, the attorney would have to return that undisputed sum. Here, although the attorney and the client agreed that the client was entitled to some of the bonds, instead of creating an undisputed interest in some of the bonds, this actually creates a disputed interest in each bond, unless and until the parties agree which bonds the client owns. Therefore, ownership of all of the bonds was in dispute, and the attorney acted properly in retaining them all. Answer choice A is incorrect because the attorney was required to keep all of the bonds, as there was no resolution to the amount of bonds owed to each, even if the attorney acknowledged that some unresolved amount was owed to the client. As there was no amount of interest not in dispute, the attorney was required to maintain all of the bonds. Answer choice B is incorrect because the attorney should have retained the disputed property, which in this case amounts to all of the bonds, as there has been no resolution regarding the specific amount owed to the client. Answer choice C is incorrect because the rules do not distinguish between this and other forms of payment in satisfaction of the debt owed to an attorney.

Which of the following statements regarding the work-product doctrine is FALSE? A) The work-product doctrine protects documents prepared for use in connection with a client's case. B) Work product is generally not subject to discovery. C) A court may order a lawyer to disclose work product if there is a substantial need for the information and no other means to gather the information without undue hardship. D) Work product includes everything subject to the duty of confidentiality.

D) Work product includes everything subject to the duty of confidentiality

An attorney's standard retainer contract in divorce cases provides for the payment of a fee of one-third of the amount of alimony or property settlement secured by the attorney. The attorney declines to represent clients who do not agree to this arrangement. Is the attorney's standard retainer contract proper? A) Yes, because clients often prefer to pay a lawyer a fee based on the outcome of the case. B) Yes, if a fee of one-third is not excessive. C) No, because a lawyer may not acquire a proprietary interest in a cause of action. D) No, because the fee is contingent.

D) No, because the fee is contingent. Answer choice D is correct. Under MRPC 1.5(d)(1), a lawyer may not charge a fee in a domestic relations case that is contingent on obtaining a divorce or on the amount of support recovered. Answer choice A is not correct, as client preferences do not override the Model Rules, which prohibit a contingent fee under these circumstances. Answer choice B is not correct, as whether the fee is excessive is not relevant, since a contingent fee is prohibited here by the Model Rules. Answer choice C is not correct because the proprietary interest issue is irrelevant. MRPC 1.5(d)(1) prohibits charging a contingent fee under these circumstances.

Four years ago, a judge in a state court of general jurisdiction heard a civil case in which the plaintiff prevailed and secured a judgment for $50,000, which was sustained on appeal. Since then, the judge has resigned from the bench and returned to private practice. The defendant has filed suit to enjoin enforcement of the judgment on the grounds of extrinsic fraud in its procurement. The plaintiff has now asked the former judge to represent her in defending the suit to enjoin enforcement. Is it proper for the former judge to accept the representation of the plaintiff in this matter? A) Yes, because the former judge would be upholding the decision of the court. B) Yes, if the plaintiff's conduct of the first trial will not be in issue. C) No, unless the former judge believes the present suit is brought in bad faith. D) No, because the former judge had acted in a judicial capacity on the merits of the original case.

D) No, because the former judge had acted in a judicial capacity on the merits of the original case. Answer choice D is correct. Under MRPC 1.12(a), a lawyer may not represent anyone in connection with a matter in which the lawyer participated personally and substantially as a judge, unless all parties to the proceeding give informed, written consent. MRPC 1.12(a). Here, the former judge acted as a judge in the original case on the merits. Accordingly, it would not be proper for him to now accept representation of the plaintiff. Answer choice A is not correct, as MRPC 1.12(a) prohibits the former judge from representing the plaintiff because the former judge previously participated personally and substantially as a judge in the original case and all parties have not given informed, written consent to the representation. Answer choice B is not correct for the same reason. Answer choice C is not correct, because even if the present suit was brought in bad faith, it would not permit the former judge to avoid the prohibition of MRPC 1.12(a).

Prior to being appointed to the court, a federal court judge was employed as an immigration attorney. While so employed, he delivered speeches to various legal organizations and wrote articles in legal publications in which he advocated a lenient enforcement of the immigration laws. The judge has been assigned to preside over the criminal prosecution of an alien for violation of these laws. The judge is not related personally or professional to the parties or the attorneys in this case and has not previously been involved in the case in any way. The prosecution has requested that the judge disqualify himself from hearing this case because of the public stance he took on the enforcement of the immigration laws while employed as an immigration attorney. Must the judge disqualify himself? A) Yes, because the judge has taken a public position on the enforcement of the immigration laws. B) Yes, because the judge has advocated for a lenient enforcement of the immigration laws. C) No, because the judge is not related personally or professional to the parties or the attorneys in this case and has not previously been involved in the case in any way. D) No, because the impartiality of the judge on this matter may not reasonably be questioned.

D) No, because the impartiality of the judge on this matter may not reasonably be questioned. Answer choice D is correct. In determining when a judge's "impartiality may reasonably be questioned," a judge who has expressed a general opinion or formulated policy prior to becoming a judge that runs counter to a party's position in the current proceeding is not required disqualify herself. A judge is presumed to be able to put aside general preferences and be impartial with regard to the case over which the judge is presiding. Consequently, the judge is not required to disqualify himself on the basis of his public statements regarding the enforcement of the immigration laws because these statements are not grounds to overcome the presumption that the judge can be impartial with regard to the present case. Answer choice A is incorrect. A judge must disqualify himself if he, while a judge or a judicial candidate, has made a public statement, other than in a court proceeding, judicial decision, or opinion, that commits or appears to commit the judge to reach a particular result or rule in a particular way in the proceeding or controversy. Here, the judge has not commented on the criminal prosecution of the defendant at hand, and the general statements that the judge has made were made before the judge was appointed to the court. Answer choice B is incorrect because, in determining when a judge's "impartiality may reasonably be questioned," a judge who has expressed a general opinion or formulated policy prior to becoming a judge that runs counter to a party's position in the current proceeding is not required disqualify himself. A judge is presumed to be able put aside general preferences and be impartial with regard to the case over which the judge is presiding. Answer choice C is incorrect because, while a judge must disqualify himself if he or his spouse or domestic partner has a third-degree relationship or closer to a party, lawyer, material witness, or person who has more than a de minimis interest that could be affected by the proceedings or has a prior relationship with the case at hand, the absence of such relationships does not protect a judge from having to disqualify himself if his impartiality may reasonable be questioned in other ways. However, the pubic position espoused by the judge while he was an attorney regarding enforcement of the immigration laws does not give rise to a reasonable basis for questioning the judge's impartiality.

An attorney represented an individual in a personal injury action against a chain saw manufacturer. While the action was in its discovery phase, the attorney agreed to represent a lender in a class action lawsuit. The lender was accused of imposing improper fees on borrowers. Among the members of the class was the individual, who was not named as a representative of the class. Is the attorney subject to discipline? A) Yes, because the attorney failed to seek the individual's informed consent before representing the lender. B) Yes, because representation of the lender is directly adverse to the individual, who is a current client. C) No, because the class action lawsuit is not substantially related to the personal injury action. D) No, because the individual was an unnamed member of the class in the action against the lender.

D) No, because the individual was an unnamed member of the class in the action against the lender. Answer choice D is correct. The lawyer who represents an unnamed class member in an unrelated matter is typically not required to obtain that class member's consent before representing an opponent in the class action. Answer choice A is incorrect. Generally, a lawyer is required to obtain informed consent in writing before the lawyer may act as an advocate in one matter against a person the lawyer represents in some other matter, even if the matters are wholly unrelated. However, this rule does not apply when one of the clients is an unnamed member of a class action against the other client. Answer choice B is incorrect because, even though the attorney's representation of the lender is directly adverse to the interests of the individual as a member of the class that is suing the lender, the lawyer who represents an unnamed class member in an unrelated matter is typically not required to obtain that class member's consent before representing an opponent in the class action. Answer choice C is incorrect. Although, absent consent, a lawyer generally may not act as an advocate in one matter against a person the lawyer represents in some other matter, even when the matters are wholly unrelated, the lawyer who represents an unnamed class member in an unrelated matter is typically not required to obtain that class member's consent before representing an opponent in the class action.

An attorney represents a client who is under indictment for homicide. In the course of the representation, the client told the attorney that she had previously killed two other people. These murders are completely unrelated to the murder indictment for which the attorney is providing representation. With the client's consent, the attorney made a tape recording of the client's confession regarding the unrelated homicides. At the attorney's request, the client also drew a map of the remote locations of the victims' graves from the unrelated killings. Those bodies have not been found by the police, and the client is not a suspect in either crime, both of which remain unsolved. Is the attorney subject to discipline if he fails to voluntarily disclose to the authorities his knowledge of the two prior murders and the locations of the victims' bodies? A) Yes, because as an officer of the court, the attorney must disclose any knowledge that he has, whether privileged or not, concerning the commission of the prior crimes by his client. B) Yes, because the attorney is impeding the state's access to significant evidence. C) No, because the attorney did not represent or advise his client with respect to the prior crimes. D) No, because the information was obtained by the attorney in the course of the representation.

D) No, because the information was obtained by the attorney in the course of the representation. Answer choice D is correct. In general, MRPC 1.6(a) provides that "[a] lawyer shall not reveal information relating to the representation of a client unless the client gives informed consent." Further, no exception to the confidentiality rule allows or requires the attorney to disclose the information. Answer choice A is incorrect because there is no exception to MRPC 1.6(a), the confidentiality rule, permitting or requiring a lawyer to disclose information relating to a client's representation when the information concerns the client's past crimes of violence. There are exceptions that allow the lawyer to disclose client confidences in certain circumstances (e.g., to prevent a future death or to prevent or rectify financial harm caused by a crime or fraud involving the lawyer's services), but these exceptions do not apply here. Answer choice B is incorrect because criminal laws involving obstruction of justice, as well as judicial opinions, require a lawyer under certain circumstances to disclose physical evidence of a crime when the lawyer possesses that evidence. But the laws and opinions regarding obstruction of justice do not require disclosure of the client's confidential information in this case. Answer choice C is incorrect because the attorney's knowledge is information "relating to the representation" of the client in the homicide case, and therefore it must be kept confidential under MRPC 1.6(a), even though the information relates to murders other than the one for which the client is on trial. The information about the undiscovered homicides would potentially be relevant to the attorney in representing the client competently with regard to the crime charged. For example, the lawyer's knowledge of this information might influence the lawyer's advice to the client about whether to plead guilty or whether to testify at trial, and the information might influence the lawyer's strategic decisions.

The judicial district in which a judge sits has a rule that allows litigants two postponements as a matter of right. After that, a litigant who moves for a postponement must convince the presiding judge that a postponement is appropriate. The judge routinely grants additional postponements because, in her view, "What harm is done if one of the litigants wants a postponement? The worst that can happen is that the parties have more time to negotiate and thus are more likely to settle." Are the judge's actions proper? A) Yes, because the judge is exercising her judicial discretion. B) Yes, because a party objecting to a postponement can seek appellate review. C) No, because judges have no official obligation to encourage private settlements. D) No, because the judge should expedite the determination of matters before her.

D) No, because the judge should expedite the determination of matters before her. Answer choice D is correct. Under CJC Rule 2.5(A), a judge is required to perform judicial and administrative duties diligently, which includes monitoring and supervising cases in ways that reduce or eliminate dilatory practices, avoidable delays, and unnecessary costs. CJC Rule 2.5, cmt 4. A blanket policy granting additional postponements conflicts with the judge's duty to expedite the determination of matters before her. Answer choice A is not correct because the judge's discretion is circumscribed by a duty to expedite the determination of matters before her. Answer choice B is not correct, as it is irrelevant to the determination of whether the judge's actions are proper under the Code of Judicial Conduct that a party can seek appellate review. What controls is the judge's ethical duty as a judge to expedite the determination of matters before her. Answer choice C is not correct, as it is not improper for a judge to attempt to encourage private settlements. The difficulty here is that the judge has imposed a blanket policy of granting additional postponements that may lead to avoidable delays and unnecessary costs, which conflicts with the judge's duty under CJC Rule 2.5(A).

An attorney entered into a contingency fee agreement with a client who wanted to bring a negligence action stemming from an automobile accident. The agreement complied with the professional conduct rules regarding contingency fees. During the discovery phase, the defendant unexpectedly impleaded an automobile mechanic who had worked on the defendant's car shortly before the accident. Because the litigation now threatened to consume more of the attorney's time than she had reasonably anticipated and make her obligation impracticable, she obtained the client's informed consent to modify their original agreement after explaining the matter thoroughly to the client. The modified agreement fairly increased the percentage of the attorney's portion of any recovery by a substantial, but reasonable, amount over the percentage set out in the initial agreement. As with the initial agreement, the modified agreement complied with the requirements imposed on a contingency fee arrangement. Within days after the modified agreement was executed, the defendant offered to settle, and the attorney's client accepted the offer. The client has balked at paying the attorney the increase in the attorney's contingency fee reflected in the modified agreement. The client pointed out that the attorney had expended very little additional effort since the agreement had been modified. Can the client avoid the modified agreement? A) Yes, because, since the attorney had expended very little additional effort after the agreement's modification, she had not earned the increased fee. B) Yes, because an agreement between a client and a lawyer may not be modified after the representation has commenced if the case is settled before going to trial. C) No, because the client cannot avoid the modified agreement once she accepted the defendant's settlement offer. D) No, because the modified agreement was fair and reasonable to the client at the time it was modified.

D) No, because the modified agreement was fair and reasonable to the client at the time it was modified. Answer choice D is correct. Since a client-lawyer relationship is in part a fiduciary relationship, a modification of the fee arrangement beyond a reasonable time after the beginning of the representation may be avoided by a client unless the lawyer is able to establish that the modified contract is fair and reasonable to the client. Since the agreement in question as modified was fair and reasonable to the client at the time that it was modified, the client may not avoid the modified agreement. While some jurisdictions require the modification to be supported by consideration, an agreement to modify a contract that is not supported by consideration is nevertheless enforceable when there are unforeseen difficulties, and one of the parties agrees to compensate the other when the difficulties are discovered if those difficulties would make performance impracticable, as is the case here. Answer choice A is incorrect because, although as a result of the modification the attorney will receive a substantially larger fee that is not justified on the basis of the additional effort expended by the attorney, the attorney can show that, at the time that the agreement was modified, the modified agreement was fair and reasonable. Answer choice B is incorrect because, while the modification of a fee agreement once a reasonable time after the beginning of the representation has past is subject to special scrutiny, the modification can withstand a client's challenge if the lawyer can show that the modified agreement is fair and reasonable. Answer choice C is incorrect because, even though the client, knowing of the modified agreement, had accepted the defendant's settlement offer, the client was not prohibited from avoiding the agreement unless the lawyer could establish that the agreement was fair and reasonable, which the attorney here could do.

An attorney was retained by a defendant to represent him in a paternity suit. The defendant's aunt believed the suit was unfounded and motivated by malice. The aunt sent the attorney a check for $1,000 and asked the attorney to apply it to the payment of the defendant's fee. The aunt told the attorney not to tell the defendant of the payment because, "The defendant is too proud to accept gifts, but I know he really needs the money." Is it proper for the attorney to accept the aunt's check? A) Yes, if the aunt does not attempt to influence the attorney's conduct of the case. B) Yes, if the attorney's charges to the defendant are reduced accordingly. C) No, because the aunt is attempting to finance litigation to which she is not a party. D) No, unless the attorney first informs the defendant and obtains the defendant's consent to retain the payment.

D) No, unless the attorney first informs the defendant and obtains the defendant's consent to retain the payment. Answer choice D is correct. Under MRPC 1.8(f), a lawyer may not accept payment for representation from someone other than the client, unless (among other things) the client gives informed consent. In the absence of informed consent by the defendant, it is not proper for the attorney to accept the aunt's check. Answer choice A is not correct. MRPC 1.8(f) requires that in order for a lawyer to accept payment for representation from someone other than the client, there must be no interference with the lawyer's professional judgment and there must be informed consent by the client. Answer choice B is not correct, as there is no requirement that the attorney's charges be reduced. All that is required for a lawyer to accept payment for representation from someone other than the client is that: (i) the client gives informed consent, (ii) there is no interference with the lawyer's professional judgment, and (iii) lawyer-client confidentiality is preserved. MRPC 1.8(f). Answer choice C is not correct. MRPC 1.8(f) permits such financing, so long as the above conditions are met.

An attorney has tried many contested cases before a certain judge up for reelection. The attorney believes the judge is lacking both in knowledge of the law and in good judgment and that a different attorney would make an excellent judge. The attorney wishes to defeat the judge and assist the other attorney in getting elected. The attorney intends to contribute $5,000 to the other attorney's campaign. Is it proper for the attorney to do so? A) Yes, the attorney may give $5,000 to the other attorney personally for his campaign. B) Yes, if the attorney's contribution to the other attorney is made anonymously. C) No, because the attorney is practicing before the court to which the other attorney seeks election. D) No, unless the attorney gives the $5,000 to a committee formed to further the other attorney's election.

D) No, unless the attorney gives the $5,000 to a committee formed to further the other attorney's election. Answer choice D is correct. A judicial candidate is not permitted to solicit or accept campaign contributions other than through a campaign committee. CJC Rule 4.1. Thus, unless the attorney's contribution is made to a campaign committee, it would be improper. Answer choice A is incorrect, as CJC Rule 4.1 prohibits a judicial candidate from personally accepting campaign contributions. Answer choice B is incorrect because, even if made anonymously, a contribution to a candidate for judicial office must be made through a campaign committee established to accept contributions on behalf of the candidate. Answer choice C is incorrect, as neither the Model Rules nor the Code of Judicial Conduct prohibits campaign contributions by lawyers who practice before a court to which a candidate for judicial office seeks election.

A woman has asked an attorney to represent her in obtaining compensation for a tract of land that is being condemned by the state department of transportation to build a new highway. Two years ago, the attorney had been employed by the department and had been assigned to search title on several tracts of land, including the one owned by the woman. The attorney remembers a department engineer had drafted a confidential memorandum advising against running a new highway across the woman's land because of a potential adverse environmental impact. Because of this information, the attorney believes it is possible to prevent the condemnation of the woman's land or to increase the settlement amount. What is the proper action for the attorney to take? A) Represent the woman on the issue of damages only and not disclose the information that might prevent the condemnation. B) Represent the woman and attempt to prevent the condemnation by using the information about the adverse environmental impact. C) Refuse to represent the woman but disclose to her the information about the adverse environmental impact. D) Refuse to represent the woman and not disclose the information about the adverse environmental impact.

D) Refuse to represent the woman and not disclose the information about the adverse environmental impact. Answer choice D is correct. MRPC 1.11(a)(1) provides that a lawyer who was formerly employed by a government agency may not later represent a client "in connection with a matter in which the lawyer participated personally and substantially" as a public employee, unless the government agency consents. Having participated personally and substantially in an early stage of the condemnation matter by conducting a title search on the woman's property, the attorney may not now represent the woman in opposing or seeking additional damages for the condemnation. Further, the attorney may not disclose the information learned while working at the department because, under MRPC 1.11(a)(2) and MRPC 1.9(c), a lawyer who was formerly employed by the government must not reveal information relating to a former representation. Answer choices A and B are incorrect, as having participated personally and substantially in an early stage of the condemnation matter by conducting a title search on the woman's property, the attorney may not now represent the woman in opposing or seeking additional damages for the condemnation. Answer choice C is incorrect, as the attorney may not disclose information learned while working at the department because, under MRPC 1.11(a)(2) and MRPC 1.9(c), a lawyer who was formerly employed by the government must not reveal information relating to a former representation.

A client retained an attorney to appeal his criminal conviction and to seek bail pending appeal. The agreed-upon fee for the appearance on the bail hearing was $100 per hour. The attorney received $1,600 from the client, of which $600 was a deposit to secure the attorney's fee and $1,000 was for bail costs in the event that bail was obtained. The attorney maintained two office bank accounts: a fee account, in which all fees collected from clients were deposited and from which all office expenses were paid, and a clients' trust account. The attorney deposited the $1,600 in the clients' trust account the week before the bail hearing. She expended six hours of her time preparing for and appearing at the hearing. The effort to obtain bail was unsuccessful. Dissatisfied, the client immediately demanded return of the $1,600. What should the attorney do with the $1,600? A) Transfer the $1,600 to the fee account. B) Transfer the $600 to the fee account and leave $1,000 in the clients' trust account until the attorney's fee for the final appeal is determined. C) Transfer $600 to the fee account and send the client a $1,000 check on the clients' trust account. D) Send the client a $1,000 check and leave $600 in the clients' trust account until the matter is resolved with the client.

D) Send the client a $1,000 check and leave $600 in the clients' trust account until the matter is resolved with the client. Answer choice D is correct. MRPC 1.15(e) requires that when the lawyer has a dispute with a client over funds in the lawyer's possession, the lawyer must deliver to the client the funds that undisputedly belong to the client and hold the remainder in the clients' trust account until the dispute is resolved. Therefore, the attorney is required to hold in the clients' trust account the amount of disputed legal fees ($600) and return the remaining $1,000 to the client. Answer choice A is incorrect. When legal fees are paid in advance, they must be kept in the clients' trust account until earned. When the representation ends, the lawyer must return the unearned portion (here, $1,000) to the client. The lawyer may withdraw fees as earned except when, as in this case, they are disputed, in which case they must be kept in the clients' trust account until the dispute is resolved. Answer choice B is incorrect because both steps are wrong. The $600, which represents the amount in dispute, must be kept in the clients' trust account until the dispute is resolved. The remaining $1,000, which represents unearned fees, belongs to the client, and it must therefore be returned to the client. Answer choice C is incorrect. Although the lawyer is right to disburse $1,000 from the clients' trust account to the client (since that amount represents unearned fees and therefore undisputedly belongs to the client), the $600 that is in dispute must be kept in the clients' trust account until the dispute is resolved.

An attorney places an advertisement in the local newspaper that includes only true information. For which, if any, of these statements is the attorney subject to discipline? A) My credentials are: B.A., magna cum laude, Eastern College; J.D., summa cum laude, State Law School; LL.M., Eastern Law School. B) My offices are open Monday through Friday from 9:00 a.m. to 5:00 p.m., but you may call my answering service twenty-four hours a day, seven days a week. C) I speak modern Greek fluently. D) The attorney is not subject to discipline for any of these statements.

D) The attorney is not subject to discipline for any of these statements. Answer choice D is correct. Answer choice A will not subject the attorney to discipline, as a lawyer is generally permitted to advertise her services through a newspaper, so long as the communications regarding such services are not false or misleading or in violation of the rules against solicitation of clients. MRPC 7.2(a). These are true statements about the attorney's credentials and do not violate the rules. Answer choice B will not subject the attorney to discipline, as MRPC 7.2 permits public dissemination of information concerning office hours and a lawyer's availability through an answering service. Answer choice C will not subject the attorney to discipline, as MRPC 7.2 permits a lawyer to disseminate a true statement about her foreign language ability. Therefore, because all of the statements in the attorney's advertisement are permissible, answer choice D is the best answer.

A brother and sister were charged with robbery after they attempted to rob a jewelry store together. The brother and sister hired a private defense attorney to jointly represent them in their trial. The attorney reasonably believed he would be able to provide competent and diligent representation to both the brother and the sister, and each defendant gave informed consent to the representation, confirmed in writing. In the course of the representation, the attorney frequently met with the brother and sister at the same time. At these meetings, the brother and sister told the attorney that they both agreed to rob the jewelry store. However, one day the brother asked the attorney to meet with him separately. During this meeting, the brother told the attorney that the sister had masterminded the entire robbery, and that she had blackmailed the brother into going along with the plan against his will. The brother also showed the attorney an audio recording of a telephone call with his sister that supported his allegation and severely incriminated his sister. Who can the attorney now properly represent? A) The attorney can represent the brother but must withdraw from representing the sister. B) The attorney can represent the sister but must withdraw from representing the brother. C) The attorney can represent both the brother and the sister. D) The attorney must withdraw from representing both the brother and the sister.

D) The attorney must withdraw from representing both the brother and the sister. Answer choice D is correct. Ordinarily, a lawyer will be forced to withdraw from representing all of the clients if the common representation fails. Here, the common representation has failed because a substantial conflict has arisen by reason of substantial discrepancy in the parties' testimony. Initially, the attorney was under the impression that the brother and sister agreed to rob the bank together. However, he later learned that according to the brother, the sister had masterminded the entire robbery and forced her brother to go along with the robbery against his will. Common representation of the brother and sister would not be successful at this point as there is a clear conflict of interest. Therefore the attorney must now withdraw from representing both the brother and the sister. For this reason, answer choice C is incorrect. Answer choice A is incorrect because the attorney is not permitted to continue to represent the brother because his duties to the sister as a former client will severely impede his ability to argue in the brother's defense. Likewise, answer choice B is incorrect because the attorney's duties to the brother as a former client would severely impede his ability to argue in the sister's defense.

Which of the following is NOT a requirement for a firm to avoid imputed disqualification with respect to an incoming lawyer who has a conflict of interest? A) The lawyer is immediately screened on the matter. B) The lawyer does not receive any part of the fee earned by the firm from the matter. C) The client is notified. D) The client consents.

D) The client consents.

Which of the following is NOT required for a lawyer to take payment in the client's stock or by a mortgage on the client's property? A) The agreement to do so must be in writing. B) The arrangement must be fair and reasonable. C) The client must be informed in writing of the prudence of retaining independent counsel to review the transaction. D) The property must be the subject matter of the litigation.

D) The property must be the subject matter of the litigation.

Three months ago, an attorney was retained by his friend, who was a car collector, to assist in the sale of one of his cars. After the sale was complete, the friend asked the attorney to keep the funds from the car sale for him, as he planned to purchase another car in the near future and would need the assistance of the attorney again. The attorney agreed and asked her bookkeeper to deposit the funds for safekeeping until the friend bought a new car. The attorney did not give the bookkeeper any instructions as to where to deposit the funds, nor did she follow up with the bookkeeper to determine where the funds were deposited. The bookkeeper decided to deposit the friend's funds into the attorney's personal savings account because it had a high interest rate. The bookkeeper kept the friend's financial records in accordance with generally accepted accounting practices, so he was confident that he could accurately keep track of the friend's funds despite the fact that they were commingled with the attorney's personal funds. Because the attorney rarely checked the balance of her savings account, she was not aware that the bookkeeper had commingled the friend's funds with her own personal funds. Is the attorney subject to discipline? A) No, because the attorney was not aware that the bookkeeper had commingled the friend's funds with her own personal funds. B) No, because the bookkeeper maintained the friend's financial records on a current basis in accordance with generally accepted accounting practices. C) Yes, because an attorney is always liable for the misconduct of a non-lawyer over which he has direct supervisory authority. D) Yes, because an attorney with direct supervisory authority over the work of a non-lawyer must make reasonable efforts to ensure that the supervised person's conduct conforms to the Model Rules.

D) Yes, because an attorney with direct supervisory authority over the work of a non-lawyer must make reasonable efforts to ensure that the supervised person's conduct conforms to the Model Rules. Answer choice D is correct. A lawyer with direct supervisory authority over the work of another lawyer in the firm or a non-lawyer employed by the firm must make reasonable efforts to ensure that the supervised person's conduct conforms to the Model Rules. Here, the attorney did not make any efforts to ensure that the bookkeeper's conduct conformed to the Model Rules. She did not give the bookkeeper any instructions as to where to deposit the friend's funds, nor did she do any follow up after the funds were deposited. As a result, the bookkeeper violated the Model Rules by commingling client funds with an attorney's personal funds. For this reason, the attorney is subject to discipline. Answer choice A is incorrect because even though the attorney was not aware that the bookkeeper had commingled the friend's funds with her own personal funds, she is still subject to discipline. Answer choice B is incorrect. Although the bookkeeper maintained the friend's financial records in accordance with generally accepted accounting practices, he is still in violation of the Model Rules because he commingled client funds with an attorney's personal funds. For this reason, the attorney is subject to discipline as his direct supervisor. Answer choice C is incorrect. An attorney is not always liable for the misconduct of a non-lawyer over which he has direct supervisory authority, but an attorney must make reasonable efforts to ensure that the supervised person's conduct conforms to the Model Rules.

An attorney represented a client in an action against the client's former partner to recover damages for breach of contract. During the representation, the client presented the attorney with incontrovertible proof that the former partner had committed perjury in a prior action which was resolved in the partner's favor. Neither the attorney nor the client was involved in any way in the prior action. The attorney believes that it would be detrimental to the client's best interests to reveal the perjury because of the implication that might be drawn from the former close personal and business relationship between the client and the former partner. Would it be proper for the attorney to fail to disclose the perjury to the tribunal? A) No, because the information is unprivileged. B) No, because the attorney has knowledge that the former partner perpetrated a fraud on the tribunal. C) Yes, because neither the client nor the attorney was involved in the prior action. D) Yes, because the attorney believes that the disclosure would be detrimental to the client's best interests.

D) Yes, because the attorney believes that the disclosure would be detrimental to the client's best interests. Answer choice D is correct. The information about perjury committed by the client's former partner is "information relating to the representation," which generally may not be disclosed without client consent. Confidential information may be disclosed under MRPC 1.6(a) when "the disclosure is impliedly authorized in order to carry out the representation," but there would not be implied authorization when, as here, the disclosure would be likely to prejudice the client, not advance the client's objectives. Answer choice A is incorrect because the lawyer's duty to protect client information under MRPC 1.6(a) covers more than just privileged information. It covers all "information relating to the representation of a client." Answer choice B is incorrect; a lawyer has a duty under MRPC 3.3(b) to take remedial measures when the lawyer knows that a person is engaged in criminal conduct related to a proceeding in which the lawyer represents a client. But that duty does not apply here, because the attorney did not represent the client in the proceeding in which the client's former partner committed perjury. Therefore, the duty to maintain confidentiality applies to this information. Answer choice C is incorrect; a lawyer has a duty under MRPC 3.3(b) to take remedial measures when the lawyer knows that a person is engaged in criminal conduct related to a proceeding in which the lawyer represents a client. But the duty continues only to the end of that proceeding. Here, the proceeding in which the former partner committed perjury is over. Therefore, the fact that neither the client nor the attorney was involved in the prior action is irrelevant, since even if the lawyer had represented the client in that proceeding, the lawyer would have no duty at this time to disclose the former partner's perjury.

An attorney opened a trust account at a local bank into which she deposited a check from a client that constituted an advance payment for future services to be rendered by the attorney. Subsequently, the attorney received an engagement fee from another client, which the attorney also deposited into the account. The engagement fee represented payment to the attorney for accepting the case, being available to handle the case, and agreeing not to represent another party in the case. The fee did not require the attorney to perform specific legal services. The attorney maintained records related to all account transactions in accordance with the state rules of professional conduct, which were identical to the ABA Model Rules of Professional Conduct, but she did not pay bank service charges on the account from her own funds. Is the attorney subject to discipline with regard to her actions concerning the trust fund account? A) No, because the attorney properly maintained the records related to the account. B) No, because the attorney maintained a trust fund account to keep her property separate from her client's property. C) Yes, because the attorney failed to pay the bank service charges on the account from her own funds. D) Yes, because the attorney did not keep her client's property separate from her own property.

D) Yes, because the attorney did not keep her client's property separate from her own property. -Read the call of the question first. This is an issue of CLIENT'S PROPERTY/FEES. The RULE is: the lawyer MUST SEGREGATE his money from the client's money. If the fee IS NOT for LEGAL WORK, then the money BELONGS TO THE ATTORNEY. If the fee IS for legal services (even for future legal services) IS THE CLIENT'S MONEY. -A) is wrong because is not the point, the issue is about segregating attorney's money from client's money. -B) is wrong because it is not factually accurate. -C) is wrong because the lawyer could the bank charges, but is not required to. -D) is correct because the engagement fee is the lawyer's property and the retainer pay is still the client's property.

An attorney represented a plaintiff in the plaintiff's action for defamation against a defendant. After the defendant's lawyer had filed and served an answer, the attorney, at the plaintiff's direction, hired a licensed private investigator and instructed him to attempt to interview the defendant without revealing his employment. The investigator succeeded in interviewing the defendant privately and obtained an admission from the defendant that the statements she had made were based solely on unsubstantiated gossip. Is the attorney subject to discipline for obtaining the statement from the defendant in this matter? A) No, because the attorney was following the plaintiff's instructions. B) No, because the statement obtained was evidence that the defendant's allegations were unfounded. C) Yes, because the attorney should have interviewed the defendant personally. D) Yes, because the attorney instructed the investigator to interview the defendant.

D) Yes, because the attorney instructed the investigator to interview the defendant. Answer choice D is correct. Under MRPC 4.2, the attorney or a representative of the attorney is prohibited from talking to a party represented by a lawyer about the subject matter of the representation. Answer choice A is not correct, as the interview of the defendant violated MRPC 4.2 even if it was at the instruction of the plaintiff. Answer choice B is not correct, as the mere fact that the interview took place violated MRPC 4.2. Answer choice C is not correct, as MRPC 4.2 is violated by the attorney or a representative of the attorney interviewing a party represented by a lawyer.

A client hired an attorney to help with a complex real estate transaction involving the sale of her restaurant to a large conglomerate. The attorney conducted all of the necessary research involved with the valuation of the restaurant, gathered and analyzed the restaurant's financial records, ensured the restaurant's permits were all valid, and facilitated the negotiations between the two parties. The attorney also prepared the final draft of the contract for the sale of the restaurant and sent it to the conglomerate's lawyers for review. However, when emailing the final draft of the contract, the attorney inadvertently included metadata that revealed confidential client information. The conglomerate's lawyers used this information to their advantage and offered a lower price for the restaurant, ultimately causing the client to lose many thousands of dollars on the transaction. The client subsequently filed a negligence action against the attorney for malpractice. Is the client likely to succeed? A) No, because a client cannot recover in a malpractice action for an attorney's violation of a rule of professional responsibility. B) No, because the attorney was a diligent and competent lawyer, and she did not intend to send the metadata in the contract. C) Yes, because a violation of a rule of professional responsibility is negligence per se. D) Yes, because the attorney's breach of her duty of care to the client caused the client actual damages.

D) Yes, because the attorney's breach of her duty of care to the client caused the client actual damages. Answer choice D is correct. A lawyer owes a duty of care to the client. The duty of care is generally the competence and diligence exercised by lawyers of similar experience under similar conditions. Here, the attorney breached her duty of care to the client by inadvertently including the metadata that revealed confidential client information which ultimately caused the client to lose many thousands of dollars on the transaction. Therefore, the client is likely to succeed in a malpractice claim against the attorney based on negligence. Answer choice A is incorrect. In addition to professional discipline, a lawyer who violates an ethics rule may also potentially be subject to civil liability for malpractice. Answer choice B is incorrect. Although in general the attorney was a diligent and competent lawyer, and she did not intend to send the metadata in the contract, she can still be liable under a negligence theory of recovery for breaching her duty of care by inadvertently revealing confidential client information. Answer choice C is incorrect. The mere violation of a rule of professional responsibility does not automatically result in a finding of negligent malpractice. It is not negligence per se. Violation of a rule is, however, generally treated as evidence that the lawyer's conduct violated the duty of care.

An attorney represented a client who was charged with first-degree murder of his employer. The client allegedly murdered his employer, who was deathly allergic to peanuts, by putting peanut butter in a cake that he made for the employer's birthday. The employer unknowingly ate the cake, and soon thereafter died from anaphylaxis. During a deposition of the client conducted by the prosecution, the prosecutor asked the client if he knew that the employer had a peanut allergy. The client answered "No, I did not know that my employer was allergic to peanuts." After the deposition, the client was talking to the attorney, and said the following to her: "Could you tell I was lying during the deposition? I knew all along that my boss had a peanut allergy!" Is the attorney required to report the false statement made during the deposition to the tribunal? A) No, because the attorney would violate her duty of confidentiality to the client if she did so. B) No, because the false statement was made during a deposition. C) Yes, because an attorney is obligated to report all false statements made by her client. D) Yes, because the duty of candor applies to ancillary proceedings conducted pursuant to the tribunal's authority.

D) Yes, because the duty of candor applies to ancillary proceedings conducted pursuant to the tribunal's authority. Answer choice D is correct. A lawyer who is representing a client in the proceedings of a tribunal has the duty of candor. This duty also applies when the lawyer is representing a client in an ancillary proceeding conducted pursuant to the tribunal's authority, such as a deposition. Here, the client made the false statement regarding his knowledge about the employer's peanut allergy during a deposition, which is an ancillary proceeding conducted pursuant to the tribunal's authority. Therefore, the attorney was required to report the false statement to the tribunal. For this reason, answer choice B is incorrect. Answer choice A is incorrect. The duty of candor applies even if compliance requires disclosure of information otherwise protected by the ethical duty of confidentiality. Answer choice C is incorrect because it is too broad. An attorney is not required to report all false statements made by her client. For instance, if the client lies to the attorney separate from any proceeding conducted pursuant to the tribunal's authority, the attorney is not required to report the false statement to the tribunal under the duty of candor.

An attorney was retained by a corporation to draft employment contracts for its employees. Two years after the attorney stopped representing the corporation, the attorney was hired by a former employee of the corporation to recover bonuses allegedly owed to him for services rendered to the corporation. The attorney filed a complaint on behalf of the employee arguing that the employment contract she had drafted for the corporation two years ago, which allowed for termination of the employee without paying him a bonus, was void. The corporation subsequently filed a motion asking the court to disqualify the attorney from representing the employee in this action due to her prior relationship with the corporation. Should the attorney be disqualified from representing the employee in the current action? A) No, because the duty of loyalty to the corporation does not extend beyond the duration of representation. B) No, because there is no conflict of interest between the former and current client. C) Yes, because a lawyer who has previously represented a client in a matter can never subsequently represent another person in the same or a substantially related matter. D) Yes, because the employee had materially adverse interests to the corporation.

D) Yes, because the employee had materially adverse interests to the corporation. Answer choice D is correct. When a lawyer has been directly involved in a specific transaction, subsequent representation of other clients with materially adverse interests in that transaction clearly is prohibited. Here, the employee is seeking representation involving the same contract the attorney had drafted for the company and has materially adverse interests to the corporation, because the attorney was arguing that the employment contract that she originally drafted for the corporation was void. Therefore, the attorney should be disqualified from representing the employee in the current action. For this reason, answer choice B is incorrect. Answer choice A is incorrect. The duty of loyalty to a client extends beyond the duration of the representation and can limit a lawyer's ability to represent other clients or use information obtained in representing the client. Answer choice C is incorrect because it is an incorrect statement of the applicable rule. A lawyer who has previously represented a client in a matter must not subsequently represent another person in the same or a substantially related matter in which that person's interests are materially adverse to the interests of the former client, unless the former client gives informed consent, confirmed in writing. However, representation concerning the same or a substantially related matter is permitted where the current client's interests are not materially adverse to those of the former client, or where the former client gives informed consent, confirmed in writing.

An attorney represented a group of consumers that brought an action for an emergency hearing to halt the unfair public disclosure of the consumers' sensitive personal information on the internet by a company that buys and sells consumer debt. The temporary restraining order sought by the attorney would enjoin the company from disclosing any sensitive information without first implementing reasonable safeguards to maintain and protect the privacy, security, and confidentiality of the information. Despite his due diligence, the attorney was unable to give notice of the emergency hearing to the company. No one representing the company appeared at the hearing, and the attorney asked for a temporary restraining order from the judge. The attorney argued in good faith that exposure of the consumers' sensitive information placed them at substantial risk of identity theft, financial account fraud, and invasion of privacy. However, the attorney did not reveal to the court that an independent expert had evaluated the company's use of the consumers' sensitive information and concluded that reasonable safeguards were in place to protect the information. Is the attorney subject to discipline? A) No, because the attorney had a good-faith basis for his argument. B) No, because the attorney would have breached his duty of loyalty by disclosing the expert's opinion to the court. C) Yes, because the attorney engaged in ex parte communications with the judge. D) Yes, because the ex parte nature of the hearing required that the attorney inform the court of material facts known to the attorney.

D) Yes, because the ex parte nature of the hearing required that the attorney inform the court of material facts known to the attorney. Answer choice D is correct. Although litigators generally do not have a duty to disclose adverse facts to other parties, Model Rule 3.3(d) imposes a special obligation on lawyers participating in ex parte proceedings. Specifically, lawyers must inform the tribunal of all material facts known to the lawyer that will enable the tribunal to make an informed decision, whether or not the facts are adverse. Thus, here the attorney should have told the judge about the expert's conclusion, even though the information was adverse to his client's interests. Answer choice A is incorrect. The fact that the attorney had a good-faith belief supporting his argument does not justify the failure to disclose material facts at an ex parte hearing as required by Model Rule 3.3(d). Answer choice B is incorrect. The duty of candor under Model Rule 3.3(d) requires disclosure of all material facts during an ex parte proceeding, despite the fact that the disclosure may hurt the lawyer's client. Answer choice C is incorrect. Under certain circumstances, a rule of civil procedure allows proceedings to be conducted on an ex parte basis. As indicated in the facts, the attorney filed a petition for an emergency hearing seeking a temporary restraining order. Communications in such a proceeding do not violate the prohibition on ex parte communications with a judge because such communications are authorized by law.

A state hospital created a group legal service plan for its medical staff. Under the plan, the medical staff can pay a monthly fee to receive legal help for common legal matters. Hospital administrators manage the plan, and any private attorneys that are hired by them are paid $40 per hour to provide legal assistance to the medical staff. Any profits derived from the plan are put into the state's cancer research fund. Nonlawyer plan agents use in-person contact to solicit all new medical staff to join the legal service plan. Can private attorneys participate in the state hospital's group legal service plan? A) No, because plan agents use in-person contact to solicit new medical staff to join the legal service plan. B) No, because the private attorneys share legal fees with the state. C) Yes, because the legal service plan is not operated by a for-profit organization. D) Yes, because the legal service plan is not owned or directed by the private attorneys.

D) Yes, because the legal service plan is not owned or directed by the private attorneys. Answer choice D is correct. Under Model Rule 7.3(e), lawyers may participate with a prepaid or group legal service plan that uses live person-to-person contact to enroll members for the plan from persons who are not known to need legal services in a particular matter covered by the plan. As is the case here, the plan must be operated by an organization and not owned or directed by the lawyers. Since this legal service plan meets these requirements, the private lawyers are permitted to participate. Answer choice A is incorrect. Lawyers may participate with a prepaid or group legal service plan that uses live person-to-person contact to enroll members for the plan from persons who are not known to need legal services in a particular matter covered by the plan. Here, the nonlawyer plan agents use in-person contact to solicit all new medical staff to join the legal service plan, not merely those known to be in need of legal services. Therefore, this use of in-person solicitation is permitted. Answer choice B is incorrect. The facts indicate that state hospital medical staff pay a monthly fee to receive legal assistance under the plan and profits from the plan go into the hospital's cancer research fund. The private attorneys do not share their legal fees with the state hospital, but their legal fees are paid by the state hospital from the participants' monthly fees. Answer choice C is incorrect. The propriety of the solicitation does not turn on whether the plan is a for-profit plan. Rather, it turns on the fact that the plan is operated by an organization and not owned or directed by the private attorneys.

An attorney represented a husband in a divorce proceeding. In addition to their other belongings, the couple jointly owned a motorcycle that was currently in the husband's possession. The couple had already sold their marital home, and they had nowhere to store the motorcycle, so the husband asked the attorney if could store the motorcycle for him. The attorney had space in his personal garage where he kept his own car and other personal belongings, so he agreed to keep the motorcycle there until the divorce proceedings were over. The attorney lived in an area with low crime rates, and the entrances to the garage were securely locked. One week later, an electrical fire accidentally started in the attorney's garage, destroying the motorcycle. Is the attorney subject to discipline? A) No, because the attorney did not cause the electrical fire. B) No, because the attorney's garage was a safe place to store the motorcycle. C) Yes, because attorneys are only permitted to store client funds for safekeeping. D) Yes, because the motorcycle was commingled with the attorney's personal property.

D) Yes, because the motorcycle was commingled with the attorney's personal property. Answer choice D is correct. During the course of a representation, a lawyer has a duty to protect a client's property that is in the lawyer's possession. All property that is the property of clients must be kept separate from the lawyer's business and personal property. Here, the attorney kept the client's motorcycle in his personal garage where he kept his own car and other personal belongings. Under the Model Rules, this is not permissible. Therefore, the attorney is subject to discipline. Answer choice A is incorrect because it is not relevant that the attorney did not cause the electrical fire. The attorney is already subject to discipline for storing the motorcycle with his personal property. Answer choice B is incorrect because although the garage may have been a safe place to store the motorcycle, an attorney cannot commingle a client's property with his own personal property. Answer choice C is incorrect because an attorney is permitted to store client property in addition to client funds.

Attorney Alpha filed a complaint on behalf of a client against a corporation, alleging that the corporation had breached a valid oral contract, entered into on the corporation's behalf by the president and chief executive officer of the company, to sell the client certain merchandise for a specified price. Attorney Beta, representing the company, has filed an answer denying the contract and asserting the Statute of Frauds as a defense. Attorney Beta has given notice to Alpha that he will take the deposition of the president on the grounds that the president will be out of the country on the date the case is set for trial. The president is not a shareholder of the corporation. Alpha would like to interview the president, prior to the taking of the deposition, to better prepare her cross-examination. Is Alpha subject to discipline if she interviews the president without Beta's knowledge and consent? A) No, unless the president will be personally liable to the corporation for damages in the event judgment is rendered against it. B) No, because the president allegedly entered into the contract on behalf of the corporation. C) Yes, because the president is being called as an adverse witness. D) Yes, because the president is the president of the corporation.

D) Yes, because the president is the president of the corporation. Answer choice D is correct. The president of the corporation is considered a client represented by Attorney Beta. In the case of a represented organization, a lawyer is prohibited from communicating with a constituent of the organization who supervises, directs, or regularly consults with the organization's lawyer concerning the matter or has authority to obligate the organization with respect to the matter, or whose act or omission in connection with the matter may be imputed to the organization for purposes of civil or criminal liability. MRPC 4.2, cmt 7. Under MRPC 4.2, Attorney Alpha is not permitted to communicate about the subject of the representation with a person the lawyer knows to be represented by another lawyer in the matter, without the consent of the other lawyer or authorization by law or a court order. Answer choice A is not correct, as it does not matter whether the president will be personally liable to the corporation for damages. What controls is that the president is considered a client of Attorney Beta under MRPC 4.2. Answer choice B is not correct, as the fact that the president may have entered into the contract on behalf of the corporation does not control the issue. The president is considered a client of Attorney Beta with regard to the matter, and Attorney Alpha cannot interview the president without Beta's consent. Answer choice C is not correct because even if the president is called as an adverse witness, the president is considered Beta's client under MRPC 4.2 and cannot be interviewed by Attorney Alpha without Beta's consent.

Three years ago, a defendant was convicted of burglary and sentenced to prison for eight years. Recently, new DNA evidence that proved the defendant's innocence by clear and convincing evidence was discovered by the prosecutor who prosecuted the case. The prosecutor reported this evidence to the court, and the court appointed a lawyer to represent the defendant. The prosecutor then disclosed the DNA evidence to the court-appointed lawyer as well. A state law requires a court to set aside a conviction when newly discovered evidence proves the innocence of a convicted defendant. Pursuant to this law, the court-appointed lawyer moved to set aside the defendant's conviction. Despite the clear and convincing evidence of the defendant's innocence, the prosecutor felt that he had a professional duty to oppose the motion and argue in favor of upholding the defendant's conviction. The court ultimately granted the motion to set aside the conviction. However, the prosecutor's decision to oppose the motion led to a lengthy litigation, during which the defendant remained in prison. Is the prosecutor subject to discipline for opposing the motion? A) No, because the prosecutor disclosed the DNA evidence to the court. B) No, because the prosecutor disclosed the DNA evidence to the defendant's court-appointed lawyer. C) Yes, because the court ultimately granted the motion to set aside the conviction. D) Yes, because the prosecutor should have supported the motion to set aside the defendant's conviction.

D) Yes, because the prosecutor should have supported the motion to set aside the defendant's conviction. Answer choice D is correct. Model Rule 3.8(h) requires a prosecutor to seek to remedy a conviction in the prosecutor's jurisdiction when the prosecutor knows of clear and convincing evidence that a defendant was wrongfully convicted. This obligation goes beyond disclosing evidence to the defendant or the defendant's new lawyer. Here, the prosecutor must seek to remedy the defendant's conviction by supporting the motion to set aside his conviction of burglary. Therefore, the prosecutor is subject to discipline for arguing to uphold the defendant's conviction, despite knowing there was clear and convincing evidence of the defendant's innocence. Answer choices A and B are incorrect. Where the prosecutor knows of new, credible, and material evidence creating a reasonable likelihood of a wrongful conviction, not only must the prosecutor disclose the evidence to the appropriate court but, if the conviction was obtained in the prosecutor's jurisdiction, the prosecutor must also disclose it to the defendant. However, under Model Rule 3.8(h), if that evidence is clear and convincing, the prosecutor must still seek to remedy the conviction. Therefore, the prosecutor's conduct was improper. Answer choice C is incorrect. Because the prosecutor knew there was clear and convincing evidence of the defendant's innocence, arguing to uphold the defendant's conviction was improper, regardless of whether the court ultimately granted the motion to set aside the conviction.

A prosecutor was prosecuting a case involving a criminal defendant who stole millions of dollars from people who invested with him through a Ponzi scheme. The defendant was alleged to have conned hundreds of senior citizens out of their retirement funds. The case was highly publicized, so the jury was sequestered, and the general public was kept out of the courtroom during the trial. The prosecutor also made an effort to keep her comments about the defendant in public neutral and to a minimum due to the amount of public animosity towards him. On the day the jury went into deliberations, the prosecutor left the court room and was approached by the press for a statement. The prosecutor, feeling like she could finally speak her mind, said the following: "I think the defendant is the biggest crook in the world! He has no morals, he is greedy, and this isn't the first time he has scammed people out of their live savings. He should be locked up for the rest of his life!" Is the prosecutor subject to discipline? A) No, because prosecutors are permitted to make any statements in order to support the successful outcome of their case. B) No, because the prosecutor was only stating her opinion about the defendant, and the statement was made after the jury went into deliberations. C) Yes, because prosecutors are under an obligation to not make any extrajudicial comments about a defendant during a trial. D) Yes, because the prosecutor's comments had a substantial likelihood of heightening public condemnation of the defendant.

D) Yes, because the prosecutor's comments had a substantial likelihood of heightening public condemnation of the defendant. Answer choice D is correct. Except for statements that are necessary to inform the public of the nature and extent of the prosecutor's action and that serve a legitimate law enforcement purpose, a prosecutor must refrain from making extrajudicial comments that have a substantial likelihood of heightening public condemnation of the accused. Here, the prosecutor's statements about the defendant have a substantial likelihood of heightening public condemnation of the defendant since she calls him the "biggest crook in the world" and implies that he has committed a similar crime in the past. Therefore, the prosecutor is subject to discipline. Answer choice A is incorrect because prosecutors are not free to make any statements in order to support the successful outcome of their case. Rather, as discussed above, they may only make statements that are necessary to inform the public of the nature and extent of the prosecutor's action and that serve a legitimate law enforcement purpose. Answer choice B is incorrect. Although the prosecutor was only stating her opinion about the defendant, and the statement was made after the jury went into deliberations, the prosecutor was still under an obligation to not make extrajudicial comments that have a substantial likelihood of heightening public condemnation of the accused. Answer choice C is incorrect. Prosecutors are permitted to make extrajudicial comments about a defendant as long as they are necessary to inform the public of the nature and extent of the prosecutor's action and that serve a legitimate law enforcement purpose.

A woman hired an attorney to represent her in a breach of contract cause of action. One month after the client-lawyer relationship began, the woman and the attorney engaged in consensual sexual relations with one another. Concerned that this would be a conflict of interest, the attorney had the woman waive the conflict by obtaining her informed consent, confirmed in writing. Is the attorney subject to discipline? A) No, because the woman waived the conflict through her informed consent, confirmed in writing. B) No, because the sexual relationship did not exist when the client-lawyer relationship began. C) Yes, because a lawyer is always prohibited from having sexual relations with a client. D) Yes, because the sexual relationship began after the client-lawyer relationship commenced.

D) Yes, because the sexual relationship began after the client-lawyer relationship commenced. Answer choice D is correct. A lawyer is prohibited from engaging in sexual relations with a client, unless a consensual sexual relationship existed between the lawyer and client when the client-lawyer relationship commenced. Here, the sexual relationship began after the client-lawyer relationship commenced, therefore the attorney is subject to discipline. For this reason, answer choice B is incorrect. Answer choice A is incorrect because a client is not permitted to waive this conflict. Answer choice C is incorrect because a lawyer is not always prohibited from having sexual relations with a client. Sexual relations are permitted if a sexual relationship existed when the client-lawyer relationship began.

An attorney assisted an elderly woman in drafting her will one year ago, and the representation terminated once the will was finalized. Recently, the woman's adult daughter, from whom the woman had previously been estranged, contacted the woman. After reuniting, the daughter discovered that the woman no longer recognized her. Concerned that the woman may no longer recognize her own children, the daughter reasonably believed that the woman may not have been able to act in her own best interests when she executed her will because she lacked testamentary capacity. For this reason, the daughter asked the attorney to disclose the contents of the woman's will. The woman has not asked the attorney to disclose her will to her daughter, and the attorney has refused to disclose the will to the woman's daughter. The attorney has not given the daughter any other information or legal advice. Was it proper for the attorney to refuse to disclose the woman's will to her daughter? A) No, because the attorney should have informed the daughter that she could seek the appointment of a guardian for the woman. B) No, because the woman's daughter reasonably believes that the woman may not have been acting in her own best interests. C) Yes, because the attorney-client relationship terminated a year ago when the will was finalized. D) Yes, because the woman did not consent to disclosing her will to her daughter.

D) Yes, because the woman did not consent to disclosing her will to her daughter. Answer choice D is correct. The attorney acted properly in refusing to disclose confidential client information to the daughter because the woman did not consent to disclosure and no exception to disclosure under Model Rule 1.6 applies. Answer choice A is incorrect. Although the daughter may be concerned about her mother's mental capacity, the facts do not indicate that the attorney shared her concern or that woman is at risk of substantial physical, financial, or other harm. Therefore, the attorney had no obligation to inform the daughter that she could seek the appointment of a guardian for the woman. Answer choice B is incorrect. Disclosure of confidential client information is not justified simply because the daughter reasonably believes that the woman may not have been acting in her own best interests. Under Model Rule 1.14(b), a lawyer may take reasonably necessary protective action, such as disclosing confidential client information, only when the lawyer reasonably believes that the client has diminished capacity and is at risk of substantial harm. The facts do not support the attorney's taking such protective action and therefore disclosure of confidential client information would be improper. Answer choice C is incorrect. A lawyer's duty to preserve confidential client information does not end when the lawyer completes work on behalf of the client.

An attorney successfully defended a sole proprietor in an age discrimination action. The sole proprietor recommended the attorney to a successful businessperson who had been served with a similar discrimination complaint. The businessperson arranged for a consultation with the attorney. The attorney listened as the businessperson described the circumstances that gave rise to the complaint, but, deciding that he would rather play golf than handle the matter, declined to represent the businessperson. The attorney did not give the businessperson a reason for declining the case, but did give the businessperson the names of three other local attorneys who were competent to the handle the matter. Was the attorney's conduct proper? A) No, because the attorney did not have a good reason for refusing to represent the businessperson. B) No, because the attorney failed to inform the businessperson as to the reason for declining to represent the businessperson. C) Yes, because the businessperson was not a person of limited means. D) Yes, because there is no duty to accept representation of any client unless the court appoints an attorney to represent a client.

D) Yes, because there is no duty to accept representation of any client unless the court appoints an attorney to represent a client. Answer choice D is correct. In general, a lawyer is not under a duty to accept representation of any client. Answer choice A is incorrect because, although a lawyer does have a duty to accept court appointments unless a good cause exists to decline the representation, a lawyer otherwise is not generally required to have a good cause for declining to represent a client. Answer choice B is incorrect because a lawyer is not required to disclose to a potential client the lawyer's reason(s) for declining to represent the potential client. Answer choice C is incorrect because, while Rule 6.1 suggests that a lawyer should offer at least fifty hours of pro bono legal services per year, and that a substantial majority of the fifty hours of such legal services be provided to persons of limited means or to organizations designed primarily to address the needs of persons of limited means, this rule does not require a lawyer to accept representation of any particular person who is of limited means.

FILL IN THE BLANKS. A lawyer may not contact a person that the lawyer __________ to be represented by counsel. A lawyer may not __________ a person who is unrepresented by counsel. A) reasonably believes, contact B) reasonably believes, mislead C) knows, contact D) knows, mislead

D) knows, mislead


Related study sets

[BSC1005] Extra Credit 4 - Answers

View Set

Chapter 7 Photosynthesis: Using Light to Make Food

View Set

Chapter 17: Pregnancy at Risk: Pregnancy-Related Complications

View Set

NCLEX 6th Edition/ Maternal-NB/ Intrapartum

View Set

Chapter 4.2 Main Idea Review (Civics)

View Set